Sie sind auf Seite 1von 181

Pre NEET

Pediatrics

Taruna Mehra
MBBS MD PEDIATRICS (MAMC)

JAYPEE BROTHERS MEDICAL PUBLISHERS (P) LTD


New Delhi Panama City London Dhaka Kathmandu

Jaypee Brothers Medical Publishers (P) Ltd


Headquarters
Jaypee Brothers Medical Publishers (P) Ltd
4838/24, Ansari Road, Daryaganj
New Delhi 110 002, India
Phone: +91-11-43574357
Fax: +91-11-43574314
Email: jaypee@jaypeebrothers.com
Overseas Offices
J.P. Medical Ltd
83, Victoria Street, London
SW1H 0HW (UK)
Phone: +44-2031708910
Fax: +02-03-0086180
Email: info@jpmedpub.com

Jaypee-Highlights Medical Publishers Inc.


City of Knowledge, Bld. 237, Clayton
Panama City, Panama
Phone: +507-301-0496
Fax: +507-301-0499
Email: cservice@jphmedical.com

Jaypee Brothers Medical Publishers (P) Ltd


17/1-B Babar Road, Block-B, Shaymali
Mohammadpur, Dhaka-1207
Bangladesh
Mobile: +08801912003485
Email: jaypeedhaka@gmail.com
Website: www.jaypeebrothers.com
Website: www.jaypeedigital.com

Jaypee Brothers Medical Publishers (P) Ltd


Shorakhute, Kathmandu
Nepal
Phone: +00977-9841528578
Email: jaypee.nepal@gmail.com

2013, Jaypee Brothers Medical Publishers


All rights reserved. No part of this book may be reproduced in any form or by any means
without the prior permission of the publisher.
Inquiries for bulk sales may be solicited at: jaypee@jaypeebrothers.com
This book has been published in good faith that the contents provided by the author contained
herein are original, and is intended for educational purposes only. While every effort is made
to ensure accuracy of information, the publisher and the author(s) specifically disclaim any
damage, liability, or loss incurred, directly or indirectly, from the use or application of any of
the contents of this work. If not specifically stated, all figures and tables are courtesy of the
author(s). Where appropriate, the readers should consult with a specialist or contact the
manufacturer of the drug or device.
Pre NEET Pediatrics
First Edition: 2013
ISBN : 978-93-5090-314-8
Printed at

Dedication
I dedicate my work to my teachers,my parents
and most importantly my patients.

Preface
Why should I change anything in your life till the time you decide to
change yourself
Quran
With the challenging task ahead in month of November and December
with all the major exams within a period of 20 days, the committee
has decided to release Pre NEET Pediatrics inclusive of DNB questions
and important last minute revision points so that you are confident in
attempting maximum questions in all the exams.This book has been
written keeping in mind that maximum time taken to revise pediatrics
is less than one day.
All the best may God help you cross the bridge

Taruna Mehra
Do not count the days make the days count

From the Publishers Desk


We request all the readers to provide us their valuable suggestions/errors
(if any) at:
jaypeemcqproduction@gmail.com
so as to help us in further improvement of this book in
the subsequent edition.

Contents
1. Pediatrics in Last Minutes ..........................

1 65

2. Pre NEET Pediatric Questions ....................

66 77

3. Pre NEET Pediatric Answers .......................

78 150

4. Previous Years Questions of DNB .............. 151 174

PEDIATRICS
in Last Minutes
Developmental Milestones
Age

Gross motor

Fine motor

Language

1 mo

Momentarily lifts Has tight grasp, Responds to


head when prone follows objects sound of bell
to midline

2 mo

Holds head in
No longer
midline. Lifts
clenches fist
chest when prone tightly, follows
objects past
midline

3 mo

Head holding
achieved

Social
Regards face
intently

Smiles after
Recognize
being stoked or parent, social
talked to
smile

Reaches for
familiar people
or objects,
anticipates
feeding

Follows moving
objects in a
circular fashion,
converges and
focuses

Coos (produces
vowel sounds
in a musical
fashion), laughs
aloud

4-5 mo Rolls over, sits


with support

Grasps objects/
rattle crudely

Enjoys looking
Orients to
around
voice/bell
environment
(localizes
laterlly);
"ahgoo", razzes

6 mo

Transfers
objects from
one hand to
another

Babbles

Stranger
anxiety, smiles
back at mirror
image of self

Responds to
name, says
mama/dada
(non-specific)

Responds to
social play,
plays pat-acake, starts to
explore
environment
(9-10 mo)

9 mo

Takes foot to
mouth, lifts head
and upper chest
with support

Uses pincer
Sits unassisted (8
grasp, probes
mo), Crawls,
with forefinger,
cruises, pulls to
holds feeding
stand
bottle

Contd...

Pre-NEET Pediatrics

Contd...
Age

Gross motor

Fine motor

10 mo Pulls from supine


to sitting; from
sitting to
standing, stands
holding furniture

Language

Social

Understands to
some "mama"

Gives hand held Says 'mama,


Imitates actions
objects to
dada' (specific)
mother when
asked, turns 2-3
pages at a time

12 mo

15 mo Walks well
without support,
Walks backwards
and sideways
18 mo Starts to run,
climb stairs with
help

Uses 3-5 words Temper


meaningfully
tantrums,
separation
anxiety
Makes a tower
of 3-4 cubes,
scribbles
spontaneously,
may draw a
vertical line

About 10 words Copies parents


in tasks, toilet
spoken
including name training started

2 yrs

Runs well, climbs


stairs alone;
walks on tiptoes
(30 months)

Makes a tower
of 6-7 cubes,
turns one page
of a book at a
time, may draw
horizontal line

Points to at
least one
named body
part, simple
2 word
sentences
(2 words at 2
years)

Follows 2-step
commands

3 yrs

Pedals tricycle,
jumps with both
feet off ground,
can alternate feet
when climbing
stairs

Make a tower of
9-10 cubes,
draws (copies) a
circle

Uses plurals, 3
word
sentences (3
word at
3 years)

Dresses and
undresses
partially
buttons/
unbuttons

4 yrs

Hops, alternates
feet going
downstairs (At 4
yr hop off the
floor)

Draws (copies)
a Cross

Knows colours

Buttons
clothing fully,
plays with other
children

5 yrs

Jumps on one
foot, heel to toe
walk

Draws (copies)
a Square

Dresses
without
supervision

Max, 10-12 mo
Mand, 3-4 mo

4-5mo

18-21mo

24-30mo

Birth

30-36mo

Max, 7-9 yr
Mand, 8-10 yr

Lateral incisors

Cuspids (canines)

Firstpremolars (biscuspids)

Secondmolars (biscuspids)

First molars

Secondmolars

Third molars

fetal mo
fetal mo
fetalmo
fetal mo
fetal mo

3-4mo

5th
5th
6th
5th
6th

Central incisors

Secondary teeth

Primary Teeth
Central incisors
Lateral incisors
Cuspids (canines)
Firstmolars
Second molars

18-25 yr

14-16yr

9-10yr

12-14yr

12-13yr

12-15yr

10-11yr

9-10yr

1824 mo
1824 mo
3036 mo
2430 mo
36 mo

Complete At

Calcification

Begins At

17-22 yr

12-13yr

6-7yr

10-12yr

10-11yr

11-l2yr

8-9yr

7-8yr

68 mo
811 mo
1620 mo
1016 mo
2030 mo

17-22 yr

12-13yr

6-7yr

11-13yr

10-12yr

9-11yr

7-8 yr

6-7yr

57 mo
710 mo
1620 mo
1016 mo
2030 mo

78 yr
89 yr
11l2 yr
1012 yr
1012 yr

67 yr
7-8 yr
911 yr
1012 yr
1113 yr

Mandibular

Age at Shedding

Mandibulaur Maxillary

Age at Eruption
Maxillary

Pediatrics in Last Minutes


3

Pre-NEET Pediatrics

Delayed eruption is usually considered when there are no teeth


by approximately 13 months of age.

Causes of delayed eruption


Idiopathic (most common), Trisomy 21, Hypopituitarism,
Trisomy 21 (Down syndrome)

Hypothyroidism, Familial, Hypoparathyroidism,


Cleidocranial dysplasia Remember
Central incisors is first to develop in primary dentition.

1st molar is first to develop in secondary dentition.


Second molar is last to develop in primary dentition.

Third molar is last to develop in secondary dentition.


Weight

10% of body weight lost in first few days of life;


regained by 2 weeks. Birth weight doubles by 4
months, triples by 12 months, quadruples by 24
months.

Height

Height is increased by 50% at 1 year of age, doubles


at 4 years and triples at 13 years.

Head circumference

Measured during 1-3 years of life 5 cm growth during


age 0-3 months; 4 cm in 3-6 months, 2cm in 6-9
months and 1cm in 9-12 months. (newborn = 35 cm;
3 months = 40 cm, 9 months 45 cm, 3 years =50 cm,
9 years = 55cm.

SURVEILLANCE OF GROWHAND DEVELOPMENT


Surveillance of growth and development is an important component
of the routine anticipatory care of children.
The main purpose of growth surveillance is to identify those children
who are not growing normally. Surveillance for physical growth can
be done in following ways.
1. Weight for age

Measurement of weight and rate of gain in weight are the


best single parameters for assessing physical growth.
The weight should be carefully repeated at intervals:
Birth - 1 year Monthly
Second year Every two months

2-5 years Every 3 months

Pediatrics in Last Minutes

These measurements when compared with the reference


standards of weight of children of same age, the trend of
growth becomes obvious.

Weight for age can be used to classify malnutrition and


determine its prevalence.
80 % of the median weight for age of the reference is cut off
point below which children should be considered
malnourished.

2. Height for age


Height is a stable measurement of growth as opposed to body
weight.

Whereas weight reflects only the present health status of the


child, height indicates the events in the past also. Low height
for age.
This is also known as nutritional stunting or dwarfing.
It reflects past or chronic malnutrition.
The cut off point commonly taken for the diagnosis of stunting
is 90 percent of the united states NCHS height for age.

3. Weight for height


Weight in relation to height is now considered more important
than weight alone. It helps to determine whether a child is
within range of normal weight for his height.

Low weight for height


This is also known as nutritional wasting or emaciation (Acute
malnutrition).

It is associated with increased risk of mortality and morbidity.


A child who is less than 70% of the expected weight for height
is classed as severely wasted.

4. Head and chest circumference


Chest circumference
At birthLess than 2 cm from head circumference
6-9 monthsTwo measurements become equal

>69 monthsOvertakes head circumference


In severely malnourished children this overtaking may be
delayed by 3 to 4 years.

Pre-NEET Pediatrics

Wasting (deficit in weight for height) Acute malnutrition

Stunting (deficit in height for age) Chronic malnutrition


Wasting and stunting Acute on chronic malnutrition

Underweight (low weight for age) Combined indicator to


reflect both acute and chronic malnutrition.

WHO Classification of Under Nutrition


Moderate undernutrition
Weight for
height (wasting)
Height for age
(stunting)

SD score -2 to -3
(70-79% of expected)
SD score -2 to -3
(85-89% of expected)

Severe undernutrition
SD score <-3
(<70%
of
expected)
SD score <-3
(<85% of expected)

Age-Independent Anthropometric Indices

Dugdales
Raos
Kanawati
MAC
(1-5 years)

Weight(kg)/ (height in cm)1.6 X 100


Weight(kg)/ (height in cm)2 X 100
Mid arm circumference (cm)/
Head circumference (cm)
Midarm circumference

Normal

Severely
malnourished

0.88-0.97
0.15-0.16
0.32-0.33

<0.79
<0.14
d0.25 cm

>13.5

<12.5 cm

Gomez classification is a classification system to assess


Protein Energy Malnutrition of PEM
It is based on weight retardation

The child on the basis of his/her weight is compared with a normal


child of the same age
The normal reference child is the 50th centile of the Boston
standards

Weight for age (%):


90-110%:
75 89%:
60 74%:
Under 60%:

Weight of child
100
Weight of normal child of same age

Normal Nutritional status


Mild Malnutrition (1st degree)
Moderate Malnutrition (2nd degree)
Severe Malnutrition (3rd degree)

Pediatrics in Last Minutes

The Gomez system has distinct advantages and disadvantages


Advantages

Disadvantages

Classification is easy to compute as


weight is a widely recorded parameter

Classification has prognostic values


for hospitalized patients (This is
because the cut off values were set
during a study of risk of death
based on weight for age at admission
to a hospital unit)

Some normal children may be


classified as 1st degree malnutrition
because a cut-off point of 90
percent of reference is high.
Does not distinguish between
sudden acute episode of
malnutrition and along standing
chronic malnutrition as observation
only consider weight for age
measurements.

Facts to remember regarding other classification systems for


PEM:

Indian academy of Pediatrics (IAP) classification system is based


on defecit in weight for age

Wellcome Trust classification system is based on deficit weight for


age and presence or absence of edema.

Waterlow classification is based on Height for age (Stunting /


Chronic PEM) measurments and Weight for Height measurments
(Wasting /Acute)
Drop in Height for Age Ratio Chronic PEM or Stunting
Drop in Weight for Height ratio acute PEM or Wasting

WHO classification is also based on Height for Age and Weight


for Height measurement.

Recommended daily intake of energy


Group

Body weight Kg.

Infancy
0-6 months
7-12 months
Children
1- 3 years
4- 6 years
7 - 9 years

Energy allowance per day


K.Cals
MJ

118
108 } K.Cal/kg/day

12.03
18.87
26.37

1240
1690
1950

5.1
7.0
8.1

Pre-NEET Pediatrics

Table Comparison of Human Milk and Cows Milk


Parameter
Bacterial contamination
Anti-infective substances

Protein
Total
Casein
Amino acids
Cystine
Taurine
Fat
Total
Saturation of fatty acids
Linoleic acid (essential)
Cholesterol
Lipase to digest fat
Lactose
Salts (mEq/1)
Sodium
Chloride
Potassium
Minerals (mEq/Il)
Calcium
Phosphate
Iron

Vitamin

Human
None
Antibodies, Leukocytes,
Lactoferrin Bifidus factor,
Others

Cow
Likely
Not active

1%
0.5%

4% (too much)
3% (too much)

Enough for growing brain


Enough for brain, retina
and bile acid conjugation

Not enough

4% (average)
Enough unsaturated
Enough for growing brain
Enough
Present
7% (enough)

4%
Too much saturated
Not enough
Not enough
None
4.5%

6.5 (correct amount)


12 (correct amount)
14 (correct amount)

25 (too much)
29 (too much)
35 (too much)

350 (correct amount)


150 (correct amount)
Small amount, but wellabsorbed (enough)

1,400 (too much)


900 (too much)
Small amount,
poorly absorbed
(not enough)
Extra needed

Enough

Pediatrics in Last Minutes

Approach to Short Stature

Bone-Age
Bone age is delayed compared to chronological age:

In almost all cases of short stature

In case of constitutional delay, undernutrition and systemic illness,


bone age is less than chronological age and equals height age.

In case of growth hormone deficiency and hypothyroidism, bone


age may be even less than height age if the endocrine condition is
diagnosed late.

Advanced bone age is common when a child has had


prolonged elevation of sex steroid levels, as in:

10

Pre-NEET Pediatrics

Precocious puberty or congenital adrenal hyperplasia (A delayed


bone age in congenital adrenal hyperplasia indicates glucocorticoid
over-treatment.

Bone age may be significantly advanced in genetic overgrowth


syndromes, such as Sotos syndrome, Beckwith wideman
syndrome, and Marshall Smith syndrome.

Delayed Bone age (Retarded skeletal maturation)

Chronic ill Health

Congenital heart disease - cyanotic


Renal failure
Inflammatory bowel disease
Malnutrition
Rickets
Maternal deprivation
Any other chronic illnesses

Endocrine Disorders

Hypothyroidism
Steroid therapy and Cushings disease
Hypogonadism (including Turners syndrome)
Hypopituitarism - panhypopituitarism, growth hormone deficiency,
Laron dwarfism

Chromosome disorders

Trisomy 21, Trisomy 18

Other Congenital Disorders

Bone dysplasias
Malformation syndromes

Advanced Bone Age (Generalized Accelerated Skeletal


Maturation)
Endocrine disorders

Adrenal and gonadal tumours

Pediatrics in Last Minutes

11

Hyperthyroidism
Idiopathic sexual precocity
Intracranial masses in the region of the hypothalamus (haniartoma,
astrocytoma, optic chiasm glioma, hydrocephalus, encephalitis)

Congenital Disorders

McCune Albright syndrome: polyostotic fibrous dysplasia with


precocious puberty
Cerebral gigantism (Sotos syndrome)
Lipodystrophy
Pseudohypoparathyroidism
Acrodysostosis
Weaver smith syndrome
Marshall syndrome

D/D of Rickets
Serum Ca

Serum
P043

Serum
Alkaline

Serum
Serum
Parathormone HCO 3
phosphatase

Normal

9-10.5 mg/dl

3-4.5mg/dl

30-120 IU

10-55 units

Hypophosphatemic
Rickets
Vit D dependent
Rickets
Hyperparathyroidism
Nutritional rickets
Renal tubular
acidosis (distal)

21-30
meqlL
N

N/ (7.5-8)

N/

/N

N
N

Enuresis

Enuresis is defined as the voluntary or involuntary repeated


discharge of urine into clothes or bed after a developmental age
when bladder control should be established (mostly mental age
of 5 years).
Diagnosis of enuresis requires voiding of urine twice a week for
3 consecutive months or clinically significant distress in childs
life as a result of wetting.
Enuresis is more common in males than females
Most common cause of diurnal enuresis is micturition deferral
(waiting until the last minute to void).

12

Pre-NEET Pediatrics

Treatment

First line treatment for enuresis is behavioral therapy. It consists of


rewarding the child for being dry at night, child should void before
retiring and the use of conditioning devices (e.g. bed alarm that
rings when the child wets a special sheet).

Consistent dry bed training with positive reinforcement has a


success rate of 85% and bed and pad alarm systems have a success
rate of approximately 75% with relapse rate that are lower than
those with pharmacotherapy.

Pharmacotherapy is second line treatment and should be reserved


for those patients who have failed behavioural therapy. Imipramine
and desmopressin are two important drugs useful for enuresis.

Fast action of desmopressin (orally or intranasaly) suggests a role


for special occassions when rapid control of enuresis is required.
However, recurrence rate is very high.
Imipramine is associated with cardiac conduction disturbances
and is deadly in overdose.

Pediatrics in Last Minutes

13

PRENATAL AND PERINATAL FACTORS AFFECTING


FETUS
Drugs Taken During Pregnancy And Their Adverse
Effects - Teratogenic
Drugs
Phenytoin
Carbamazepine
Phenobarbitone
Sodium vaiproate

Corticosterojds
Diethyl stilbestrol

Anti-thyroid drugs
Clomiphene
Synthetic
progestins
Chloramphenicol
Sulphonamides
Tetracyclines
Aminoglycosides
Anti-malarials
Quinine,
chloroquine

Adverse effects
Anticonvulsants
Fetal hydantoin syndrome (microcephaly, cleft palate,
hypoplastic changes, IUGR)
Spina bifida, ? NTD
Relatively safe
Neural tube defect (1-2%), hypospadias, microstomia,
developmental delay.
Hormonal agents
Growth retardation, cleft palate and lip
(used as morning-after pill) Vaginal adenosis in female
offspring in adolescence Clear cell vaginal
adenocarcinoma in teenagers In male offspring (risk of
testicular cancer in later life)
Neonatal hypothyroidism and goiter
NTD, multiple gestation, Downs syndrome
Masculinization in female fetus, hypospadias
Antibiotics
Gray baby syndrome (peripheral vascular collapses)
Kernicterus, Methemoglobenemia
Dental discoloration (yellow) and deformity, Inhibition of
bony growth, cataracts,
Fetal ototoxicity due to eighth N. damage
Intra-uterine death
Retinopathy, congenital deafness, comeal opacities

Psychiatrics drugs and substances of abuse


Lithium
Ebstiens anomaly
LSD (lysergic
fractured chromosomes anomaly in fetus, stunted growth
acid diethylamide)
Alcohol
Foetal alcohol syndrome: prenatal-onset growth deficiency,
developmental delay, facial dysmorphism (short palpebral
fissures, ptosis, strabismus, ear abnormality, long philtrum
with a thin upper lip), multiple joint anomalies and cardiac
defects (ASD>VSD); mental subnormality
Contd...

14

Pre-NEET Pediatrics

Contd...
Heroin
Cocaine
Beta-blockers

Vitamin K
(large dose)
Warfarin
Aspirin

Irritability, hyperactivity, tremors


Abruptio placentae, preterm labour, cerebral infarction
Foetal bradycardia
Anticoagulants
Hyperbilirubinemia (hemolysis) and kernicterus
Conradis syndrome: skeletal and facial anomalies,
chondrodysplasia punctata, haemorrhage
Haemorrhagic disease of newborn

Other Drugs
Cytotoxic drugs

Multiple foetal malformations and abortion

Isotretinoin

CNS defects, facial palsy, deafness, cardiac defects

Oxygen in high

Retrolentalfibroplasia and blindness (>35%)

concentrations
Thalidomide

Phocomelia (seal limbs), cardiac malformations

Thiazide diuretics

Neonatal thrombocytopenia

ACE inhibitors

Renal tubular dysgenesis, lung hypoplasia, anuria,


oligohydramnios

Misoprostol

Mobius syndrome, arthrogryposis

(prostaglandin)
Vitamin D

William syndrome (infantile hypercalcemia,


supravalvular aortic stenosis, elfin facies)

Drugs taken during perinatal/neonatal period and their effects on the neonate
Oxytocin (used for induction of labour)

Hyperbilirubinemia in babies

Prolonged cortisone

Adrenal crisis in infants

NSAIDS

Premature closure of ductus


arteriosus

Dexamethasone

Periventricular leuocmalacia
Medications to baby

Chloramphenicol

Grey baby syndrome

Erythromycin
Vitamin K

Pyloric stenosis
Bleeding, hepatotoxicity

Pediatrics in Last Minutes

15

Maternal Conditions And Fetal Outcome


Disease
Bronchial asthma
Chronic cardiac disease
Chronic Crenal disease
Hypertension
Thyroid disorders

SLE
Smoking

Outcome/Comment
IUGR; Feal goiter and hypothyroidism (due to drugsbeta agonists)
IUGR, abortion, asphyxia, prematurity
IUGR, prematurity
Placental vasculopathy, IUGR
Maternal hypothyroidism may cause congenital
hypothyroidism (TSH does NOT cross placental, barrir
but LATS does)
Congenital complete heart block, IUGR
IUGR/LBW; Sudden infant death syndrome (SIDS);
Increased ororfacial clefts in the fetus. Developmental
lag for first few years of life: adverse effects on
language skills and visual and spatial abilities.

NEONATOLOGY
Five Cleans of Intranatal Care

Clean hands
Clean delivery surface
Clean blade
Clean cord
Clean tie (for cord)

AAP- AHA Guidelines for Neonatal Resuscitation

Initial steps DO NOT include giving supplemental oxygen. if


cyanosis persist despite giving free flow oxygen, give positive
pressure ventilation

Routine intrapartum oropharyngeal and nasopharyngeal


suctioning of babies born through meconium stained liquor no
longer advisable

For Term Babies

Use of 100% oxygen is recommended when baby is cyanotic or


when positive pressure ventilation is required during neonatal
resuscitation
In situations where supplementary oxygen is not readily available,
positive pressure ventilation should be started with room air.

16

Pre-NEET Pediatrics

For Preterm Babies

Begin PPV with oxygen concentration between room air and 100%
oxygen
Increase oxygen concentration up or down to achieve saturation
between 90 and 95%
If heart rate does not response by increasing rapidly to 100 per
minute, correct any ventilation problem and use 100% oxygen

Regarding PPV
LMA should NOT be used
In the setting of meconium stained amniotic fluid
When chest compression is required
In VLBW babies
For delivery of medications
Naloxone NOT to be given by ET route; epinephrine preferably by
intravenous route only
Capnography (exhaled CO2) recommended for confirming ET
tube placement.
Normal APGAR sore = 8-10 at minute after birth; <7 indicates
asphyxia.
NO NEED for resuscitation if these 5 criteria are fulfilled by
newborn:
Full term, crying, clear of meconium, pink, good muscle tone.
2 absolute indications for bag and mask ventilation are
Apnea at birth; HR < 100/minute.
Absolute contraindications for bag and mask ventilation are:
Diaphragmatic hernia, meconium aspiration.
Indications to start chest compression are:
HR < 60; ventilate with oxygen for a full 30 seconds initially
and proceed with chest compressions if the HR remains <60/
mm.
Indications for endotracheal intubation are:
Prolonged PPV; Ineffective bag and mask ventilation;
Diaphragmatic hernia, meconium aspiration.

Pediatrics in Last Minutes

17

Drugs which can be given endotracheally are:


Naloxone, Adrenaline, Lignocaine, Atropine (NALA).

DO NOT Resuscitate if
When gestation birth weight or congenital anomalies are a/w
almost certain death (class Ha)
Anencephaly
Chromosomal anomalies incompatible with life (e.g., trisomy 13)
APGAR Score

Appearance (Colour)

Blue or pale

Body pink,

Pink

extremities
blue
Pulse (Heart rate)

<100

> 100

Grimace to catheter in nose

No Response

Grimace

Cries, Coughs

Flaccid

Some

Actively

flexion

moving

(Reflex stimulation)

or Sneezes

Activity (Muscle Tone)

extremities
Respiratory effort (not rate)

Slow

Good crying

irregular

Timing of Selected Primitive Reflexes


Reflex

Onset

Fully developed at

Persist till

Palmar grasp

28 wk

32 wk

2-3 months

32 wk

36 wk

Less

(grasps objects in palm)


Rooting (nipple seeking)

prominent after
1 month
Moro (extenson of

28-32 wk

37 wk

5-6 months

35 wk
7-8 months

1 month
10-11 months

6-7 months
Persists
throughout life

limbs when startled)


Tonic neck
Parachute

18

Pre-NEET Pediatrics

Birth Trauma
Caput succedaneum
Present at birth;
May extend over suture lines;
Diffuse, ecchymotic, oedematous
overlying skin;
Disappears spontaneously within
24hrs after birth;
No treatment needed

Cephalhematoma
Seen at 2-4 days old;
Limited by suture lines;
Normal overlying skin;

Reabsorbed over 2 weeks to 3


months; can calcify;
May require phototherapy for
jaundice
1. # Clavicle MC bone fractured due to birth trauma; appears at 1-20+ days
2. Sternomastoid tumor appears at 7-20+ days. Intrauterine posture is also a
cause. Located at junction of upper and middle third of muscle. Disappears by
6 months of age.
3. Brain MC internal organ to be injured during birth; next MC is liver

Umbilical Cord
Usually cord is cut 2.5 inches or 6 cm from the umbilical base.
Contains 2 arteries and 1 vein.
Single umbilical artery
Is a/w genitourinary anomalies (MC), CVS anomalies,
esophageal atresia, trache-esophageal fistula, imperforate
anus

Incidence is 1%, also a/w DM, prematurity, asphyxia.


Cord clamping
Early cord clamping done in
Prematurity, Rh incompatibility, birth asphyxia, IUGR,
baby of diabetic mother.
Delayed cord Clamping done in
Prematurity and cord around neck

Cord blood is useful for


Estimation of TSH, T3 and T4; in screening of congenital
hypothyroidism (TSH levels > 50micrU/ml in cord blood are
diagnostic of neonatal hypothyroidism).
Screening for inborn errors of metabolism/tandem mass
spectroscopy (TMS); e.g., in PKU, cystic fibrosis, G6PD
deficiency.
Sampling of and infant born to Rh positive mother: or study of
blood group, Rh type, serum bilirubin, Coombs test.

Pediatrics in Last Minutes

19

Neonatal sepsis

Early onset neonatal sepsis occurs within 72 hours of life and


is caused by organisms prevalent in the maternal genital tract or
in the labour room. Group B Streptococci are the MC organisms
(kelbsiella, E.coli and S.aureus are MC in India).
Late onset neonatal sepsis is a nosocomial infection acquired
from the nursery or lying in ward and occurs 72 hours after
birth. Gram negative bacilli are MC organisms.
Sepsis screen includes (usually 2 markers are considered
significant)
a. ANC<1000
b. Leucopenia < 5000/mm3
c. Band cell to neutrophil ratio or I:T ratio (immature to total
polymorphs) > 0.2 (band cells > 20%)
d. CRP> 8microgm/ml is positive
e. Micro ESR> 15 mm/first hour
f. Presence of > 5 PMN/hpf in gastric aspirate (limited utility in
blood stained or meconium stained liquor)
Other markers - procalcitonin and cytokines
Blood culture is confirmatory
Listeria infection may produce preterm delivery,
intrauterine death
Early conjugated hyperbilirubinemia with bilirubin > 0.5
mg/dl is noted in neonatal sepsis.

Hypoxic Ischemic Encephalopathy (HIE)

Initial response to hypoxia is increased cerebral blood flow due to


redistribution of cardiac output by the diving reflex and rise in
BP. Cerebral edema aggravates HIE.
Status marmoratus is seen in kernicterus and due to basal ganglia
defect.
In preterm babies deeper vessels are deficient and hence they
develop periventricular ischemia and leukomalacia which leads
to spastic diplegia. Disability is more in LL
Sarnat and Sarnat staging is use for HIE - useful only when baby
> 36 weeks.

20

Pre-NEET Pediatrics
Preterm babies
Can tolerate hypoxia for longer periods without sequelae
Suffer more periventricular ischemia as cortical vessels are
more superficial
Periventricular insults are more common than term babies

Term babies
Suffer more cortical ischemia and infarcts
It may lead to multifocal necrosis, porencephalic cyst,
hydrencephaly

ROP screening criteria

All infants born at d 32 weeks.

All infants with birth weight d 1500 grams

Very sick babies who require ventilatory support, multiple blood


transfusions.

Prematurity + oxygen toxicity predisposes to

Retinopathy of prematurity (ROP)

Bronchopulmonary dysplasia (BPD)

Antenatal steroid therapy (ANS)

It is a/w reduced incidence of RDS/HMD, NEC

Postnatal corticosteroids

Useful in prevention of HMD, BPD, but recent data are


inconclusive and not recommended anymore.

They have some therapeutic utility in management of sclerema


neonatorum

Excess administration of vitamin E is a/w


Intracranial hemorrhage
NEC
Deficiency of Vitamin E is a/w
ROP
BPD
Hemolytic anemia

Pediatrics in Last Minutes

21

Causes of Impaired Bilirubin Conjugation/


Unconjugated Hyperbilirubinemia
Physiological jaundice of newborn

- Decreased UGT activity

Breast milk jaundice

- Inhibition of UGT activity bilirubin

Crigler Najjar syndrome

- Genetic deficiency of bitirubin UGT


activity

Type I

- Autosomal recessive /Absent UGT


activity

Type II

- Autosomal dominant! Decreased


UGT activity

Gilbert syndrome
Diffuse hepatocellular disease

- Decreased UGT activity due to mixed


etiologies

Two other syndromes are often asked.

These are associated with

Dubin Johnson syndrome Q

Impaired biliary excretion of bilirubin

Conjugated hyperbilirubinemia.
(Autosomal Recessive)

glucuronides due to Canalicular


membrane carrier defect

Rotors syndromeQ
(Autosomal Recessive)

Decreased hepatic uptake and


storage, or possibly decreased biliary
excretion?

Cause of Late Onset Jaundice After 72 Hours of Age


and Within First 2 weeks

Breast milk jaundice

Biliary atresia

Hypothyroidism

Gilberts syndrome

Infection: UTI, Herpes, Hepatitis

Parenteral alimentation in VLBW

Metabolic diseases like galactosemia, alpha 1 antitrypsin


deficiency, cystic Fibrosis, hereditary fructose intolerance
tyrosenemia

22

Pre-NEET Pediatrics

Guidelines for Phototherapy and Exchange transfusion in a


term Neonate
Age

Total serum bilirubin (mg/dl)


Phototherapy
Exchange transfusion
15
18
20

24-48hrs
48-72hrs
72hrs

20
25
25

Guidelines for Management of Jaundice in Preterm babies


Total serum bilirubin (TSB) (mg/dL)
Gestation and birth
weight Preterm babies

Healthy baby
Phototherapy
Exchange
transfusion

Sick baby
Phototherapy Exchange
transfusion

<1000
1001-1500g
1501-2000g
2001-2500g
Term babies

5-7
7-10
10-12
12-15

11-13
13-15
15-18
18-20

4-6
6-8
8-10
10-12

10-12
11-13
13-15
15-18

>2500g

15-18

20-25

12-15

18-20

Causes of Hydrops Fetalis

Immune

Rh incompatability

Non-immune
Anemia
Cardiac dysarrhythmias

Structual cardiac defects

Vascular

Lymphatic

CNS
Thoracic

-thalassemia, G-6PD deficiency


Supraventricular tachycardia, AF,
congenital heart block
Tricuspid insufficiency, endocardial
cushion defect, cardiomyopathy,
hypoplastic left heart, premature
closure of foramen ovale
Chorioangioma of placenta, Twin-Twin
transfusion, umblical artery aneurysm,
thrombosis of renal or umbilical vein or
IVC
Lymphangiectasia, cystic hygroma,
Noonan syndrome
Encephalocele, intracranial hemorrhage
Mediastinal teratoma, diaphragmatic
hernia
Contd...

Pediatrics in Last Minutes

23

Contd...

Teratomas
Tumor & storage diseases

Chromosomal
Bone diseases

Congenital infections

Others

Choriocarcinoma, sacrococcygeal teratoma


Neuroblastoma, hepatoblastoma, Gaucher
disease, Niemann-Pick disease,
Mucopolysaccharidosis
Trisomy 13, 15, 16, 18, 21
Osteogenesis imperfecta, skeletal
dysplasias
CMV, rubella, Toxoplasmosis, Syphilis,
Parvovirus, Leptospirosis, disease
Congenital nephrosis, Myotonic dystrophy,
Infant of diabetic mother, Maternal therapy
with indomethacin, Hepatic fibrosis

Erythroblastosis Fetalis

Haemolytic disease of the newborn (erythroblastosis fetalis)


is caused by blood group incompatibility between the mother and
fetus.
Maternal IgG crosses the placenta and destroys fetal erythrocytes.

An Rh - mother carrying an Rh + fetus is at highest risk


Risk increases when fetal blood crosses into the maternal
circulation as in abortion, ectopic pregnancy, amniocentesis and
motor vehicle accidents.

Affected neonates present with: anemia, hvperbilirubinemia


hepatosplenomegaly, pulmonary edema and ascites.
The direct Coombs test is positive.

Treat with exchange transfusions (see below).


Prevent disease with anti-Rh IgG injections for high-risk mothers
at 28 weeks of delivery, and at any other time of exposure to fetal
blood.

Indications for exchange blood transfusion in infants


with Rh- hemolytic disease of the newborn

Cord Hb of 10g/dl or less


Cord bilirubin of 5 mg/dl or more

Unconjugated serum bilirubin of 10 mg/dl within 24 hours or 15


mg/dl within 48 hours or rate of rise of >0.5 mg/dl per hour

24

Pre-NEET Pediatrics

INFANT OF DIABETIC MOTHER


Congenital Defects

Cardiac (VSD, ASD, TGA Coarctation of aorta)


Neural tube defect
Holoprosencephaly
Sacral agenesis (most specific)
Hydronephrosis
Renal agenesis
Duodenal atresia
Anorectal malformations

General

Macrosomia
Normal head size

Increased subcutaneous fat


Birth trauma
Hairy pinna

Other

Renal vein thrombosis


Respiratory distress syndrome
Polycythemia
Small (lazy) left colon syn.

Cardiovascular

Cardiomegaly
Transient hypertonic cardiomyopathy
Persistent fetal circulation

Metabolic

Hypoglycemia
Hypocalcemia
Hypomagnesemia
Hyperbilirubinemia

Pediatrics in Last Minutes

25

D/D of Neonatal Vomiting


First 1 to 3 days
Very common
Feeding problem

End of first week


Common
Hiatus hernia

Gastric irritation, swallowed blood. Infection


Functional ileus in premature,

Less common

stressed.
Infection

Necrotizing enterocolitis

Neurological: asphyxia/birth injury, Obstructive: pyloric stenosis,


intraventricular haemorrhage.

volvulus, anal stenosis, small left colon,


Hirschsprungs disease.

Uncommon

Uncommon

Obstruction: Duodenal atresia,

Renal failure, CAH, galactosemia,

stricture or web, annular pancreas,

organic acidosis, lactic acidosis.

Ladds bands, mid gut volvulus


Meconium ileus, Meconium plug,
Hirchsprings disease, Anal atresia.

Neonatal Hypoglycemia
The whole blood glucose level of < 40 mg/dL indicates hypoglycemia
1. Transient hypoglycemia

Prematurity; small for date infants; infant of diabetic mother;


smaller of twins.
2. Persistent hypoglycemia

Hyper-insulinemia, Nesidioblastosis, Adenoma of beta cells,


Beckwith syndrome, leucine sensitivity.
Deficiency of hormones such as glucagons, growth hormone,
epinephrine, ACTH.
Deficiency of substrate as in ketotic hypoglycemia and Maple
syrup urine disease.
Disorders of carbohydrate metabolism such as glycogen
storage disease, galactosemia, and fructose intolerance.

3. Other etiologies
Idiopathic; Sepsis; Drugs (maternal tolbutamide); liver disease
(reye syndrome), carcinoma etc.

26

Pre-NEET Pediatrics

Age of Onset of Convulsions


First day

Between 1-3 days

4th to 7th day

Hypoxic-ischaemic
encephalopathy
Birth injury (cerebral contusion)
First day hypocalcemia
(normal phosphate)
Pyridoxine dependency
Accidental injection of
local anesthetic

Intracranial haemorrhage

Tetany

Hypoglycemia
Narcotic withdrawal

Meningitis
TORCH infections

Inborn errors of metabolism Developmental


Kernicterus
Idiopathic

Head Malformations
1. Anencephaly: Due to failure of closure of the rostral neuropore
2. Holoprosencephaly: Incomplete separation of the cerebral
hemispheres. Seen in Pataus syndrome.
3. Porencephaly Cysts or cavities in the brain may result from
developmental defect or acquired lesions including infarction of
tissue
4. Lissencephaly: Bat like brain with no cerebral convolutions and a
poorly formed sylvian fissure due to faulty neuroblast migration
(agyria). Hypoplasia of optic nerves and microphthalmia are
common.
5. Schizencephaly: Unilateral or bilateral cleft in the cerebral
hemispheres, microgyria.
6. Scaphocephaly: MC type of cranoisynostosis.
7. Encephalocele: Is a malformed diverticulum of CNS tissue
extending through a defect in the cranium.
8. Shapiros syndrome: Agenesis of corpus callosum
Torch Infections
Infection

Description

Treatment

Prevention

Toxoplasmosis

3Cs Convulsions,
intracranial Calcification,
Chorioretinal scar,
hydrocephalus Ring
enhancing lesions on
head CT
Deafness (MC)
Bluberry muffin rash,

Pyrimethamine,
sulfadiazine,
spiramycin

Avoid exposure to
cats and cat feces
during pregnancy;
avoid raw
undercooked meat

None

Immunize mother
prior to pregnancy

Rubella

Contd...

Pediatrics in Last Minutes

27

Contd...
Infection

Description

Treatment

Cataracts and salt and


pepper chorioretinitis
PDA and multiple
Pulmonary stenoses
encephalitis
(periventricular
calcification) maximum
fetal transmission if
iinfected between 6-8
weeks of pregnancy
Cytomegalovirus Petechial rash,
Ganciclovir
periventricular calcifications, Acyclovir
microcephaly, chorioretinitis
Herpes
Skin, eye and mouth vesicles,
can progress to severe
systemic/CNS infectioin
Syphilis
Maculopaular skin rash,
Penicillin
lymphadenopathy, snuffles
peri-osteitis Interstitial
keratitis If infected during
first trimester (maximum risk
of abortion)

Prevention

Avoid exposure
Perform a
Caesarean section
if mother has active
lesions at time of
delivery
Treat seropositive
mothers with
penicillin

Management of Neonate Born to HBSAg+ve Mother

Risk of fetal transmission after maternal exposure to HBV

Depends upon the time of exposure and status of maternal


serological markers.

Perinatal transmission occurs MC during passage through


the birth canal from infected blood and fluids at the time of
delivery.

HBV is usually transmitted at the time of birth (rarely in


utero) and commonly results in carrier state (increased
risk of cirrhosis and HCC later in life) or it may cause
severe hepatitis in infancy.

All neonates born to HBSAg +ve mother should be given


HBIG (hepatitis B Ig) 0.5 ml IM + active immunization with
HB vaccine preferably within 12 hours of delivery.

Perinatal Tuberculosis

True congenital TB is rare. Congenital TB in a neonate is acquired

28

Pre-NEET Pediatrics
by transplacental transmission through a lesion in the placenta or
ingestion of infected liquor.

Transplacental transfer MC affects the liver; hepatomegaly is usual


manifestation.
Postnatal TB is more common, usually from an open infectious
case, MC the mother.

Management
If the child has clinical features of perinatal TB

Usual treatment Give regimen of 2HRZ +7 HR


If a high risk mother delivers a baby who is asymptomatic

Usual treatment - continue breast feeding; gve BCG to


child + ATT to mother and screen the baby (if mother is
Mantoux positive, CXR and screening +ve or open case
of TB)
If screening of baby normal - give prophylactic ATT to
baby (6 HR)
If screening of baby abnormal - treat as perinatal TB

Advanced maternal Age


(>35 years) is a/w
Down syndrome
Mental subnromality
Premature labour
(Aperts syndrome)
Edward syndrome
Dizygotic twins

Advanced Paternal Age


(>50 years) is a/w
- Marfans syndrome
- Achondroplasia
- Craniosynostosis with syndactyly
- Downs syndrome (with fusion of
chromosome 21,22)
- Osteogenesis imperfecta
- Congenital deafness

HIGH YIELDING FACTS


Congenital Diaphragmatic Hernia

Bochdaleks Hernia (posterolateral hernia): This is really the


persistence of the pleuroperitoneal canal. MC diaphragmatic
hernia in children, more common on left side. Classic triad of
respiratory distress, apparent dextrocardia and a scaphoid
abdomen. Pulmonary hypoplasia is the MC cause of morbidity
and mortality.

Pediatrics in Last Minutes

29

Morgagni Hernia (retrosternal anterior hernia): Defect is between


the sternal and costal attachments of the diaphragm. Usually occurs
on right side, in adults, particularly females. MC involved viscus
is the transverse colon.

Ponderal Index (PI) In Neonate

Ponderal index has been used as an indicator of fetal growth status,


especially to assess assymetrical IUGR.
The Ponderal index is calculated by multiplying the weight in grams
by 100 and then dividing it by cube of length in cm.

Ponderal Index = Weight (gm)/Length (cm)3 x100

A ponderal index below the 10th percentile may be used to identify


IUGR infants correctly.
A low neonatal ponderal index is defined as less than 1 SD below
a mean 2.0.
PI is usually less than 2 in assymmetric growth retarded baby and
2 or more in a baby who has either normal growth or has
symmetrical growth retardation.
Fetal ponderal index can also be calculated by USG examination
and compared with neonatal P1. Fetal P1 had been found to be
predictor of IUGR with the sensitivity and specificity of 76.9 and
82%, respectively. If the fetal P1 is less than 1 SD, the fetal and
neonatal well being is compromised.

Apt Test

The Apt test is most commonly used in cases of vaginal bleeding


late during pregnancy (antepartum hemorrhage) to determine if
the bleeding is from the mother or the fetus.
Exposing the blood to NaOH (alkali) will denature the adult but
not the fetal hemoglobin. The fetal hemoglobin will appear pinkish
color under the microscope while the adult hemoglobin will appear
as a yellow - brownish color. - Positive test Blood is of fetal
origin. - Negative test Blood is of maternal origin.
The Apt test can be used after birth (post partum hemorrhage) if
the newborn has bloody vomiting, bloody stool, or active bleeding
from nasogastric tube. A positive Apt test would mean that the
blood is either due to GI or pulmonary bleeding from neonate. A
negative Apt test would indicate that the blood is of maternal

30

Pre-NEET Pediatrics
origin, suggesting that the neonate swallowed or aspirated maternal
blood, either during delivery or during breast feeding (from breast
fissures). Kleihauer Betke test (KB test)
The KB test is the standard test for detecting fetal - maternal
hemorrhage. It is used to measure the amount of fetal hemoglobin
transferred from a fetus to a mothers blood stream.
It is usually performed on Rh(-ve) mothers to determine the
required dose of Rho (D) immunoglobulin to inhibit formation of
Rh antibodies in the mother and prevent Rh hemolytic disease in
future Rh - positive children.
A standard blood smear is prepared from the mothers blood and
exposed to an acid bath. This removes adult hemoglobin, but not
fetal hemoglobin, from the red blood cells. Subsequent staining
makes fetal cells (containing HbF) appear rose pink, while adult
red cells are only seen as ghosts.
KB test quantifies the fetal - maternal hemorrhage.
Apt test

Source of sample Maternal or neonatal


How it works
Adding 1% NaOH
destroys adult HbA but
not fetal HbF
Assessment type Qualitative
Results
Positive means blood
is of fetal origin

Kleihauer Betke test


Maternal
Adding acid destroys adult
HbA but not fetal HbF
Quantitative
Reported in estimated mls of
fetal blood

Hematopoesis in Fetus

The anatomic sites of hematopoesis undergo developmental


changes during embryonic and fetal life.
RBC formation can be observed within the developing blood
vessels of the yolk sac at 2 weeeks of gestation.
By 8 weeks of gestation the site of RBC formation begins to shift
to the sinusoids of the liver, where granulocyte precursors and
megakaryocytes are also seen. Hematopoesis in the liver is
maximal till 20-24 weeks and declines thereafter.
Hematopoesis in the bone marrow is evident by the 4th month of
gestation and this becomes the predominant site of hematopoesis
during the rest of gestation and later.

Pediatrics in Last Minutes

31

Hematopoiesis After Birth

Upto puberty: It occurs throughout the seletaon.


After puberty: Only the vertebrae, ribs, sternum, skull, pelvis
and proimal epiphyseal regions of humerus and femur retain red
marrow

Circulatory Adjustments at Birth


These are brought about because of a shift from placental dependance
for gas exchange in the fetus to pulmonary gas exchange in neonate.
1. Pulmonary circulation: Immediatly after birth lungs expand
due to first few breaths of neonate.
This causes a fall in pulmonary vascular resistance (oxygen
causes pulmonary vasodilatation).
This results in increased flow into pulmonary trunk and arteries.
The pulmonary artery pressure falls due to lowering of
pulmonary vascular resistance.
The pressure relations between aorta and pulmonary trunk
are reversed so that the blood flow through the ductus
arteriosus is reversed Instead of blood flowing from the
pulmonary artery to aorta, the direction of flow through ductus,
is from aorta to pulmonary trunk.
Increasing oxygen saturation causes the muscle of ductus to
constrict In full term neonates the ductus arteriosus closes
within 10 to 25 days.
2. System circulation and circulation through heart
Loss of placental circulation and clamping of the cord after
birth results in increase in systemic vascular resistance.
This tends to increase the aortic blood pressure and the left
ventricular systolic pressure. The loss of placental circulation
results in sudden reduction of flow through ductus venosus
which closes off
Flow through ductus venosus disappears by the 7th
day of posnatal life.
The loss of placental flow results in a decrease in the volume
of blood returning to right atrium
Right atrial pressure decreases.
The left atrial pressure becomes higher than right atrial pressure
and the septum primum which acts as a valve of fossa ovalis,
approximates with the septum secundum to close off formen
ovale.

32

Pre-NEET Pediatrics
Functional closure of foramen ovale occurs very
quickly.

Over a period of months, the septum primum and septum


secundum become firmly adherent resulting in anatomical
closure of the foramen ovale.

After closure of ductus arteriosus, there is establishment of


postnatal circulation:

The blood reaching the right atrium through IVC and SVC is
emptied into the right ventricle from where it is pumped into
pulmonary trunk.

After coursing through lungs for gas exchange, it reaches the


left atrium and ventricle.

The left venthcle pumps it out for distribution in the body for
oxygenation of the tissues.

The venous return again comes back to right atrium through


IVC and SVC.

All of the blood leaving the right ventricle, after coursing


through lungs, reaches the left ventricle The two ventricles
are connected in series and therefore, the output of right and
left ventricles are same (in contrast to fetal circulation, where
right ventricular output is more).

Congenital Heart Disease


Left to right shunts (Acyanotic)

VSD (MC congenital heart


disease)
ASD
PDA
VSD> ASD > PDA

Right to left shunts (cyanotic)

Transposition of great arteries (MC


cyanotic lesioin)

Tetralogy of Fallot
Truncus arteriosus
Tricuspid atresia
TAPVR
S-I-M 5Ts

Pediatrics in Last Minutes

33

ACYNOTIC CONGENITAL HEART DISEASE - LEFT TO


RIGHT SHUNTS
Disease

Anatomy/ Symptoms

Signs

Ventricular
septal defect
(MC Cong
HD)

Patent ductus arteriosus

ASD

90% in membranous
part of septum; 70-80%
have spontaneous
closure
In infants FTT, CCF.
MC cong. Heart
disease complicated
by IE

In fetus, shunt is right to left, (normal), through the ductus


arteriosis which connects
pulmonary artery and aorta.
In first few days of neonatal period, ductus arteriosus
should close, if not lung
resistance decrease and
shunt becomes left to right
with subsequent RVH
and failure (abnormal).
Risk factors: Maternal rubella
in 1st trimester, high altitudes.

MC in Prematures (recurrent apnea, RDS, CCF); in infants


(FIT in additioin); in adults
(dyspnea, SBE).

- Ostium secundum (MC,


situated in fossa ovalis)
Ostium primum (AV septal
defect, cleft anterior leaflet
of mitral valve)
Child: asymptomatic, usually
Adult breathless from
pulmonary HTN

Widely split, variable S2.


Pansystolic murmur
at left sternal border.
Small VSD - Loud
murmur (Maladie de
Roger)
CXR: biventricular
hypertrophy, pulmonary
plethora
Continuous machinery
murmur in left
infraclavicular area
(Gibson's murmur)
Mid diastolic flow
murmur at mitral valve
Differential cyanosis
occurs in PDA with
reversal of shunt
Indomethacin is used to
close PDA. S-I-M
"Come IN and close
the door"
PGE (alprosatdil) is used
to keep PDA open
which is necessary
to sustain life in
conditions like TGV. S-IM "Keep OP(G)En
with PGE".
Left parasternal heave
(RVH)
Widely fixed split S2
Ejection systolic murmur
over pulmonary area
Mid diastolic tricuspid
flow murmur

34

Pre-NEET Pediatrics

ACYANOTIC CONGENITAL HEART DISEASE OBSTRUCTIVE


Disease

Anatomy/Symptoms

Aortic stenosis (AS)

Infant: valvular (MC)


- CCF if severe
stenosis

Coarctation of aorta
(COA)

Supravalvular:
asymptomatic,
a/w Williams syn.
Infantile aortic
stenosis proximal to
insertion of ductus
(preductal)
Adult: postductal,
MC coarctation
is just below
origin of left
subclavian A.
A/w HTN in upper
extremities, weak
pulses in lower
extremities
A/w Turner syn.

Signs

Thrill over
suprasternalnotch/
carotids, ejection
systolic murmur
maximal in 2nd
right I/C space
with neck radiation
Pulse: radiofemoral
delay
Elevated upper limb
BP
Palpable arterial
pulsations in
interscapular region
(Suzzman's sign)

Aortic ejection systolic


murmur
CXR: 3 sign, Dock sign
(rib notching due to
collateral circulation)

Cyanotic Congenital Heart Disease


Disease

Anatomy/Symptoms

Signs

Transposition of
great arteries
(TGA)

Cyanosis persists in 100%


oxygen which may even
worsen cyanosis by
causing closure of ductus.

Single Heart sound, murmur


often absent
XCR: Egg on side/string
appearance
- Central cyanosis, clubbing,
right ventricular heave.
Single S2
Ejection systolic murmur

Cyanosis from birth


or shortly after,
proportional to
shunt through
foramen ovale,
ductus arteriosus
or VSD.

Tetralogy of
Fallot
(Pulmonary
stenosis, RVH,

Infant: progressively
deeper cyanosis,
weeks or few
months old.

Pediatrics in Last Minutes


Contd...
Disease

Anatomy/Symptoms

Overriding of
aorta, VSD)
-

Eisenmenger's
syndrome

Cyanotic spells
from infundibular
spasm.
Childhood: squatting
after exertion to
overcome
dyspnea, SBE,
cerebral abscesses,
polycythemia.
CCF is very RARE.
Squatting cause
compression of
femoral arteries
increase pressure
threby decrease
the right to left shunt
and directing more
blood from the
RV to the lungs

Signs
at third left interspace.

- CXR: cour en sabot (boot


shaped heart due to RVH).
Oligemia

Uncorrected VSD,
- Loud P2
ASD, PDA
increase pulmonary
vascular resistance
due to arteriolar
thickening
progressive
pulmonary hypertension.
As pulmonary resistance
increase the shunt
reverses from
L R to R L
shunt which causes
late cyanosis (clubbing
and polycythemia)

35

36

Pre-NEET Pediatrics

COMMON MIXING CONGENITAL HEART DISEASE:


ACYANOTIC AND CYANOTIC
Disease

Anatomy/Symptoms

Signs

Neonate: breathless, CCF

Truncus
arteriosus

Total anom
alous
pulmonary
venous
connection
(TAPC)

- Newborn: obstructed
venous return
simulates
RDS. Onset 2
months to 2
years. Breathless,
FTT

Key:
CCF
CHD
FTI'
TE
LVH
PS
RDS
RVH
SBE

=
=
=
=
=
=
=
=
=

Minimal cyanosis,
Bounding pulse, Single
S2, short systolic murmur
Cyanosis NOT improved
in 100% oxygen. Poor
pulse. Loud S2. Murmur
often absent, tachypnea,
right ventricular heave.
CXR: pulmonary edema
with Snowman heart
(figure of 8 heart of
'cottage leaf')

congestive cardiac failure;


Conegnital heart disease;
failure to thrive;
Infective endocarditis;
left ventricular hypertrophy;
Pulmonary stenosis;
respiratory distress syndrome;
right ventricular hypertrophy;
subacute bacterial endocarditis.

ADDITIONAL POINTS ABOUT CONGENITAL HEART


DISEASE

Holt-Oram syndromeASD with first-degree heart block;


hypoplastic thumb or absent radii.
Critical aortic stenosissevere aortic stenosis that presents in early
infancy is termed critical aortic stenosis and is associated with left
ventricular failure and signs of low cardiac output.
Endocardial fibroelastosisIn infants with critical aortic stenosis,
the left ventricular shortening fraction is usually decreased and
the endocardium may be bright on ECHO, indicating the
development of endocardial fibrous scarring known as endocardial
fibroelastosis.

Pediatrics in Last Minutes

37

Shone complex - Coarctation of aorta + mitral valve


abnormalities (supravalvular mitral ring or parachute mitral valve)
+ subaortic stenosis this group of left sided obstructive lesions
occurring together is called Shone complex.
Blalock-Taussig shunt is the MC aortopulmonary shunt procedure
performed for TOF.
Norwood procedure - is performed for hypoplas tic left heart
syndrome.
Heath Edwards classification is a grading for the histopathologic
changes occurring in small pulmonary arterioles and muscular
arteries in Eisenmenger's syndrome.

Congenital Cardiac Disease Associations

Disorder
Rubella
Turner's syndrome
Infant of diabetic mother
Wiliam's syn.
Marfan's syn.
Down's syn.

22q11 syn.

Defect
PDA
COA
TGV
Supravalvular AS
Aortic insufficiency
Ostium primum ASD, VSD, AV
septal defect (endocardial cushion
defect)
Truncus arteriosis, TOF

Primary Pulmonary Hypertension

Characterized by pulmonary vascular obstructive disease and rightsided heart failure.


RVH Dilated pulmonary artery Pulmonary insufficiency
tricuspid insufficiency (late stages).
Predominant symptoms include exercise intolerance and easy
fatigability. Ejection click after S1 is present and S2 is closely split.
Chest X-ray prominent pulmonary artery and right ventricle.
ECG spiked P waves.
Treatment
Calcium channel blockers (nifedipine);
Intravenous (epoprostinol, treprostinil) or nebulised (iloprost)
prostacyclin (PGI2.);
Oral endothelin antagonist (Bosentan),
Oral PDE-5 inhibitor (sildenafil, tadalafil).
Definitive therapy for non-responders lung transplantation.

38

Pre-NEET Pediatrics

NADAs Criteria
The Assessment of a child for the presence or absence of heart disease
can be done with the help of some guidelines suggested by NADA.
These guidelines are called NADA 's criteria.
Major
Systolic murmur,
grade III or more,
especially with thrill
Diastolic murmur
Cyanosis
Congestive cardiac failure

Minor
Systolic murmur < grade III
Abnormal 2 heart sound
Abnormal ECG
Abnormal X Ray
Abnormal BP

Presence of one major or two minor criteria are essential for


indicating the presence of heart disease.
Criteria for Diagnosis of Rheumatic fever - Presence of 1 major
and 2 minor criteria in the presence of essential criteria is
necessary for diagnosis of rheumatic fever.
Major Criteria

Minor Criteria

1. Carditis
2. Arthritis
3. Subcutaneous nodules
4. Chorea
5. Erythema marginatum

A.
i.
ii.
iii.

Essential criteria
Evidence of recent
streptococcal
infection as indicated
by
a. Increased antistreptolysin
'O' titer
b. Positive throat culture
c. Recent scarlet fever

Clinical
Fever
Arthralgia
Previous rheumatic fever
or rheumatic heart
disease

B. Laboratory
i. Acute phase reactants;
leukocytosis, elevated
sedimentation rate and C
reactive protein
ii. Prolonged PR interval in the
electrocardiogram

Pediatrics in Last Minutes

39

Surgical procedure for selected CHD lesions (palliative procedures)


Procedure

Anatomy involved

Result

Indicated for

BlalockTaussig
shunt
Waterston
shunt

Subclavian artery
to ipsilateral
pulmonary artery
Aorta to right
pulmonary artery

Rashkind
procedure

Balloon atria
septostomy

BlalockHanlon
procedure
Balloon
angioplasty

Operative
atrial
septostomy
Valves and
vessels

Increased
pulmonary
blood flow
Increased
pulmonary
blood flow
Increased
atrial
mixing
Increased
atrial
mixing
Dilation
of valves/
vessels
Decreased
pulmonary
blood flow

Tetralogy of Fallot
Pulmonary valve atresia
Tetralogy of Fallot
pulmonary
valve atresia
Tricuspid atresia
Transposition of
great arteries
Tricuspid atresia
Transposition of
great arteries

Pulmonary
Pulmonary
artery banding artery

Fontan
procedure

Mustard
procedure

Norwood
procedure

Corrective procedures
Right atrium
Atrium
to pulmonary
functions
artery
as right
anastomosis
ventricle
Intera-atrial
RV remains
baffle
systemic
arteries
ventricle
A complex two
stage procedure

Pulmonary valve
stenosis, Aortic
valve stenosis
Endocardial
cushing
defect, Single
ventricle
Tricuspid atresia single
ventricle, pulmonary
atresia
Transposition of great
vessels

Hypoplastic left heart

Characteristics of murmurs in selected heart lesions


Lesion
Aortic
stenosis
Aortic
regurgitation
Pulmonary
stenosis
Pulmonary

Loudest
Right 3rd ICS

Quality
Harsh, SEM

Radiation
Carotids

MLSB

Blowing, Early Apex


diastolic

Left 2nd ICS


Left 2nd ICS

Harsh, SEM
Low pitched,

Lungfields
Little to none
Contd...

40

Pre-NEET Pediatrics

Contd...
Procedure

Anatomy involved

regurgitation
Mitral stenosis Apex

Mitral
regurgitation
ASD
VSD

Apex

PDA

Left 2nd - 3rd ICS

Left 2nd ICS


LLSB

Result

Indicated for

early-mid
diast
Low pitched,
raid diast.
Rumble
Blowing,
holosystolic
Harsh, SEM
Harsh,
Holosystolic
Continuous

None

Left axilla
Lung fields
None
Lung fields

Single Gene Defects - Account for 3% of CHD

Autosomal dominant:
Marfan's - aortic and mitral valve incompetence, dilatation of
the ascending aorta
Holt - Oram - VSD, ASD
Noonan's - PSASD cardiomyopathy
Autosomal recessive:
Pompe's (Type II A glycogen storage disease) - cardiomyopathy
Ellis - VanCreveld - AVSD, common atrium
X-linked:
Duchenne Muscular Dystrophy - cardiomyopathy
Polygenic inheritance:
Well described for PDA with a recurrence risk of 2.5 inn siblings
(recurrence risk increased to 10% if > than one family member
affected).

Respiratory Infections

Etiology
Presenting
age
Clinically

Croup (laryngo
tracheobronchitis)

Acute epiglotitis
(emergency)

MC parainfluenza virus
1; also by RSV
3 months to 3 years
Prodrome with URI
symptoms for
1-7 days; low
grade fever, ins

MC by H. inflenzae type b.
3-7 years
Rapid onset (4-12 hours); high
fever, dysphagia, drooling,
muffled voice, stridor patients
may be in "sniffing" position
Contd...

Pediatrics in Last Minutes

41

Contd...

X-ray
Treatment

Croup (laryngo
tracheobronchitis)

Acute epiglotitis
(emergency)

piratory stridor that


worsens with agitation;
hoarseness of voice,
barking cough,
"Steeple sign" on lateral
neck film
Mist therapy, oxygen,
aerosolized racemic
epinephrine,
dexamethasone

with neck hyperextended and


chin protruding

"Thumb sign" on lateral neck


film
Call anesthesia, transfer to
operation theatre,
endotracheal ntubation and IV
antibiotics

Causes Of Increased Sweat Sodium of 70 MmoI/L or


more

Cystic fibrosis

HIV infection
Metabolic: Glucose-6-phosphate deficiency, Adrenal insufficiency,
Nephrogenic Diabetes Insipidus, Mucopolysaccharidoses,
Fucidosis

Insufficient Sweat: Hypothyroidism, Ectodermal dysplasia, Riley


Day syndrome, Malnutrition
False negative sweat test: Edema, Hypoproteinemia

Pediatric Gastrointestinal Conditions

Intussusception
Telescoping of a bowel segment into itself may - edema, arterial
occlusion, gut necrosis and death.

Affects children 4 mo -2 years;


MC cause of bowel obstruction in first 2 years of life; usually
ileocecal;

Presents as paroxysmal abdominal pain, 'currant jelly' stools


(blood and mucus), bilious vomiting; palpable sausage shaped
mass; sign de Dance;
Barium enema is both diagnostic and theraputic;
a/w HSP and cystic fibrosis

42

Pre-NEET Pediatrics

Volvulus
Incomplete fixation to the posterior abdominal wall, causing
a malrotated gut to twist on itself.
Affects children 0-2 years;
Sudden onset of pain, distension, peritonitis, "bird's beak" on
X- ray,
Treat with surgery immediately since gut may necrose due to
superior mesenteric artery occlusion.

Necrotising enterocolitis
Intestinal necrosis occurring primarily in watershed
distributions,
Affects children 0-2 months;
Prematurity and congenital heart disease are risk factors;
Fever, rectal bleeding, air in bowel wail (pneumatosis
intestinalis) and/or hepatobiliary/ portal air;
Treat with NPO, IV fluids and antibiotics.

Meckel's diverticulum
A remnant of omphalomesenteric duct that persists as an
outpouching of the distal ileum; can contain ectopic gastric
mucosa
Affects children 0-2 years;
Rule of 2s for Meckel's diverticulum =2% of population
affected (MC GI tract abnormality; remnant of
omphalomesenteric duct), 2 inches long, within 2 feet of
ileocolic junction, presents in the first 2 years of life.
Meckel's diverticulum can cause intussusception, obstruction
or volvulus;
Use Meckel scan (technetium radionucide scan) to detect
gastric mucosa;
Treat with surgery

Meconium ileus
In cystic fibrosis, meconium plug obstructs intestine preventing
stool passage.
Affects children 0-2 weeks; may cause late feculent vomiting,
rectal prolapse

Pediatrics in Last Minutes

43

Congenital Gastrointestinal Malformations in Children

Pyloric stenosis
obstruction;

Duodenal
atresia

Tracheoesopha
geal fistula

Hirschsprung's
disease

Hypertrophy of pylorus - gastric outlet


affects first-born male infants at 2 weeks
of life (range 2 wk- 4 mo);
nonbilious projectile vomiting palpable
olive shaped mass in epigastrium;
hypochloremic, hypokalemic metabolic
alkalosis secondary to emesis;
U/S is gold standard;
treat by first correcting dehydration and
metabolic abnormalities, surgery is
pyloromyotomy (Ramstedt's).
Affects infants 0-1 week;
early bilious vomiting with proximal
stomach distension ("Double bubble");
a/w Down's syndrome;
treat with duodenoduodenostomy
Blind esophageal pouch;
fistula between distal esophagus and
trachea MC type;
presents in the first few hours of life with
copious secretions, choking, cyanosis,
respiratory distress; aspiration
pneumonia; gastric distension from air
occurs
Absence of ganglion cells/enteric nerve
plexuses (Auerbach's and Meissner's) in
colon on rectal bisopsy;
Narrowing of anganglionic segment with
dilation of proximal normal colon; can
be a short (75%) or long segment;
Presents at infancy or within first 2 years
of life;
Failure to pass meconium, abdominal
distension, chronic constipation;
Staged procedure with initial diverting
colostomy and later resection when
infant> 6 months old

44

Pre-NEET Pediatrics

Common pediatric renal diseases


Nephrotic Syndrome

MC cause in children is minimal change disease (normal histology


on light microscopy).
Clinically:
Massive edema
Massive selective proteinuria (>2 g/day)
Hypoalbuminemia (<2.5 g/dl)
Hypercholesterolemia (>200 mg%)
Gene mutation a/w congenital NS (Finnish) type = Nephrin
(NPHS1). S-I-M PHinnish = NePHrin.
Gene mutation a/w steroid resistant NS = Podocin (NPHS2)
Treatment Initial longer course of corticosteroids; relapses are
common.

Pediatrics in Last Minutes

45

Also Know
Infrequent relapses: If a patient gets 3 or less relapses in a year
Frequent relapses: If a patient has 4 or more relapses in a year
Steroid dependent: When relapse occurs within 2 weeks of
discontinuation of drugs

Acute Post Streptococcal Glomerulonephritis

Following h/o recent streptococcal pyoderma or pharyngitis.


Edema, Oliguria, hematuria, hypertension, decreased C3 levels

Fluid restriction, furesemide, antihypertensives


> 95% have good prognosis.
Common Pediatric Hematologic Disorders
HUS

HSP

TTP

ITP

MC age

Children

Children

Young
adults

Children or
adults

Previous infection

Diarrhea
(EHEC,
Shigella)

URI

None

Viral (esp. In
children)

RBC count

Platelet count

Peripheral smear

Hemolysis

Hemolysis

Kidney effects

ARF,
hematuria

Hematuria

ARF,
proteinuria

None

Treatment

Supportive*

Supportive

Plasmapheresis,
NSAIDs;
No
platelets**

Steroids***,
splenectomy if
drugs fail

Key differential
points

Age,
diarrhea

Rash,
abdominal
pain,
arthritis,
melema

CNS
changes,
age

Antiplatelet
antibodies

Key
HUS = Haemolytic uremic syndrome; HSP = Henoch-Scholnlein
purpura; Ti? = Thrombotic thrombocytopenic purpura; ITP =
Idiopathic thrombocytopenic purpura; ARF = Acute renal failure;

46

Pre-NEET Pediatrics

EHEC = Enterohemorrhagic E.coli.


* In HUS and HSP, patients may need dialysis and transfusions.
** Do not give platelet transfusion to patients with TTP, clot may
form.
*** Give steroids only if the patient is bleeding.

Grading of Vesico-Ureteral Reflux (VUR)


Grade 1 Reflux up normal caliber ureter without pelvicalyceal filling
Grade 2 Reflux up normal caliber ureter with pelvicalyceal filling
Grade 3 Reflux up dilated ureter into the dilated pelvicalyceal system
Grade 4 Reflux up markedly dilated ureter and collecting system
Micturation Cystourethrogram (MCUG) best investigation for VUR.

Febrile Convulsions

Simple benign febrile


convulsions

Atypical complex
febrile convulsions

Fits occur within


24 hours of onset offever
Last < 10 minutes and
are usually single per
febrile episode
Convulsions are
generalized

Anything different from features


of simple febrile convulsions are
atypical fits. Presence of family
h/o epilepsy, neurodevelopmental
retardation and atypical episodes
increase risk of febrile episodes and
subsequent epilepsy.

No post-i ctal neurological


deficit
Family history may be
present.
Treatment
Antipyretics (paracetamol,
ibuprofen AVOID aspirin
due to risk of Reye's
syndrome)
Hydrotherapy, tepid sponging
oxygen.
IV diazepam
or phenobarbitone for
control of seizures,

Prophylaxis
Intermitent diazepam, clabazam,
midazolam- most desirable.

Continuous prophyl axis - in form of


antiepileptic drug therapy is
advocated
in event of failure of intermittent
therapy, especially in recurrent
atypical seizures or family h/o
epilepsy (valproate or
phenobarbitone).

Pediatrics in Last Minutes

Common Pediatric Epilepsy Syndromes


Syndrome

Symptoms

Diagnosis

Treatment

Absence
seizures

Multiple brief
staring episodes

Generalized
3-Hz spike
and wave
pattern on
EEG

Ethosuximide

Infantile
spasms
(West
syndrome)

Affects infants
<1 year,
'jackknife
spasms',
developmental
regression

Hypsarrythmia on
EEG.
A/w tuberous
sclerosis

ACTH

Lennox
Gastaut
syndrome

First seizure
between 1-7
years of age;
multiple
progressive
difficult to treat
type seizures
with GTCS
and drop attacks

Atypical spike
and wave
pattern in
frontal region
on EEG

No effective
treatment

Juvenile
mycolonic
epilepsy

Affects healthy
adolescents;
myoclonic
jerks in
early morning
hours upon
wakening

Positivefamily
history

Easily treated
with a variety of
antiepileptic
medications

Benign
partial
epilepsy

Partial seizures
during
wakefulness
(oral, vocal)

Classic interictal
spikes
from the
centrotemporal,
(rolandic) region

Seizures usually
disappear by
adolescence

Landau
Klefner
syndrome

Normal children
and lose
language
between
3- 6 years;
often confused
with autism

Bilateral temporal
spike and sharp
waves on EEG.

Antiepileptic
medications

47

48

Pre-NEET Pediatrics

ANTIMICROBIAL THERAPY OF CNS BACTERIAL


INFECTIONS BASED ON PATHOGENA
Organism

Antibiotic

Neisseria meningitides
Penicillin - sensitive
Penicillin - resistant

Penicillin G or ampicillin
Ceftriaxone or cefotaxime

Streptococus pneumoniae
Penicillin - sensitive
Penicillin - intermediate
Gram-negative bacilli (except
Pseudomonas spp.)
Pseudomonas aeruginosa
Staphylococci spp.
Methicillin-sensitive
Methicillin-resistant
Listeria monocytogenes
Haemophilus influenzae
Streptococcus agalactiae

Penicillin G (Ceftriaxone or cefotaxime)


Vancomycin
Ceftriaxone or cefotaxime
Ceftazidime or Cefepime or Meropenam
Nafcillin
Vancomycin
Ampicillin + gentamicin
Ceftriaxone or cefotaxime
Penicillin G or ampicillin

Cerebral palsy: it is defined as a non progressive neuromotor


disorder of cerebral origin

CLINICAL MANIFESTATIONS OF CP
A. Spastic CP
MC type, 65%
May be diplegia, hemiparesis or quadriplegia
Stretch tendon reflexes always brisk
Variable degrees of mental, visual and behavioral problems
Seizures are common
B. Hypotonic or Atonic CP
Patients are atonic or hypotonic
Tendon reflexes are normal, or brisk & Babinski response is
(+)
Severely mentally retarded
C. Extrapyramidal CP
Dyskinesia such as athetosis, choreiform movements, dystonia,
tremors and rigidity are seen.
Arms, leg, neck and trunk may be involved.
Mental retardation and hearing deficits may be present.
Cerebellar involvement occurs in less then 5% of cases.

Pediatrics in Last Minutes

49

Mental Retardation

MR - Abnormalities in intellectual/adaptive function.


Age on onset before 18 years/age of maturity
Most common cause = Chronic anomalies (Down's syndrome)
Most common cause in males = Down's syndrome (mild - to moderate MR)
Most common cause of severe MR in males = Fragile X syndrome
Normal
= 90 - 120
Borderline
= 70 - 90
Mild
= 51 - 70/ 50 -70
Moderate
= 36 - 50/ 35 - 50 (trainable)
Severe
= 21 - 35 (able to guard cannot manage self)
Profound
= <20 (unable to guard self)
Severe and
profound MR = custodial

Intelligence Assessment Scales

Weschler scale - Adult


WISC - R - Weschler intelligent scale for children revised
WPPSI ' Pre school
Mallin's ' Modification of Weschler

50

Pre-NEET Pediatrics

The "Original" Six Exanthematous Illnesses


1. First disease
2. Second disease
3. Third disease
4. Fourth disease
5. Fifth disease
6. Sixth disease

Measles
Scarlet fever
Rubella (German measles)
Duke's disease (scarlantinella)
Erythema infectiosum
Exanthem subitum (Roseola
infantum)

Measles (Rubeola, First Disease)


Cause

RNA paramyxovirus; IP = 8-12days

Spread

By airborne droplets from nose and throat secretions, usually 4


days before and 5 days after the appearance of the rash.
Secondary attack rate is 80%.

Age

Children 6 months -3 years of age in developing countries and


older children (>5 years) in developed countries.

Immunity

One attack of measles generally confers life long immunity.


Maternal antibodies protect in infants up to 6 month of age.

Nutrition

Mortality 400 times higher in the malnourished child (also vit A.


deficiency) and also these children excrete measles virus for longer
periods indicating prolonged risk to themselves and, of spread to
others.

Prodromal Symptoms

Last 2-3 days, cough, coryza, conjunctivitis, fever, conjunctivae


look glassy, and then the semilunar fold swells (Meyer's sign).

Pediatrics in Last Minutes

51

Koplik's Spots

Are 1-2 mm bluish-white spots on a red base on the buccal mucosa


opposite the first/second lower molars, pathogenomic of measles,
generally seen during the first 2 days of symptoms and are often
fading as the rash appears

Comby's Sign

Thin, whitish patches on the gums and buccal mucosa formed of


degenerated squamous epithelium.

Rash

On the fourth/fifth day, the macular rash appears consisting of


discrete lesions that begins behind the ears and become confluent
as rash spreads from hairline downward, sparing palms and soles;
Rash and fever disappear in 3-4 days signalling the end of the
disease. Prolonged fever suggests a complication of measles. The
entire illness lasts about 10 days.

Complications

Respiratory: otitis media, pneumonia, fatal giant cell pneumonitis


in the immunosuppressed, flare up of primary TB.
Neurological: febrile fits, meningitis, encephalitis, subacute
sclerosing panencephalitis (late and rare):
Digestive: resistant diarrhoea, achlorhydria, hepatitis, appendicitis
(due to lymphoid tissue blocking the lumen of the appendix);
Miscellaneous: myocarditis, glomerulonephritis, thrombocytopenic
purpura, tuberculin anergy, keratoconunctivitis sicca, intrauterine
infection may cause fetal malformations.

Diagnosis

Serology for measles IgM antibody. Warthin-Finkleday cells are


multinucleate giant cells with inclusion bodies in the nucleus and
cytoplasm, found in respiratory and lymphoid tissues,
pathognomonic of measles.

52

Pre-NEET Pediatrics

Prevention

Measles vaccine: a live attenuated, tissue-culture vaccine presented


as freezdried product.
WHO recommends immunization at 9 months of age. 0.5ml SC
injection, reconstituted vaccine should be kept on ice and used
within 1 hour. 5-10 days after immunization, a mild 'measles' illness
(fever and rash) may occur - self limiting.
Immunity develops 11-12 days after vaccination. Susceptible
contacts over 9-12 months age, may be protected against measles
with measles vaccine, provides this is given within 3 days of
exposure.

Rubella (German Measles, Third Disease)


Cause

Togavirus, RNA virus; IP= 14-21 days (average 18 days). The


patient is infective 5 days before and 5 days after the day the rash
starts.

Presentation

Postauricular, cervical and suboccipital lymphadenopathy (may


appear as early as 7 days before the appearance of the rash).
Macular rash spreads from hairline downwards, clearing as it
spreads. The rash disappears aitogether bythe third day. -

Forschemier's Spots

(palatal petechiae), also seen in scarlet fever and infectious


mononucleosis.

Test

The most widely used serological diagnostic test is the


haemagglutination inhibition test.

Treatment

Usually none needed, self limiting

Pediatrics in Last Minutes

53

Complications

Arthritis especially among women, encephalitis, thrombocytopenic


purpura, malformations in utero.

Infection during the first 4 weeks of pregnancy: Cataract in 70%;


during weeks 4-8: patent ductus arteriosus; during weeks 8-12:
deafness.

Prevention

Live attenuated RA 27/3 vaccine, produced in human diploid


fibroblast; seroconversion occurs in more than 95%; pregnancy
is considered a contraindication to rubella immunization.

Erythema Infectiosum (Fifth Disease)

Cause: Human parvovirus B19, primarily affects children 3-12


years old..

Manifests as a bright-red 'slapped cheek' appearance (a raised,


fiery flush on the cheeks), followed by a diffuse lacy reticular rash
(on the limbs) that waxes and wanes over 3 weeks.

Adults with fifth disease often have arthritis, and fetal hydrops
can develop in association with this condition in pregnant women.

Parvovirus B19 is also the cause of aplastic crises in sickle cell


disease.

Exanthem Subitum (Roseola Infantum, Sixth Disease)

Cause: Human herpes virus 6 (HHV-6B).

MC in children under 3 years of age. As in erythema infectiosum,


rash usually appears after fever has subsided. It is a rose-pink
maculopapular rash (spa rinface); resolves within 2 days.

Chicken Pox (Varicella)


Etiology

Varicella-zoster virus also called human (alpha) herpes virus


3. Recovery from primary infection is commonly followed by
the establishment of latent infection in the sensory ganglia
often without clinical manifestations.

54

Pre-NEET Pediatrics

Source of infection is usually a case of chicken pox. Spread


by respiratory droplets and from the vesicular fluid during the
first 3 days of the illness. The scabs however are not infective.

Period of infectivity is from 1-2 days before the appearance of


the rash and 4-5 days thereafter.

Secondary attack rate is about 90%. One attack usually gives


immunity for life.

Presentation

Incubation period =14 to 16 days.

Prodromal stage consists of fever, malaise and shivering.

Rash comes on the day the fever starts. It is centri petal in


distributiort,first appears on the trunk where it is abundant
and then on the extremities. Mucosal surfaces are also
involved, but palms and soles are unaffected.

The rash advances quickly through the stages of macule,


papule, vesicle, pustule and scab (pleomorphic rash)

Complications

Secondary bacterial infection of the rashes; varicella


pneumonia; haemorrhages into the lesion; encephalitis, acute
cerebellar ataxia; purpura fulminans; arthritis; glomeruloniphritis; Reye's syndrome.

Treatment

Symptomatic and supportive, since this is a self-limiting


condition. Aspirin should NOT be used as it may increase the
risk of developing Reye's syndrome.

Prevention

Varicella zoster immunoglobulin, 1.25 to 5 ml given within 72


hours of exposure will modify or prevent the disease.

Live attenuated chicken pox vaccine is now available.

Pediatrics in Last Minutes

55

Periods of Infectivity In Childhood Infectious Disease


Disease
Chickenpox
Diphtheria
Mumps
Rubella
Measles

Infectious periods
5 days before rash to 6 days after last crop
2-3 weeks (shorter with antibiotic therapy)
3 days before salivary swelling to 7 days after
7 days before onset of rash to 4 days after
From onset of prodromal symptoms to 4 days after onset of
rash
Scarlet fever 10-21 days after onset of rash (shortened to 1 day by penicillin)
Whooping cough 7 days after exposure to 3 wks after onset of symptoms
(shortened to abt 7 days by antibiotics)

Precocious Puberty

Precocious puberty is defined as the onset of secondary sexual


characters before 8 yr of age in girls and 9 yr in boys.
True (central) precocious puberty (gonadotropin dependant) is
due to hypothalamic-pituitary-gonadal activation. Caused by
Idiopathic
Organic brain lesions: brain tumours, severe head trauma,
hydrocephalus, hypothalamic hamartoma, CNS infections.
Hypothyroidism, prolonged and untreated.
Precocious pseudopuberty (peripheral, gonadotropin
independent): here secondary sex characters appear but there is
no activation of hypothalamic-pituitary-gonadal axis. Caused by
Females: Congenital adrenal hyperplasia, Hypothyroidism,
Exogenous estrogen, McCune Albright syndrome, Aromatase
excess syndrome.
Males: Congenital adrenal hyperplasia, Hypothyroidism,
Exogenous androgens, Activating LH receptor mutations,
HCG-secreting tumour.

Common Pediatric Abdominal Malignancies


Wilms Tumour

MC renal malignancy of early childhood (2-4 years).


Risk factors:
WAGR - Wilms tumour, Ariiridia, Genital anomalies, mental
Retardation.
Denys-Drash syndrome (Gonadal dysgenesis and nephropathy
leading to renal failure, dominant negative mutation in the
WT-1 gene;

56

Pre-NEET Pediatrics

Beckwith-Wideman syndrome (Enlargement of body organs,


hemihypertrophy, renal medullary cysts, adrenal cytomegaly).

Huge palpable flank mass, hemihypertrophy, hematuria,


hypertension.
Diagnosis by a/Ultrasound and confirm by excisional biopsy.
Treatment transabdominal nephrectomy followed by
chemotherapy (vincristine, dactinomycin); prognosis is usually
good.

Neuroblastoma

MC tumour of adrenal medulla but can occur anywhere along


sympathetic chain; affects children <5 years.

Risk factors: N-myc oncogene; Neurofibromatosis, tuberous


sclerosis, pheochromocytoma, Hirschsprung's disease.
Clinically: abdominal mass, hepatomegaly, leg edema, periorbital
bruising.

HVA in urine
Treatment localized tumours cured with excision; chemotherapy
and radiotherapy can be used as adjunct. Good prognosis if
diagnosed <1 year of age. Shimada classification is used.

D/D of Wilms' Tumour, Abdominal Neuroblastoma


Wilms tumor

Neuroblastoma

Age in years

2-5

<2 yrs, 30% under 1 year

Health

Well

Usually ill, Lethargic

Clinical

Swollen abdomen

Pale, weight loss, diffuse bone


pain

Mass

Lobulated

Irregular edge, craggy hard

Crosses midline

Rare

Common

Bilateral

Rare

Occasional

Renal pelvis

Grossly distorted

Pushed down by mass above

Lungs

Bone (orbits classically)

Ultrasound
Metastases

Calcification found

Pediatrics in Last Minutes

57

Approach to a Child with Anemia

FAB classification divides AML into eight types M0 to


M7.
This scheme takes into account:
i. The degree of maturation (Mo to M3)
ii. The lineage of leukemic blast (M4 to M7).
Class

Blast cells

Mo minimally differentiated AML

M1 AML without differentiation

Myeloperoxidase negative

Auer rods negative

Express myeloid lineage antigen

3 % blasts myeloperoxidase

Auer rods positive

Full range of myeloid maturation

Myeloperoxidase positive

positive
M2 AML with maturation

M3 Acute promyelocytic

Auer rods positive

Maximum Auer rods.


Contd...

58

Pre-NEET Pediatrics

Contd...
Class

Blast cells

leukemia

Myeloperoxidase positive

M4 Acute myelomonocytic
leulcemia

Both myelocytic and Monocytic


differentiation

Myeloperoxidase (+) ye } Myeloblastic


Auer rods (+) ye Monoblastic
Nonspecific esterase (+) ve Monoblastic
M5 Acute monocytic leukemia
M6 Acute erythroleukemia
M7 Acute Megakaryocytic

Non specific esterase (+) ve

Myeloperoxidase and Auer rods (-) ve

Dysplastic erythroid precursors

Myeloblasts seen in advanced age

Blasts of megakaryocytic lineage

GP IIb/ IIIa or VwF (+) ve

Juvenile Rheumatoid Arthritis can be divided in 3


major clinical types

Pediatrics in Last Minutes

59

GLYCOGEN STORAGE DISEASES

Glycogen storage diseases result from a hereditary deficiency of


one of the enzymes involved in the synthesis or sequential
degradation of glycogen.
Clinical features depend on type of enzyme deficiency and organ
involvement.

Glycogen storage diseases can be divided into:


1. Hepatic form (Liver glycogenoses)
Liver is the key player in glycogen metabolism.

It contains enzymes that synthesize glycogen for storage and


ultimately break it down into free glucose, which is then
released into blood.
So, Aninherited deficiency of hepatic enzymes that are involved
in glycogen metabolism leads to
i. Storage of glycogen in liver - Hepatomegaly.
ii. Reduction in blood glucose - Hypoglycemia.
Examples of liver glycogenoses are i. Type I glycogenosis - Glucose - 6 - phosphatase deficiency
(Von Gierke disease).
ii. Type III glycogenosis - Debranching enzyme def
iii. Type IV glycogenesis - Branching enzyme def
iv. Type VI glycogenosis - Liver phosphorylase def

2. Myopathic form (Muscular glycogenoses)


In skeletal muscle, as apposed to the liver, glycogen is used
predominantly as a source of energy.
This is derived by glycolysis, which leads ultimately to the
formation of lactate.
So, enzyme deficiency leads to:
i. Glycogen deposition in muscle
ii. Muscle cramp after exercise
iii. Exercise - induced rise in blood lactose levels owing to
block in glycolysis
iv. There may be myoglobinemia
Examples are:
i. Type V glycogenosis - phosphorylase deficiency (McArdle
disease).
ii. Type II glycogenosis -phosphofructokinase def

60

Pre-NEET Pediatrics

3. Miscellaneous types
Associated with glycogen storage in many organs and death
in early life.
Examples:
i. Type II glycogenosis - Acid maltase deficiency (Pompe 's
disease)

Pediatrics in Last Minutes

61

An Approach to IEM with Acidosis

Important Lysosomal Storage Diseases


Disease

Deficient enzyme

Sphingolipidoses
GM1, gangliosidosis
GM2, gangliosidosis
i. Tay-sachs disease
ii. Sandoff disease

-galactosidase
GM1 ganglioside
Hexosaminidase - subunit GM2 ganglioside
Hexosaminidase - subunit GM2 ganglioside

Glycogenosis type II
Pompe disease

- 1,4, glucosidase

Glycogen

- 1, Iduranidase
Induronate sulfatase
Sulfoiduranate sulfatase

Dermatan sulfate
Heparan sulfate

Mycopolysaccharidose
MPS I (Hurler)
MPS II (Hunter)
Scheie syndrome

Accumulating
metabolites

Contd...

62

Pre-NEET Pediatrics

Contd...
Disease

Deficient enzyme

Accumulating
metabolites

Sulfatidoses
Metachromatic
leukodystrophy
Krabbe disease
Gaucher disease
Fabry's disease
NiemannPick disease

Arylsulfatase A

Sulfatide

Galactosylceramidase
Glucocerebrosidase
- Galactosidase A
Sphingomyelinase

Galactocerebroside
Glucocerebroside
Ceramide trihoxoside
Sphingomyelin

Others
Wolman disease

Acid lipase

Cholesterol ester,
triglyceride

LEUCODYSTROPHY
Leucodystrophy refers to progressive degeneration of the white
matter of the brain due to imperfect growth or development of
the myelin sheath, the fatty covering that acts as an insulator
around nerve fibre.
Myelin which lends its color to the white matter of the brain, is a
complex substance made up of at least ten different chemicals.
The leucodystrophy are a group of disorders that are caused by
genetic defects in how myelin produces or metabolizes these
chemicals.
Each of the leucodystrophies in the result of a defect in the gene
that controls one (and only one) of the chemicals.
Specific leucodystrophies include:

Metachromatic
leucodystrophy
Krabbe 's disease

Adrenoleucodystrophy
PelizaeusMerzhacher
disease

Canavans disease
Alexander disease

Symptoms vary according to the specific type of leucodystrophy


and may be difficult to recognize in the early stages of the disease.

Canavan's Disease

Autosomal recessive disorder


Caused due to deficiency of the enzyme N-Aspertoacylase.
This leads to accumulation of N-Acetyl aspartic acid in brain and
urine.

Pediatrics in Last Minutes

63

It is characterized by the clinical traid of - Hypotonia, Head


lag, Macrocephaly

Adrenoleucodystrophy

X-linked recessive disorder


Caused due to deficiency of Acyl-CoA synthetase.
Onset is about 5-10 years
Main symptoms are ataxia, spasticity, motor deficits, cortical
blindness.
Marocephaly is not a key feature

Metachromatic leucodystrophy

Autosomal recessive disorder


Caused due to Arylsulfatase A deficiency
Onset is in the 2nd year.
Symptoms are inco-ordination, especially gait disturbance, then
general regression, optic atrophy, combined upper and lower
motor neuron signs.
Marocephaly usually late.

Krabbe leucodystrophy

Autosomal recessive
Caused due to deficiency of Beta-galactosidase deficiency
Onset is in the first 6 months of life
Optic atrophy, spasticity
Head often small

Wolman's Disease

Wolman disease is very rare, with only 50 reports of the disease


published in the worldwide medical literature. It is an autosomal
recessivedisease and affects both males and females.
The disease affects the breakdown and use of fats and cholesterol
in the body and belongs to a family of disorders called lipid storage
disorders.
Wolman disease is caused by mutations in the LIPA gene. The
LIPA gene encodes for an enzyme called lysosomal acid lipase
which is found in the lysosomes. It breaks down fats such as
cholesteryl esters and triglycerides so that they can be used by the
body.

64

Pre-NEET Pediatrics
A shortage of lysosomal acid lipase leads to accumulation of
triglycerides, cholesteryl esters and other fats within the cells of
the affected individuals. This accumulation as well as malnutrition
caused by the body's inability to use lipids properly result in signs
and symptoms of wolmans disease.

Clinical Features

In affected individuals harmful amounts of lipids accumulate in


the spleen, liver, bone marrow, small intestine, adrenal glands
and lymphnodes.
In addition to fat deposits, calcium deposits in the adrenal gland
are also seen.
Infants with wolman disease are healthy and active at birth but
soon develops signs and symptoms of the disorder. These may
include
Enlarged liver and spleen
Poor weight gain
Low muscle tone
Jaundice
Vomiting
Dianthoea
Developmental delays
Anemia (low amount of iron in blood) poor absorption of
nutrients from food.
Children affected by this condition develop severe malnutrition
and generally do not survive past early childhood.

Diagnosis
X-Ray Findings of Wolman Disease
Characteristic pattern of calcification (outlining the outline of
the cortex of both the glands) in enlarged but normally shaped adrenal
glands.

CT Scan and MRI

It shows enlargement of the liver and spleen, calcification of


both adrenal glands and lymphadenopathy (enlarged lymph
nodes).
The definitive diagnosis of wolmans disease is made by

Pediatrics in Last Minutes

65

demonstration of deficient lysosomal acid lipase activity in


leucocytes.
Amniocentesis can be performed for prenatal diagnosis of wolmans
disease.

Treatment

Currently there is no cure for wolmans disease so treatment focuses


on management of symptoms. If the adrenal gland is not making
enough hormones and steroids, replacement steroids and
hormones can be given.

Pre NEET Pediatric


INCLUDING RECENT

ALL INDIA AND AIIMS TOPICS

QUESTIONS
CLINICAL QUESTIONS AND QUESTION BASED ON
EXPECTED NEET PATTERN
Q 1. A neonate delivered at 37 wks of gestation with a birth weight of
1.9 kg develops intolerance to feed/ lethargy/abdominal distension on
2nd day. Sepsis screens is negative. Physical examination is
unremarkable. PCV is observed to be 72%. Which of the following
represents with best management option:
a. Hydration with IV fluids
b. Partial exchange transfusion
c. Presumptive treatment for sepsis
d. Medical management for intestinal obstruction
Q 2. A premature baby born at 33 wks of gestation with a birth weight
of 1500 gm has stable vitals. Which of the following you will choose
as initial feeding method of choice:
a. Intravenous fluids
b. Intravenous fluids and oral feeding
c. Orogastric tube/ alternate oral route
d. Total parenteral nutrition
Q 3. A 2 yrs old male child presents to paediatric emergency
department with history of fever x 2 wks. He has received multiple
antibiotics. Blood tests for Typhoid, Malaria, Dengue are all negative.
He however had sterile pyuria in one of reports (urine C/s - sterile).
BCG scar is indurated and H/o conjunctivitis, chelitis and erythematous
rash over body for initial 5 days of illness is obtained the most likely
treatment of choice is:
a. Cyclosporin
b. Prednosolone
c. Methotrexate
d. IvIg

Questions

67

Q 4. A child presented with intermittent episodes of left sided flank


pain. USG KUB reveals a large hydronephrosis with dilated renal pelvis
and cortical thinning with a normal ureter. Kidney differential function
was observed to be 19%, which of the following is the best
management:
a. Nephrectomy
b. Pyeloplasty
c. External drainage
d. Endopylostomy
Q 5. Which of the following statements about neuroblastoma is false:
a. Most common extracranial solid tumor in children
b. >50% present with metastasis at the time of diagnosis
c. Lung metastasis is common
d. Often encase aorta and its branches at time of diagnosis
Q 6. A very low birth weight preterm baby is on ventilator for
respiratory distress. Baby presents with clinical features of necrotizing
enterocolitis with perforation. What is the appropriate management?
a. Conservative management
b. Immediate laparotomy
c. Extra corporeal membrane oxygenation with surgery after
stabilization
d. Peritoneal drainage
Q 7. A child with 3 days history of upper respiratory tract infection
presents with moderate respiratory distress and stridor at rest. What is
the most appropriate management?
a. A single daily dose of I/V ceftriaxone
b. Mechanical ventilation
c. Hospitalization and Nebulization with racemic epinephrine
diluted with water.
d. Reassure parents and advise on symptomatic treatment for
fever.
Q 8. A child is brought with drowsiness, decreased deep tendon
reflexes and seizures. On examination the child has a blue line on
gums. There is a history of constipation. Which will be most appropriate
drug that should be used in this child?
a. EDTA
b. DMSA
c. BAL
d. Pencillamine

68

Pre-NEET Pediatrics

Q 9. A 7 yr old girl is bought with complaints of generalized swelling


of body. Urinary examination reveals grade 3 proteinuria and the
presence of hyaline and fatty casts. She has no history of hematuria,
which of the following statements about her condition is true:
a. No IgG deposits or C3 deposits on renal biopsy
b. L3 level will be low
c. IgA nephropathy is the likely diagnosis
d. Alport's syndrome is the likely diagnosis
Q 10. A 8 yr old male presents to paediatric casualty with non
blanching rashes on skin, swelling of knee joint x 2 days, Malena
associated with abdominal pain x 1 day. Platelet count, prothrombin
time are normal. Urine R/M shows 2+ protein, hematuria. What should
be your line of management?
a. Symptomatic treatment with rest and analgesia
b. Short course of steroids
c. Steroids with immunosuppressive
d. Skin biopsy and orthopaedic referral
Q 11. A 2 yrs old boy presents with short stature and bowing of legs.
X-ray knee shows evidence of rickets. He has been given Vit D twice
2 months back. Blood investigation shows S.calcium 9 mg/dL, S.
Phosphate 2 mg/dL, S. Alkaline phosphatase 1030 g/dL, S.PTH and
S. Bicarbonate levels are normal. What should be the next line of
management?
a. Give calcitriol in very large doses
b. Oral phosphate and Vit D supplements
c. Oral calcium and phosphate supplements
d. Restrict phosphate, give calcium and vit D supplements
Q 12. A six months old girl is having recurrent UTI. Ultrasound
abdomen shows bilateral hydronephrosis. MCU (Micturating cysts
urethrogram) shows bilateral grade IV vesicoureteral reflux. The
treatment of choice is:
a. Endoscopic injection of polyteflon at uretic orifice
b. Uretric reimplantation
c. Bilateral ureterostomy
d. Prophylactic antibiotics
Q 13. A 10 months old boy, weighing 3 kg has polyuria, polydipsia
and delayed motor milestones. Investigations show blood level of
creatinine 0.5 mg/dL, potassium 3 meq.L, sodium 125 meq/L, chloride

Questions

69

88 meq/L calcium 8.8 mg/dL, PH 7.46 and bicarbonate 26 meq/L


Ultrasonography shows medullary nephrocalcinosis. The most likely
diagnosis is:
a. Renal tubular acidosis
b. Diabetes syndrome
c. Bartter syndrome
d. Pseudohypoaldosteronism
Q 14. A 3 yr old child with acute onset of fever of 1040F developed
febrile seizures and was treated. He had similar episode at 2, 2 yr of
age. Which of the following statements is false?
a. The risk of epilepsy is 4% in this case
b. Long term phenytoin is required
c. Intermittent clobazan and paracetamol should be used
d. Consider EEG and imaging
Q 15. A 14 yr old boy has history of frequent myoclonic jerks on
awakening, making hair combing and tooth brushing difficult for past
6 months. Now, he has presented to casualty with generalised tonic
clonic seizure. Neurological examination is normal, which of the
following statements is true?
a. Drug of choice is carbamazepine
b. Carbamazepine can be discontinued after complete remission
c. Response to anticonvulsants is poor
d. EEG in this condition shows a 4-6/ sec irregular spike and
wave pattern, which is enhanced by photic stimulation.
Q 16. A 2 yr old child is bought by parents with history of seizures
and development delay. He has multiple hypopigmented macules over
the back. What is the most probable diagnosis?
a. Neurofibromatosis type I
b. Tuberous sclerosis
c. Struge Weber's syndrome
d. Vao Nippel Lindau disease
Q 17. A 5yr old child is admitted with headache, vomiting and difficulty
in walking. Physical findings include truncal ataxia, papilledema and
left lateral rectus palsy. Finger to nose test was normal. The most
likely diagnosis is:
a. Dandy Walker syndrome
b. Syringobulbia
c. Arnold Chiari malformation
d. Medulloblastoma.

70

Pre-NEET Pediatrics

Q 18. A 5 months old formula fed infant has been brought with
complaints of watery diarrhoea of 2 days duration and irritability of
one day. Physical examination reveals a markedly irritable child with
a rather doughy skin and rapid pulse, borderline CFT. Which of the
following statements is true?
a. Give I/V bolus of normal saline
b. Give I/V bolus of ringer lactate
c. Calculate water deficit and replace over 24 hrs at a rate of
0.5 meq/L/hr.
d. Treat as any dehydration and give moderate dehydration
correction.
Q 19. Study the following inheritance pattern
Which of the following statements is true?

a.
b.
c.
d.

Autosomal dominant
Mito chondrial
Autosomal recessive
X-linked dominant

Q 20. A newborn is born to a HIV positive woman an ART. He was


given Nevirapine at birth. Which of the following statements regarding
future management is false?
a. Start Cotrimoxazole prophylaxis at 1 month till HIV infection
can be ruledout.
b. Advise exclusive top feeding or exclusive breast feeding
c. Perform DNA PCR for HIV at 6-8 wks of age.
d. Do not give OPV and BCG at birth.

Questions

71

Q 21. A. Assertion: An infant presents with hypotonia and areflexia.


During his intrauterine period there was polyhydramnios and decreased
fetal movements. He is diagnosed as floppy infant.
B. Reasoning: Electromyography will show a decremental response
on repetitive stimulation and muscle biopsy will be normal.
a. Both statement A and B are true, and B is the correct
explanation of A
b. Statement A is true while statement B is false.
c. Statement A is false while statement B is true.
d. Both statements A and B are true but B is not the correct
explanation of A.
Q 22. Previously well newborn discharged as "healthy" from the
nursery at D1 of life presented casualty at D7 of life with poor feeding,
poor pulses, tacycardia, an ashen appearance. He was immediately
intubated and put on inotropic support. Sepsis serum was negative.
Urgent echo done was suggestive of small left ventricle with atretic
mitral valve. Prostaglandin E1 infusion was immediately started. He
fails to improve with same. What should be next line f management?
a. Increase inotropic support and continue same management
b. Emergency atrial septostomy
c. Corrective surgery
d. Upgrade antibiotics
Q 23. A 1 yr old child presents to paediatric casualty with irritability,
restlessness, severe respiratory distress and cyanosis. Mother gives
history of vigorous crying hr back. On auscultation, a short systolic
murmur and soft 2nd heart sound are heard in left 3rd and 4th
parasternal spaces. What should be the immediately intervention?
a. Assurance of mother that nothing should be done and he will
become alright spontaneously.
b. An urgent echo and chest x-ray is planned.
c. Knee-chest position, oxygen and Inj. Morphine administration
d. agonists like Isoxuprine administration
Q 24. A 1 yr old infant presented to paediatric casualty with excessive
crying, irritability and billous vomiting. Mother gives history of passage
of blood per rectum once mixed with stools. He is hemodynamically
stable but not allowing examination. X-ray abdomen shows ascites
with distended bowel loops.What should be next line of management:
a. Manage as a case of acute invasive gastroenteritis with I.V.
antibiotics and I.V. fluids

72

Pre-NEET Pediatrics
b. Send PT, aPTT and give Inj Vit K
c. Do urgent USG and perform hydrostatic reduction.
d. Do urgent USG and prepare for urgent laparotomy

Q 25. A 8 day old breast fed male baby presents with vomiting, poor
feeding and loose stools. On examination the heart rate is 190/min,
blood pressure 50/30 mmHg, respiratory rate 72 breaths/ minute and
capillary refill time of 4 sec. Investigations show Hb 15 gm/dL, SNa+
- 120 meq/L K+ - 6.8 meq/L, Cl- 81 meq/L, bicarbonate 15 meq/L,
creatinine 0.6 mg/dL. The most likely diagnosis is:
a. 21 hydroxylase deficiency
b. 3- hydroxy steroid deficiency
c. 11 hydroxylase deficiency
d. Aldosterone synthase deficiency
Q 26. A. Assertion: A 8 day old child presents yellow sclera, whitish
stool and turmeric colour urine with reducing substance positive on
3rd day of septicaemia on broad spectrum antibiotics.
R. Reasoning: This is a case of severe sepsis and antibiotics should be
immediately upgraded
a. A and R both correct and R is explanation of A
b. A and R both correct but R is not explanation of A
c. A correct R wrong
d. A wrong R correct
Q 27. A: Assertion: A 1 month baby presents with history of jaundice,
turmeric colored urine and pale stools since birth. Examination reveals
liver span of 10 cm TORCH IgM is positive for Rubella.
R: Reasoning: A liver biopsy should be done to differentiate neonatal
hepatitis from biliary atresia
a. A and R both correct and R is explanation of A
b. A and R both correct but R is not explanation of A
c. A correct R wrong
d. A wrong R correct
Q 28. 30 yr old lady delivered a healthy baby at 37 wk of gestation,
she was positive for HBSAg but negative for HBCAg. Which of the
following is the most appropriate treatment for the baby:
a. Both active and passive immunization soon after birth
b. Passive immunization soon after birth and active immunization
at 1 yr of age.
c. Only passive immunization soon after birth
d. Only active immunization soon after birth

Questions

73

Q 29. A 7 yr old girl presents to casualty with history of fever x 4


days, epistaxis and vomiting. She is diagnosed to be Dengue
Heorrhagic fever (DHF), which of the following does not constitute
the diagnosis of DHF?
a. Thrombocytopenia
b. Hypoalbuminemia
c. Pleural effusion
d. Decreased hematocrit
Q 30. A child died soon after birth. On examination there was pallor,
hepato splenomegally and edema all over body. Most probable
diagnosis is:
a. - Thalassemia
b. - Thalassemia
c. Herditary spherocytosis
d. ABO in computability/ sickle cell anemia

AIIMS MAY 2012


Q 1. Which of the following will favour the diagnosis RDS in newborn?
(AIIMS Nov 2010)
a. Reciept of antenatal steroids
b. Air bronchogram in chest x-ray
c. Manifests after 6 hours
d. Occurs after term gestation
Q 2. All the following can occur in a neonate for heat production
except (AIIMS Nov 2006)
a. Shivering
b. Universal flexion like a fetus
c. Breakdown of brown fat with adrenaline secretion

74

Pre-NEET Pediatrics
d. Cutaneous Vasoconstriction

Q 3. Due to advances in Cancer treatment the prognosis of which of


the following has become better?
a. Glioblastoma multiforme
b. Esophageal carcinoma
c. ALL in children
d. Cholangio carcinoma
Q 4. A newborn baby presented with profuse bleeding from the
umbilical stump after birth. Rest of the examination and PT, APTT
are within normal limits. Most probable diagnosis is:
a. Factor X deficiency
b. Glanzmann thrombasthenia
c. Van Willebrand disease
d. Bernard Soullier disease

ALL INDIA PG MEDICAL ENTRANCE EXAMINATION


2012
Q 1. A 1 year old girl presents with fever since 24 hrs and on and off
coughing. She is passing foul-smelling bulky stools and had 4 attacks
of bronchitis in past. The girl is suspected of having:
a. Cystic fibrosis
b. Maple synp urine disease
c. Phenylketonuria
d. Impaired glucoronidization
Q 2. A 2 yrs old boy keeps on looking at his own hands and does not
respond on calling. His mother development is otherwise normal,
probable diagnosis would be:
a. Autism
b. Depression
c. ADHD
d. Schizophrenia
Q 3. Regarding parenteral nutrition of newborns, 20% intralipid has
are the following advantage over 10% intralipid solution except
a. Reduced phospholipids
b. Increased EFA (essential fatty acids)

Questions

75

c. Increased Calorie density/ mL of fluid


d. Better/improved trigly ceride clearance
Q 4. Most frequently identified fetal tumor?
a. Neuroblastoma
b. Wilms tumor
c. Fibro sarcoma
d. Sacrococcygeal teratoma
Q 5. A case of meningomyelocoele was posted for surgery. Till patient
is waiting for surgery the covering of sac will be protected by a gauze
piece soaked in
a. Normal saline
b. Tincture iodine
c. Methylene blue
d. Mercuro chrome
Q 6. Least chances of infective endocarditis is seen with
a. Small VSD
b. Small ASD
c. Mild As
d. Mild AR
Q 7. True about jaundice in neonates is:
a. Can be seen after ventouse delivery
b. Physiological jaundice is seen with 48 hours of birth
c. Increased conjugated bilirubin leads to kernicterus
d. Breast milk jaundice is maximum in 7 days from birth
Q 8. A term infant born to a diabetic mother was lethargic few hours
after birth and his blood glucose was 3 mg%. What should be done
next?
a. Give 10% Dextrose orally
b. 10% Dextrose I.V.
c. Give expressed breast milk
d. Do exchange transfusion
Q 9. A mentally challenged child with dysphagia and Ophistotonic
spasms is also having Choreoathetoid movements and self-mutilating
behaviour with positive family history. Which of the following
investigations is suggested?
a. S. Uric acid

76

Pre-NEET Pediatrics
b. S. Alkaline phosphatase
c. S. LDH
d. Lead levels in blood

Q 10. A child who can use 4-5 words including meaningful nouns
and represent his ideas mostly by non-verbal communication then
what is his probable age?
a. 15 months
b. 18 months
c. 24 months
d. 12 months
Q 11. All of the following can occur in a neonate for heat production
except:
a. Shivering
b. Universal flexion like a fetus
c. Breakdown of brown fat with adrenaline secretion
d. Cutaneous vasoconstriction
Q 12. A down's child mentally retarded, a/e
a. Deleted 21
b. Trisomy 21
c. Robert Sonian
d. Mosaic

AIIMS NOVEMBER 2011


Q 1. PGE causes worsening in infant with:
a. PS without VSD
b. Hypoplastic left heart syndrome
c. Obstructive TAPVC
d. Obstruction in aorta
Q 2. Premature baby of 34 wks was delivered. Baby had bullous
lesion on the body x-ray shows periostitis. What is the next investigation.
a. VDRL for mother and baby
b. EIISA for HIV
c. PCR for T.B.
d. Hepatitis surface antigen for mother

Questions

77

Q 3. A newborn presents with congestive heart failure, on examination


has bulging anterior fantanelle with a bruit on auscultation.
Transtantanellar USG shows a hypoechoic midline mass with dilated
lateral ventricles. Most likely diagnosis is:
a. Medullo blastoma
b. Encephalocele
c. Vein of galen malformation
d. Arachnoid cyst
Q 4. What is the diagnosis in a patient who has situs inversus and
sinusitis:
a. Kartagener's syndrome
b. Good pasture syndrome
c. Cystic fibrosis
d. William Campbell syndrome
Q 5. Short child with low T4 and raised TSH and swelling of pituitary,
what is the diagnosis?
a. Pituitary tumour
b. Primary hypothyroidism
c. TSH secreting pituitary adenoma
d. TSH resistance
Q 6. Episodic anemia with jaundice since birth except:
a. G 6pd
b. Sickle cell anemia
c. PNH
d. Hereditary spherocytosis

Pre NEET Pediatric


INCLUDING RECENT

ALL INDIA AND AIIMS TOPICS

ANSWERS
CLINICAL QUESTIONS AND QUESTIONS BASED ON
EXPECTED NEET PATTEN
1. Ans. is b. Partial exchange transfusion
(Ref: Nelson 18th, Pg. 1773, Meharban Singh 7th, Pg. 305)
This is a case of neonatal polycythemia which is symptomatic in
the form of lethargy and GIT manifestations
The management of choice for such a case is partial exchange
transfusion.
The diagnosis of polycythemia is made when venous or central
hematocrit is greater than 65%

Etiology
1. Delayed clamping of cord
2. Twin-twin transfusion syndrome. Causes polycythenia in the recipient
twin
3. Infants of diabetic mother
4. Intra uterine growth retarded babies due to chronic placental
insufficiency.
5. Rare causes: Cyanotic congenital heart disease, Cretinism, Trisomy
13,18 and 21, Beckwtih-Wiedemann syndromes, dehydration.

Clinical Features
1. CardiopulmonaryTacypnea, Tacycardia, cyanosis and
cardiomegally.
2. CNSLethargy, hypotonia, poor feeding, seizures, irritability.
3. Transient renal failure due to reduced perfusion of kidneys.
4. Hyperbiliribenemia
5. Thrombotic complications including renal vein thrombosis
6. Hypoglycemia, hypocalcemia, hypomagnesemia

Answers

79

Treatment

The volume for exchange is determined by the following calculation:


Volume = (Observed HCt desired HCt)/Observed HCt X
Blood volume
HCt = hematocrit
Desired HCt = 50% (usually)
Blood volume = 80 ml/kg for term babies
100 ml/kg for preterm babies.
2. Ans. is c. Orogastric tube/ alternate oral route
(Ref: Meharban Singh 7th, Pg. 191)
The mode of initial feeding depends upon the gestational maturity and
hemodynamic stability of the baby.
Infants with severe birth asphyxia respiratory distress, apneic
attack, sezures, sepsis, NEC, shock babies should be maintained
on intravenous infusion till their clinical condition stabilizes.

80

Pre-NEET Pediatrics

Mode of Initial Feeding Based on Gestational Maturity


Gestational age
(Birth weight)

Maturation of feeding skills

Initial feeding method

<30 wks(<1200 gm)

No proper sucking
efforts
No propulsive gut motility

Intravenous fluids

3032 wks
(12001500)

Sucking bursts develop


No coordination between
suck/swallow and
breathing

Oro-gastric
Or Naso - gastric
feeds

3234 wks
(15001800 gm)

Slightly mature sucking


Coordination between
breathing and swallowing
begins

Spoon, paladay
Or cup feeding

>34 wks (>1800 gm) Mature sucking


More coordination
between breathing and
swallowing

Breast feeding

Trophic feeds are Minimal (sub nutritional) feeds (510 ml/kg/d) used in
preterm babies with a gestation of <32 wks or birth weight <1500 gm
to harness its trophic benefits on the immature gastro intestinal tract of
the baby.
3. Ans is d. IvIg
(Ref: Nelson 19th, Pg. 863-865)
This is most likely a case of Kawasaki disease (KD) formerly known as
mucocutaneous lymph node syndrome and infantile polyarteritis nodosa.
The characteristic BCG scar induration, conjunctivitis, skin rash
and fever not responding to antibiotics point to diagnosis.
Classic clinical criteria for Kawasaki disease are:
Fever persisting at least 5 days and presence of at least 4 principal
features:
a. Changes in extremities
i. Acute: Erythema of palms, soles, edema of hands, feet.
ii. Subacute: Periungual peeling fingers, toes in weeks 2 and 3
b. Polymorphous exanthema
c. Bilateral bulbar conjunctival injection without exudate

Answers

81

d. Changes in lips and oral cavity: erythem, lip cracking, strawberry


tongue
e. Cervical lymphadenopathy (>1.5 cm diameter), usually
inilateral.

Etiology
The cause of KD remains unknown, but certain epidemiologic and clinical
features support an infectious origin KD is a vascultis that predominantly
affects the medium sized arteries especially the coronary arteries.
In the absence of treatment, KD can be divided into 3 clinical
phases
1. The acute febrile phage: lasts 1-2 wks
2. The subacute phase: associated with desquamation,
thrombocytosisand development of coronary aneurysms and
generally lasts about 2 wk.
3. The convalescent phase: begins when all clinical signs of illness
have disappeared and continues until ESR returns to normal (about
6 8 wk after onset of illness)

Treatment of Kawasaki Disease


Acute stage
Interavenous immunoglobulin 2 gm/kg X 10-12 hr and
Aspirin: 80-100 mg/kg/d until patient is afebrile for at least 48 hr.

Convalescent Stage
Aspirin 3-5 mg/kg until 6-8 wk after illness onset.

Long Term Therapy for Patients with Coronary


Abnormalities

Aspirin 3-5 mg/kg once daily clopidrogel


Warfain/ LMW heparin added for those patients at high risk of
thrombosis

Acute Coronary Thrombosis


Prompt fibrinolytic therapy with tPA or other thrombolytic agent.

82

Pre-NEET Pediatrics

4. Ans is b. Pyeloplasty
(Ref: Nelson 18th, Pg. 2237, Paediatric Nephrology 5th, Pg. 446-448)
The child in question is presenting with characteristic feature of PUJ
obstruction (dilated renal pelvis with normal ureters).

PUJ (Pelvi Uretric Junction) Obstruction


It is caused mostly due to intrinsic muscular abnormalities of the ureter,
ureter polyps, ureter folds, crossing vessels and rarely secondary to
vesicoureteral reflux. The condition is often asymptomatic and detected
on antenatal ultrasonography. Incidental detection of an abdominal
mass, hamaturia and UTI are usual features in the older child.
Ultrasonography and DTPA renogram establish the diagnosis.

Epidemiology
Left side is involved more commonly than right (Bilateral = 10%)
Male more commonly involved than females (M:F = 2:1)

Management

Answers

83

A differential function of <40% is considered significant and often an


indication for pyeloplasty. Differential function of <10% is often
considered a cut off for Nephrectomy.
5. Ans is c. Lung metastasis is common
(Ref: Nelson 19th, Pg. 1753-1757)
Neuroblastoma is the most common extracranial solid tumor in children
and the most commonly diagnosed malignancy in infants. It is derived
from primordial neural crest cells and approximately half arise in the
adrenal glands and remainder in the paraspinal sympathetic ganglia.
Pathogenesis: Etiology most cases remains unknown.
Familial neuroblastoma accounts for 1-2% of all cases and associated
with a younger age at diagnosis. Genetic characteristics that are of
prognostic importance include:
i. Poor prognosis: Amplication of MYCN, loss of heterozyqosity of
1p, 11q and 14q gain of 17q.
ii. Good prognosis: Hyperdiploidy if child is <1yr at diagnosis

Clinical Manifestations
1. Localized disease: asymptomatic mass or mass related symptoms,
including spinal cord compression, bowel obstruction, SVC
syndrome.
2. Metastatic disease: Fever, Bone pain, cytopenias, orbital proptosis,
subcutaneous nodules, periorbital ecchymosis.
3. Most common sites of metastasis are lymph nodes, bones (long),
skill, bone marrow, liver and skin. Lung and brain metastasis are
rare.

Diagnosis

Usually discovered as a mass on plain radiography, CT or MRI


Tumor markers including catecholamine metabolites homovanillic
acid (HVA), VMA (vanillylmandelic acid) are elevated in urine in
95% of cases.
Biopsy from tumor tissue.

Treatment
1. Low risk
Localized tumor: Treatment surgery survival rate 98%.
2. Intermediate risk

84

Pre-NEET Pediatrics

Localized tumor with regional lymphnode extension,


metastasis to bone marrow and in infants.
Treatment: Moderate intensity chemotherapy, debulking
surgery
Survival rate: 90-95%
3. High risk
Metastasis to bone marrow and bone.
Dose intensive chemotherapy, surgery and external beam radiotherapy
to primary tumor and resistant metastatic sites.
Survival rate: 40-50%
4. Tumor stage 4S
Metastasis to liver and skin
Treatment: Supportive care because many regress spontaneously survival
rate (>90%).

Current induction chemotherapy for high risk neuroblastoma


includes cyclophosphamide, topotecan, doxorubicin, vincristine,
cisplatin and etoposide.

6. Ans is d. Peritoneal drainage


(Ref: Meharban Singh 7th, Pg. 405-407, Nelson 18th, Pg. 756)
The definitive treatment of choice for perforation in cases of NEC is
surgery (immediate laparotomy with resection of the involved segment
and reanastomosis). However, in premature infants with very low birth
weight (<1000gm), who are extremely unstable for surgery (preterm
baby on ventilator), peritoneal drainage under local anaesthesia has
been described as a safe immediate management option.

Necrotizing Enterocolitis
Vanous predisposing factors are:
Prematurity (especially <34 wks)
Perinatal hypoxia
Active resuscitation
RDS/ apnea
Assisted ventilation
Acidosis/ Hypoxia/ Shock
Umbilical artery catheterization
Use of H2 blockers

Answers

85

Early/large volume nasogastric feedings


The protective role of human milk for prevention of NEC is well
accepted.

Modified Bells Staging for NEC


Stage

Signs

Radiology

Treatment

IA (Suspected
NEC)

Lethargy apnea,
bradycardia,
elevated
pre-gavage
residuals,
emesis,
guaic positive
stool

Normal or
intestinal
dilation,
mild illeus

NPO,
Antibiotics

IB (Suspected
NEC)

Same as above
+ bright red
blood per rectum
Same as above
+ absent bowel
sounds

Same as
above

Same as
above

Same as
above +
pneumatosis
intestinalis

Same as
above

IIA (Definite
NEC)

IIB (Definite
NEC)

Same as above
+ mild
thrombocytopenia,
acidosis +
abdominal
cellulites

Same as IIA
+ portal vein
gas

Same +
Sodabicarbonate for
acidosis

IIIA (Advanced
NEC)

Same as IIB
+ DIC, neutropenia + signs
of peritonitis

Same as IIB
+ definite
ascitis

Same as
above +
assisted
ventilation
paracentesis

IIIB (Advanced
NEC)

Same as IIIA

Same as
IIIA + pneumo
peritoneum

Same as IIIA
+ Surgical
intervention
(Laparotomy)

86

Pre-NEET Pediatrics
In ELBW (<1000 gm) or unstable babies, peritoneal drainage is
established under local anaesthesia and laparotomy is delayed for
stage IIIB.

7. Ans is c. Hospitalization and Nebulization with racemic epinephrine


diluted with water.
(Ref: Nelson 18th, Pg. 1765, O.P. Ghai 7th, Pg. 351)
This is a case of infectious croup (Acute laryngotracheobronchitis) with
moderate severity and requires hospitalization and nebulisation with
racemic epinephrine.
Laryngo Tracheobronchitis, usually caused by parainfluenza type
1. Bacterial etiology is unusual.

Clinical Features
Usually there is mild cold for a few days before the child develops a
brassy cough and inspiratory stridor. The condition may worsen to
severe respiratory distress and cyanosis,
Croup is a clinical diagnosis and does not require a radiograph of
the neck. However, characteristic "Steeple sign" (Subglottic narrowing)
may be seen on PA view of neck.

Severity of Acute LTB


Mild

Moderate

Severe

General
appearance

Happy

Irritable

Agitated or altered
sensorium

Stridor

Stridor on coughing
or no stridor
at rest

Stridor at rest,
gets worse on
agitation

Stridor at rest and


worsens on agitation

Respiratory
distress

No distress

Tacypnea, chest Marked tacypnea with


retractions
chest retractions

Oxygen
saturation

>92% in room air

>92% in room
air

<92% in room air,


cyanosis

Management
Mild cases can be managed on ambulatory basis with symptomatic
treatment for fever Antibiotics have no role.
Moderately severe patient may need hospitalization and treatment
with racemic epinephrine diluted with water.

Answers

87

A single intramuscular dose of dexamethasone (0.3 - 0.6 mg/kg)


reduces overall severity during first 24 hrs. More recently inhalation of
budesonide (1mg BD) x 2 days has shown good results.
Severe croup may need hospitalization with oxygen inhalation,
steroids. Worsening distress may need short term ventilation.

Differential Diagnosis
1. Epiglottitis: H. Influenza type B is the most frequent causative agent.
a. Clinical features: High fever with difficulty in swallowing. He
often sits up leaning forwards and saliva dribbling from his
chin.
b. Diagnosis: Direct laryngoscopy where epiglottis appears angry
red.
c. Management: Hospitalization, Humidified oxygen, fluids,
antibiotics (ampicillin/chloramphenicol/ceftriaxone)
Ventilation.
2. Spasmodic croup: Occurs in children between 1 and 3 yrs. Usually
no prodromal symptoms, sudden onset brassy cough and noisy
breathing, usually in early hours of morning. Pathogenesis is
unknown.
Treatment: Racemic epinephrine and steroids are helpful.
8. Ans is a. EDTA
(Ref: Nelson 19th P. 2450)
The child in question is presenting with signs and symptoms of plumbism
as suggested by blue line on gums (Bertonian line), gastrointestinal
manifestations like constipation and symptoms of lead encephalopathy
(seizures, drowsiness).
As the patient in question in presenting with encephalopathy, he is
likely to have blood levels >70 g/dL and hence, EDTA is the single
most important agent for treatment.

Lead Poisoning
Lead is a heavy metal that is purely a toxicant in humans. Lead poisoning
may occur in utero, because it readily crosses the placenta from maternal
blood. The spectrum of toxicity is similar to that experienced by children
after birth.
Major sources of exposure include lead based paints, batteries;
cable sheathing, cosmetic, plastic, toys, mineral supplements.

88

Pre-NEET Pediatrics

Metabolism

After absorption, lead is disseminated throughout the body. Mostly


it accumulates in bone and resides for years.
About 97% in blood is bound on or in Red blood cells.
The heme pathway is susceptible to lead inhibitory effects.
Neurotransmitter release is adversely affected by lead.

Clinical Symptoms
1. GIT symptoms: anorexia, abdominal pain, vomiting, constipation
2. CNS symptoms: related to cerebral edema and increased intracranial
pressure.
Headache, Lethargy, Seizures, Papilledema, Coma.
3. Renal tubular dysfunction at high levels (>100 g/dL)
Reversible fanconi syndrome.
4. Hemolytic anemia

Diagnosis and Management


Blood lead levels (BLL) remains the gold standard for evaluation. Lead
can be measured in urine and hair but has problems of contamination
and interpretability.

Drug Treatment
A child with a venous BLL 45 g/dL or higher should be treated. Children
with BLL of 44-70 g/dL may be treated with a single drug, preferably
DMSA.
Those with BLL of 70 g/dL or greater require 2 drug treatment.
1. EDTA + DMSA / BAL for those without encephalopathy
2. EDTA and BAL for those with encephalopathy
9. Ans. is a. No IgG deposits or C3 deposits on renal biopsy
(Ref: Nelson 19th, Pg. 1804 Paediatric Nephrology 5th, Pg. 195)
This girl is having generalized edema, grade 3 proteinuria, Fatty + hyaline
casts. This is a case of minimal change Nephrotic syndrome.
IgA nephropathy and Alport's syndrome present with acute nephritic
syndrome rather than nephrotic syndrome.
Nephritic syndrome is characterized by massive proteinuria (3+ 4+ protein), Hypoalbuminemia (<2.5 gm/dL) Hypercholestrolemia,
edema.

Answers

89

Approximately 90% children with Nephrotic syndrome are idiopathic


that is nephrotic syndrome. without evidence of a specific systemic
cause.
It includes: Minimal change disease (about 85% of total cases)

Mesangial proliferation

Focal segmental glomerulo sclerosis

Membranous nephropathy

Membranoproliferative glomerulonephritis

Differential Diagnosis

Protein losing enteropathy

Hepatic failure

Heart failure

Protein malnutrition
Chronic glomenlonephritis

Treatment
Children with onset of uncomplicated nephritic syndrome between 1-8
yr of age are likely to have steroid responsive MCNS (Minimal change
NS)and steroids therapy may be initiated without a diagnostic renal
biopsy. Children with features that make MCNS less likely (Gross
hematuria, hypertension, renal insufficiency, hypo-complementenia, age
<1 yr or >8yr) should be considered for renal biopsy before treatment.
About 90% children with MCNS respond to initial steroid therapy.
The initial episode is treated with prednisolone, 2 mg/kg/d for 6 wks.
Thereafter, the dose of prednisolone is reduced to 1.5 mg/kg given on
alternate days as a single morning dose for 6 more weeks.
Further prolongation of alternate day prednisolone therapy for 6
months may be beneficial in reducing the risk of subsequent relapses.
10. Ans. is b. Short course of steroids
(Ref: Paediatric Nephrology 5th, Pg. 155-157)
This is a classical case presentation of Henoch - Schonlein purpura
with skin, joint, GIT and renal manifestation. Use of oral steroids for 23 wks is recommended. Skin biopsy is not mandatory for diagnosis.
Henoch: Schonlein purpura

90

Pre-NEET Pediatrics

(Anaphylactoid purpura)
It is the most common systemic small vessel vasculitis in children. It
occurs frequently between 3 and 10 yrs.

Diagnosis
Presence of purpura (commonly palpable) or petechiae with lower limb
predominance and one of the four following criteria:
1. Abdominal pain
2. Arthritis/ arthralgia
3. Renal: involvement (proteinuria or hematuria or presence of RBC
casts)
4. Histopathology showing Leukocytoclastic vasculitis with
predominant IgA deposits in skin or proliferative glomerulonephritis
with predominant IgA deposits in mesangium on kidney biopsy.

Treatment
1. Joint symptoms only: Rest, analgesia
2. GIT manifestations, orchites, marked subcutaneous edema ' steroids
(prednisolone x 2-3 wks)
3. Severe Nephritis or nephritic syndrome, pulmonary hemorrhage
'steroids and cytotoxic drugs.
11. Ans. is b. Oral phosphate and Vit D supplements
(Ref: Paediatric Nepthrology 5th, Pg. 324-335)

Approach to Refractory Rickets


This case is a classical presentation of hypophosphatemic rickets (normal
S. Calcium, low S. Phosphate, Normal S.PTH) and treatment of choice
is phosphate and Vit D supplements.
This is clearly a refractory rickets that has not responded to 2 doses
of Vit D.

Remember

S. Phosphate high in chronic renal failure


S. Parathyroid hormone normal in hypophosphatemic rickets.

Answers

91

Hyphosphosphatemic Rickets

Most common form of refractory rickets, inherited as


a. X-linked dominant: Most common
Mutations in the PHEX gene
(chromosome Xp22.1)
This gene is a regulator of FGF - 23
production by osteocytes.
b. Autosomal dominant
c. Autosomal recessive
d. Autosomal recessive with hypercalciuria

Remember
In hypophosphatemic rickets, symptoms of hypocalcemia (tetany, muscle
weakness) are absent.
Dental abnormalities are common in hypophosphatemic x linked
rickets.

92

Pre-NEET Pediatrics

Treatment
1. Phosphate supplements (Joulie solution: Sodium phosphate,
phosphoric acid)
2. Vitamin D supplementation (except hypercalciuric variety)

Vit D Dependent Rickets (VDDR)


Autosomal recessive, present between 3 - 6 months of age.
1. VDDR type I (Pseudo Vit D deficiency)
Due to deficiency of enzyme 1 hydroxylase
Clinical and biochemical features resemble nutritional rickets
but are not corrected by Vit D supplementation.
Treatment: Calcium, phosphate supplements, calcitriol
2. VDDR type II
Due to end organ resistance to 1,25 (OH)2 D3
High prevalence of alopecia, ectodermal defects.
Refractory to treatment - Require large doses of calcitriol, calcium
and phosphate.

Renal Tubular Acidosis

Fanconi syndrome, distal RTA present with refractory rickets.


Treatment: Bicarbonate and phosphate supplementation

Chronic Renal Failure


Treatment: Restricting phosphate, calcium supplements, calcitriol.
12. Ans is d. Prophylactic antibiotics
(Ref: Nelson 18th, Pg. 2228-2233)

Vesicoureteral Reflux
Retrograde flow of urine from the bladder to the ureter and renal pelvis
is called vesicuureteral reflux.
Reflux is usually congenital and affects approximately 1% of
children. Reflux predisposes to renal infection, reflux nephropathy (renal
insufficiency).

Classification
Based on appearance of urinary tract on a contrast voiding cystourethrogram (VCUG)

Answers
1. Grade I
2. Grade II

93

:
:

reflux into a non dilated ureter


reflux into the upper collecting system without
dilatation
3. Grade III : reflux into dilated ureter
4. Grade IV : reflux into a grossly dilated ureter.
5. Grade V : massive reflux with ureteral tortuosity, loss of
papillary impression.
Clinical manifestation: Reflux usually is discovered during evaluation for
a UTI. Among children with reflux, 80% are female.

Treatment
With bladder growth and maturation there is a tendency for reflux to
resolve over time especially lower grades.
Grade 5 reflux rarely resolves.
Surgical correction of VUR also advised for
1. Multiple recurrences of UTI while on prophylaxis
2. Non compliance to medical management
3. Parental preference for surgery
4. Appearance of new renal scars during medical therapy.

American Urological Association Recommendations


for VUR.
Grade

Age (yr)

Scarring

Initial

Treatment

I - II

Any

+/-

Antibiotic

Prophylaxis

III - IV

0-5

+/-

Antibiotic

Prophylaxis

III - IV

6 - 10

+/-

Unilateral

Antibiotic

prophylaxis
Bilateral

Surgery
Prophylaxis

<1

+/-

Antibiotic

1-5

(-)

Unilateral

Antibiotics
prophylaxis

Bilateral
V

>1 yr

(+)

Surgery

13. Ans is c. Bartter syndrome


(Ref: Paediatric Nephrology 5th, Pg. 318)

Surgery

94

Pre-NEET Pediatrics

The boy in question has


Hypokalemia

Hyponatremia

Alkalosis

Hypochloremia

S.K+ 3 meq/L
S.Na+ 125 meq/L
PH 7.46
S.CI-88 meq/L

Medullary Nephrocalcinosis USG KUB


This is a case of Bartter syndrome. Renal tubular acidosis can be easily
ruled out as there is no acidosis but alkalosis.
In diabetes Insipidus, there is deficiency of vasopressin, [(central)
or tubular unresponsiveness to ADH (nephrogenic)]. Hence, there is
loss of free water that leads to hypernatremia dehydration with
corresponding increase in serum chloride. But in question there is
hyponatremia and hypochloremia.
In pseudohypo aldosteronism, S.aldosterone is normal or high but
due to tubular unresponsiveness, clinical features of aldosterone deficiency
ie. Hyperkalemia, hyponatremia, acidosis will be seen which is not the
case in this situation.

BARTTER SYNDROME
It is an autosomal recessive disorder, characterized by hypokalemia,
metabolic alkalosis, hyperreninemia and hyperaldosteronism, normal
blood pressure and urinary wasting of K+, Na+ and Cl-.
Clinical features include failure to thrive, polyuria and polydipsia
with recurrent episodes of dehydration, muscle weakness and cramps in
older children.
Molecular defect is in the thick ascending limb of Henle. Tubular
losses of K+, Na+, Cl- and water lead to volume contraction that in
turn causes secondary hyperaldosteronism.
Bartter syndrome should be differentiated from non renal causes of
chloride loss such as vomiting, cystic fibrosis. In these conditions the
urinary chloride concentration is in variably below 10meq/L.
Therapy: Potassium supplementation and Indomethacin (cyclo oxy
genase inhibitor decrease elevated prostaglandins that are responsible
for polyuria).

GITELMAN SYNDROME
Hypokalemia, metabolic alkalosis and hypomagnesemia are chief
abnormalities.
The combination of Hypocalciuria with renal magnesium wasting
distinguishes Gitelman from Bartter syndrome

Answers

95

Clinical Features
Generally milder symptoms. Appear in older children. There are episodes
of muscular weakness, cramps, fatigue, vomiting, tetany, polyuria and
growth retardation are mild.
Molecular defect is in the apical thiazide sensitive, sodium chloride
co-transporter (NCCT) in distal tubule. Features of Gitelman syndrome
mimic Chronic thiazide administration.
Treatment is with supplementation of potassium and magnesium
chloride.
14. Ans is b. Long term phenytoin is required
(Ref: Nelson 19th, Pg. 2017-18)
This is a case of recurrent febrile seizure which can be treated with
intermittent clobazam and paracetamol. The risk of epilepsy is 4%.
Considering this to be a low intermediate risk, EEG and neuroimaging
may be considered. But certainly long term phenytoin therapy is not
required unless EEG/ Neuroimaging turn out to be abnormal.

Febrile Seizures
Febrile Seizures are seizures that occur between the age of 6 months - 5
yrs with a temperature of 380C or higher, that are not the result of CNS
infection or any metabolic imbalance and that occur in the absence of
a history of prior afebrile seizures.
A simple febrile seizure - generalized, usually tonic clonic associated
with fever, lasting for a maximum of 15 min and not recurrent within a
24 hr period.
A complex febrile seizure is more prolonged >15 min, is focal, and/
or recurs within 24 hours.

Risk Factors for Recurrence of Febrile Seizures


Major

Minor

Age < 1yr

Family history of febrile seizures

Duration of fever <24 hr

Family history of Epilepsy

Fever 38-390C

Complex febrile seizure


Male gender
Lower S. Sodium

96

Pre-NEET Pediatrics

Risk Factors for Occurrence of Subsequent Epilepsy


Risk factor

Risk for Epilepsy

1%
33%
29%
18%
11%
4%

Simple febrile seizure


Neuro developmental abnormalities
Focal complex partial seizure
Family history of epilepsy
Fever< 1hr before febrile seizure
Recurrent febrile seizure

Investigations
Lumbar puncture is recommended in children <1yr of age after their
first febrile seizure or if clinical signs and symptoms suggest meningitis

EEG/ Neuro Imaging


Should be done in selected cases with a high/intermediate risk of epilepsy.

Treatment

Rectal Diazepam/ Midazolam/ Lorazepam given at time of Seizure.


Febrile status epilepticus - I/V phenobarbitone/ phenytoin/ valproate.
Intermittent oral Diazepam/ Clobazam/ Clonazepam can be given
along with anti pyretics during febrile illness to reduce the risk of
seizure.
Chronic antiepileptic therapy - may be considered for children with
a high risk for later Epilepsy.

15. Ans is d. EEG in this condition shows a 4-6/ sec irregular spike
and wave pattern, which is enhanced by photic stimulation.
(Ref: Nelson 18th, Pg. 2463)
This is a classical case presentation of juvenile myoclonic epilepsy (JANZ
syndrome). It begins usually between ages of 12-16 yrs. Patients note
frequent myoclonic jerks on awakening. A few years later, early morning
GTCS develop in association with myoclonus. EEG shows a 4-6/sec
irregular spike and wave pattern, which is enhanced by photic stimulation.
Drug of choice is Valproate and not carbemazepine. This is required
lifelong. Discontinuation of drug causes a high rate of recurrence of
seizures.
Other types of Myoclonic Epilepsies are:
1. Benign myoclonus of infancy: EEG is normal, No anticonvulsant
are required.

Answers

97

2. Typical myoclonic epilepsy of early childhood.


a. Mean age of onset 2yrs
b. At least 1/3 of children have a positive Family history of epilepsy
c. Premorbid development is normal
d. EEG shows fast spike wave complex 2.5 Hz and a normal
back ground rhythm.
3. Complex Myoclonic Epilepsies:
a. A history of hypoxic ischaemic encephalopathy in the perinatal
period and abnormal neurological examination is a common
finding.
b. Focal or GTCS beginning in 1st yr of life antedate the onset of
myoclonic epilepsy.
c. It has a poor prognosis
Lennox Gastaut syndrome: Characterized by intractable seizures
of various types, a slow spike wave EEG and mental retardation.
Landau Kleffner Syndrome: Characterized by receptive aphasia,
seizures and behavioural problems. Valproic acid is the drug of
choice.
16. Ans is b. Tuberous sclerosis
(Ref: Nelson 18th, Pg. 2485, O.P. Ghai 7th, Pg. 564)
There are 5 Major Neurocutaneous Syndromes
i. Neurofibromatosis
ii. Tuberous sclerosis
iii. Struge Weber syndrome
iv. Von Hippel Landau disease
v. Ataxia telengiectasia

Neurofibromatosis
Inheritance is autosomal dominant
There are 2 types
1. Type I (Von Recklinghausen disease/ Peripheral NF1)
2. Type II (Central NF)
NF1: Deletion of NF gene on chromosome 17
NF2: Gene is probably located on Chr. 22
NF1: 2 or more of the following are present:
1. 6 or more Cafe-au-lait spots, each over 5 mm in diameter
before puberty or over 15 mm diameter in older persons.
2. 2 or more neurofibromas or one plexiform neuroma
3. Axillary/ Inguinal freckling

98

Pre-NEET Pediatrics

4. Optic glioma
5. 2 or more Lisch nodules, dysplasia of sphenoid bone
6. A first degree relative with NF1
NF 2 - Presence of Bilateral auditory neuroma, unilateral auditory
neuroma along with a first degree relative with meningioma, schwanomas
or juvenile posterior subcapsular lenticular opacity.

Tuberous Sclerosis

Autosomal dominant inheritance


Cardinal features: Skin lesion, Convulsions and mental retardation
Skin lesions: Hypopigmented ash leaf shaped macules, adenoma
sebacum (angiofibromas) shagreen patches, subungal fibroma
In infancy, myoclonic jerks often occur (West syndrome) - Drug of choice
Vigabatrine

Sturge - Weber - Syndrome


Characterized by facial nevus flammeus (usually in the distribution of
first branch of trigeminal nerve), contralateral focal seizures, calcification
of cortex (tram track calcification), glaucoma on the same side.

Von Hippel Landau Disease


There are retinal and cerebellar hemangioblastomas besides spinal cord
angiomas and cystic tumors of pancreas, Kidneys and epididymis.

Ataxia - Telengiectasia

An autosomal recessive inherited disease (mapped to Chr. 11q)


Usually manifests with progressive cerebellar ataxia, Oculocutaneous
telengiectasia, Choreoathetosis, pulmonary and sinus infections,
immune deficiency and lymphorecticular malignancies.
feto protein is almost always elevated.
Neuro-imaging reveals cerebellar atrophy.

Treatment is Symptomatic
17. Ans is d. Medullobastoma
(Ref: Nelson 18th, Pg. 2452, 2427)
Options a,b,c are congenital conditions whose presentations vary as
follows:

Answers

99

1. Chiari Malformation

Type I:
Typically produces symptoms during adolescence /adult life.
Patient complains of recurrent headache, urinary frequency and
progressive lower limb spasticity.
The deformity consists of displacement of the cerebellar tonsils
into the cervical canal. It is usually not associated with
hydrocephalus.
Type II:
Characterized by progressive hydrocephalus with a
myelomeningocele.
Usually presents in childhood with gait abnormality, spasticity
and incoordination. The deformity consists of elongation of
4th ventricle with displacement of inferior vermis, pons and
medulla into cervical canal.

2. Dandy Walker Malformation

Consists of a cystic expansion of the 4th ventricle in the posterior


fossa and midline cerebellar hypoplasia
Approximately 90% patients have hydrocephalus. Most children
have evidence of long tract signs, delayed motor and cognitive
milestones, cerebellar ataxia.

3. Syringobulbia

Syringomyelia is a cystic cavity within the spinal cord that may/


may not communicate with the CSF pathways.
Syringobulbia exists when the cystic cavity extends into the medulla.
Communicating syringomyelia is frequently associated with Chiari
type I malformation.

Clinical Feature
It may rarely produces symptoms during childhood due to its slow
evolution.
Asymmetric loss of pain and temperature sensation with preservation
of light touch can occur (due to disruption of lateral spinothalamic
tracts)
Later, upper motor signs develop as cavity impinges on the
corticospinal tract.

100

Pre-NEET Pediatrics

4. Medulloblastoma

CNS tumors are the second most common malignancy in childhood


after leukemias.
<1 yr: Supratentorial tumors predominate
1-10 yr: Infratentorial tumors predominate
>10 yrs: Supratentorial tumors again predominate with diffuse
astrocytomas most common.
Medulloblastoma is a cerebellar tumor which occurs predominantly in
males and at a median age of 5-7 yr of age.

Clinical Features
1. Increased ICP (ie. Headache, Nausea, Vomiting, Hypertension)
2. Cerebellar dysfunction (ataxia, poor balance, dysmetria)
Medulloblastoma is the childhood brain tumor that most
commonly metastasizes extraneuronally.
Medulloblastoma is sensitive to both chemotherapy and radiation theray
with surgery as the starting point of treatment.

Other CNS Tumors


a. Astrocytoma account for majority (40%) of CNS tumors.
Juvenile pilocytic astrocytoma (JPA) is the most common
astrocytoma and with a good prognosis.
b. Craniopharyngioma: occur in the suprasellar region
Clinical feature: Panhypopituitarism, growth failure, visual loss.
Surgery is the primary treatment modality as they are minimally invasive.
There is no role for chemotherapy in craniopharyngioma.
18. Ans is a. Give I/V bolus of normal saline
(Ref: Nelson 19th, Pg. 213-215)
This is a case of hypernatremic dehydration with shock and clinical
signs of hypernatremia - Doughy skin irritability. Cause in this case may
be improperly made ORS (more concentrated). Ina child with
hypernatremic dehydration, as in any child dehydration, the first priority
is restoration of intravascular volume with isotonic fucid. Normal saline
is preferred to Ringers solution because the lower sodium concentration
in the latter can cause the S.sodium to decrease too rapidly.

Answers

101

Other Causes of Hypernatremia


1.
2.
3.
4.
5.

Iatrogenic: Excess I/V hypertronic saline, I/V sodium bicarbonate


Hyperaldosteronism
Central and Nephrogenic Diabetes insipidus
GIT losses: diarrhea, Emesis
Renal losses: CRF, polyuric phase of ATN, Osmotic diuretics,
Diabetes mellitus.

Clinical Features

Doughy abdomen
CNS feature: irritability, lethargy, seizures (due to brain hemorrhage).
Fever
Thrombotic complications (secondary to dehydration and
hypercoagulability with hypernatremia)

Treatment
1. Restoration of Intravascular volume: Normal saline bolus.
2. Calculation of deficit and replacement over 24-48 hrs
Goal- decrease S. Sodium by <12 meq/L every 24 hr, a rate of 0.5
meq/L/hr
3. Peritoneal dialysis: Acute severe hypernatremia
19. Ans is d. X-linked dominant
(Ref: Nelson 19th, Pg. 388)
It is an X linked dominant pattern of inheritance
The features which suggest the X-linked dominant pattern are:
A single abnormal X Chromosome is sufficient to express the disease.
All of the female offsprings of a diseased male XY will receive the
abnormal X chromosome and express the disease, whereas none
of the male offspring to the diseased father will have the disease (as
sons don't receive X chromosome from father).
A diseased mother can transmit the abnormal
X chromosome to both daughter and sons equally.

102

Pre-NEET Pediatrics

X-lined recessive inheritance

Males are more commonly and severely affected than females.

Female carriers are generally unaffected.

Females carriers have a 25% risk of having an affected son, 25%


risk for a carrier daughter and 50% chance of having an unaffected
child.

Affected males will have only carrier daughters.

Affected males have no chance of having an affected son because


they will pass their Y chromosome to their sons. Male to male
transmission excludes X linkage, but is seen with autosomal
dominant and Y-linked inheritance.

20. Ans is d. Do not give OPV and BCG at birth.


(Ref: O.P. Ghai 7th, Pg. 207, Nelson 19th, Pg. 1158)
HIV in Newborn
HIV-1 and HiV-2 are members of the Retroviridae family.
HIV-2 is a rare cause of infection in children. It is most prevalent in
Western and southern Africa.
Transmission of HIV1 occurs via sexual contact, parenteral exposure
to blood, or vertical transmission from mother to child. Vertical
transmission can occur intrauterine (before birth), intrapartum during
birth), or through breast feeding (after delivery)

Answers

103

30-40% of infected newborns are infected in utero. The highest


percentage of HIV infected children acquire the virus intrapartum.
The risk of transmission through breast feeding in chronically infected
women is approx 9-16% Elective Cesarean delivery decrease transmission
by 87% if used in conjunction with Ziduvudine therapy in mother and
infant.
Transfusion of infected blood/blood products has accounted for 36% of all pediatric AIDS cases. In the pediatric population, sexual
transmission is infrequent.

Diagnosis
All infants born to HIV infected mother test antibody positive at birth
because of passive transfer of maternal HIV antibody across the placenta.
Therefore HIV EIISA (based on antibody) is done at 18 months of age
(By this time, almost all infants loose maternal antibody). Specific viral
diagnostic assays, such as HIV DNA or RNA PCR, HIV Culture or HIV
P24 antigen are essential for diagnosis of infant HIV infection.
HIV DNA PCR is method for diagnosis. 2 positive HIV DNA PCR
(1st performed at 6-8 wks of age) confirm HIV infection.
HIV P24 antigen assay is the less sensitive, HIV culture is technically
complex, expensive and time consuming (2-4 wks compared to 2-3
days with PCR) and with HIV RNA assays, data results are limited.

Prevention of Mother to Child transmission (MTCT)


Recommended Regime for pregnant (Not eligible for ART)
Antepartum: Zidovudine starting at 28 wks of pregnancy
Intrapartum: A combination of single dose Nevirapine, Zidovudine and
Lamivudine
Postpartum: A combination of Zidovudine and Lamivudine for 7 days.
Recommended treatment for pregnant women in labor and who have
not received ART is:
Intrapartum: A combination of Nevirapine, Zidovudiine, Lamivudine.
Postpartum: Combination of Zidovudine, Lamivudine x 7 wks.

104

Pre-NEET Pediatrics

Omission of Nevirapine for the mother may be considered for women


who receive at least 4 wks of Zidovudine before delivery.
Recommended regimen for Infants born to HIV positive mother
Nevirapine and Zidovudiine for 1 wk.
When delivery occurs within 2 hrs of women taking Nevirapine, the
infant should receive Nevirapine immediately and Zidovudine for
four wks. If mother receives less than 4 wks of antenatal ART, then
4 wks of Zidovudine is recommended.

Cotrimoxazole Prophylaxis
1. All HIV exposed infants starting at 4-6 wks of age and continued
till HIV infection can be excluded.
2. All HIV infected infants <1yr should receive prophylaxis regardless
of symptoms or CD4 percentage.
3. After 1 yr of age, prophylaxis is recommended for symptomatic
children or children with CD4 <25%.
Immunization: Symptomatic HIV infected children should not be given
OPV and BCG.
Breast feeding: According to WHO 2001, when replacement feeding is
available, avoidance of breast feeding by HIV infected mothers is
recommended. Otherwise, exclusive breast feeding is recommended
keeping in mind that mixed feeding during this period carries a 70%
greater risk of transmission.
21. Ans is b. Statement A is true while statement B is false.
(Ref: O.P. Ghai 7th, P. 573)
Statement 'A' is a classical example of spinal muscular atrophy (SMA)
while statement 'B' is EMG finding of myasthenia gravis.
1. Infant has areflexia (absent reflexes) but Deep tendon reflexes are
preserved in myasthenia gravis.
2. Myasthenia (congenital) does not present in the intrauterine life (so
does not cause polyhydramnios)
Hence, statement A is true but B is false.

Answers

105

Floppy Infant
He/she is an infant with marked hypotonia of the muscles and variable
associated weakness.
Common causes are:
1. CNS (Central nervous system)
Perinatal asphyxia, neonatal encephalopathy, cerebral palsy
(atonic type), Down syndrome, Inborn errors of metabolism
(amino aciduria, mucopolysaccharidosis, cerebral lipidosis)
2. Spinal cord: Anterior horn cell disease: SMA, polio
3. Peripheral Nerves: Familial dysautonomia, hereditary motor sensory
neuropathy
4. Myoneural junction: Neonatal myasthenia gravis, infantile botulism
5. Muscles: Muscualr dystrophies, congenital myopathies, polymyositis,
glycogen storage disease
6. Miscellaneous : PEM, hypothyroidism, rickets, Prader willi syndrome,
Ehler Danbos syndrome.

Clinical Features
Delayed motor milestones frequent, respiratory infections.
Antenatal H/O polyhydramnios, decreased fetal movements.

Clinical Examination
Frog like postures, diminished resistance to passive movements, excessive
joint mobility.

+++

Variable

Central

Anterior
horn cell

Peripheral
nerve

Neuromuscular
junction

Muscle

NCS - Nerve conduction studies

EMG - Electromyography

Extent of
weakness

Site of
involvement

++

++

++

+++

Arms

+++

++

+++

+++

Legs

>

<

> or =

> or =

Proximal
Vs distal
weakness

Decreased

Normal

Decreased/
absent

Absent

Normal/
increased

DTR

Short
duration
polyphasic
potentials

Decremental
response on
repetitive
stimulation

Abnormal
CNS
denervation
potentials

Denervation
potentials,
fasciculation's

Normal

EMG

Characteristic

Normal

Denervation
pattern

Neurogenic
atrophy

Normal

Muscle Biopsy

Differentiating Features of Floppy Infant According to the Site of Involvement

106
Pre-NEET Pediatrics

Answers

107

Spinal Muscular Atrophy


SMA is an autosomal recessive disorder characterized by muscle
weakness due to degeneration of motor neurons in the spinal cord
(anterior horn cells) and brainstem nuclei.
It can be inherited as autosomal dominant, X-recessive or sporadic.

Types
1. SMA type I: Werdnig Hoffmann disease. A classical patient presents
within first 6 months and is never able to sit. More than 90% die by
10 yr of age.
2. Type II: Manifest by first yr of age, unaided sitting is possible but
walking is not achieved. More than 90% survive beyond 10 yrs.
3. Type III: Kugelberg-Welander disease, can achieve walking without
aids.
4. Type IV: Presents after 30 yrs
Severity is variable
Lifespan is unaffected
The heart is not involved in SMA. Intelligence is normal.
22. Ans is b. Emergency atrial septostomy
(Ref: Avery 8th, Pg 841, Cloherty 5th, Pg. 428)
This is a classical case presentation of hypoplastic left heart syndrome
(ductal dependent lesion) which fails to respond to medical therapy. In
these neonates, in order to benefit from a PGE1 infusion, there must be
a patent foramen ovale to allow effective systemic blood flow (pulmonary
venous return) to cross the atrial septum and enter the systemic vascular
bed through the ductus. In these neonates, emergent balloon dilation of
atrial septum may be necessary. After a period of stabilization, corrective
surgery (Norwood procedure followed by a fontan operation later in
childhood) is planned.

Ductus Arteriosus Dependent Lesions


Ductus arteriosus is a ductal connection between aorta and pulmonary
artery which plays an important role in the fetus shunting pulmonary
blood flow to aorta. It is maintained patent by low oxygen and PGE1
during antenatal period.
After birth, rising concentrations of oxygen and fall in PGE1 induce
closure of ductus which is normally complete by 2-3 wks of age.

108

Pre-NEET Pediatrics

Closure of the Ductus Arteriosus is Particularly Deleterious


in Patients with Ductal Dependent
1. Systemic blood flow (SBF)
2. Pulmonary blood flow (PBF)
3. Parellel circulation

1. Congenital lesions with duct dependent systemic


blood flow.
i.
ii.
iii.
iv.

Critical aortic stenosis


Coarctation of aorta
Interruption of aortic arch
Hypoplastic left heart syndrome (HLHS)

Of the four, critical aortic stenosis manifests shortly after birth with
signs of CHF, gallop rhythm, murmur and poor perfusion. In other
three left sided lesions, most common presentation is shock (poor
perfusion) in first 1-2 wks of life.

2. Lesion with Duct Dependent Pulmonary Blood


Flow
a. Tricuspid atresia
b. Ebstein anomaly
c.

Pulmonary atresia with intact ventricular septum ("Hypoplastic right


heart syndrome")

d. Severe pulmonary stenosis

3. Parellel Circulation/ Transposition of Great


Arteries.
Treatment
Administration of PgE1 will open the ductus arteriosus and depending
upon the lesion, increase pulmonary blood flow, systemic blood flow or
intercirculatory mixing.
Failure to respond or clinical deterioration after institution of
PGE1 means
1. Initial diagnosis is incorrect

Answers

109

2. Ductus is unresponsive to PGE1 (eg. Older infant)


3. Total anomalous pulmonary venous return with obstruction.
4. Lesion with obstruction of blood flow out of left atrium
a. Hypoplastic left heart syndrome
b. Variants of mitral atresia with a restrictive foramen ovale
c. Transposition of great arteries with intact ventricular septum
and restrictive foramen ovale.
These 3 conditions require an emergent atrial septostomy to save the
neonate (as explained before).
23. Ans is c. Knee-chest position, oxygen and Inj. Morphine
administration
(Ref: Nelson 19th, Pg. 1573-1575)
This is a classical case presentation of a cyanotic spell due to most
probable Tetrology of fallot and most appropriate intervention would
be knee chest position, Inj. Morphine and Oxygen administration.
In the absence of definite history of similar spells in past and echo
study, this condition needs to be differentiated from breath holding
spells which also peak around similar age (1 - 2 yrs of age). The
presence of respiratory distress and systolic murmur with cyanosis
preceded by point toward crying a cyanotic spell. A breath holding spell
is of 2 types. They are:
i.

Cyanotic a breath holding spell:


a. Usually provoked by up sitting/ scolding an infant.
b. Brief shrill cry followed by apnea and cyanosis that may be
associated with generalised clonic jerks, opisthotonus
c. Management is parental reassurance and not reinforce the
child's behaviour after child's recovery from the spell.

ii.

Pallid spells - Much less common


a. Typically initiated by a painful experience eg. Falling, striking
head or sudden startle
b. Child stops breathing, looses consciousness, becomes pale and
may have a tonic seizure.

110

Pre-NEET Pediatrics

Treatment: Reassurance
Atropine (by blocking the vagus nerve) may be tried in refractory cases.

Tetrology of Fallot (Must know)


Pri mary defect is an anterior deviation of the infundibular septum
(septum that separates aortic and pulmonary outflows) consequence of
this deviation are 4 componnets:
i. Obstruction to right ventricular outflow (pulmonary stenosis)
ii. Malalignment type of Ventricular septal defect (VSD)
iii. Dextroposition of aorta so that is overrides the ventricular septum
iv. Right ventricular hypertrophy
Obstruction to pulmonary arterial blood flow is usually both
infundibular (subpulmonic area) and the pulmonary value (Rare)

Clinical Manifestations
1. Acyanotic/pink tetralogy of fallot: when obstruction to right
ventricular outflow is mild-moderate and a balanced shunt across
the VSD, the patient is not cyanotic.
2. Ductal dependent pulmonary blood flow in infants.
3. Older children: presentation with cyanosis, clubbing and dyspnea
on exertion.

Signs
1. Prominent left anterior hemi thorax in older children (due to long
standing right ventricular hypertrophy)
2. A systolic thrill/ systolic murmur in left sternal 3rd - 4th parasternal
space. It is caused by turbulence through right ventricular outflow
tract. It loudness directly proportional to severity of pulmonary
stenosis but it can become less prominent with severe obstruction
and hypercyanotic spell.
3. Single 2nd heart sound or soft pulmonary component.
Chest X-ray "Coeur in Sabot" (Boot shaped heart) due to cardiac apex
elevation (due to right ventricle hypertrophy) pulmonary oligemia.

Treatment of Cyanotic Spell


1. Placement of child in knee chest position (It increases systemic
vascular resistance and decrease venous return. This decreases right
to left shunt, and improves symptoms).

Answers
2.
3.
4.
5.

111

Oxygen administration
Morphine subcutaneous at dose not exceeding 0.2 mg/kg
Intravenous soda bicarbonate if metabolic acidosis present
Intravenous phenylephrine - increase systemic vascular resistance,
decrease right ' left shunt and improves symptoms.

Surgical Management
1. Palliative surgery - Blalock - Taussig shunt (systemic to pulmonary
artery shunt) performed to augment pulmonary artery flow.
Indication: Infants with less severe cyanosis without cyanotic
spells and with good growth.
Modified Blalock Taussig Shunt between subclavian artery and
pulmonary artery.
Waterson shunt between ascending aorta to main pulmonary artery.
Pott's Shunt: descending aorta to pulmonary artery.
2. Corrective surgery
a. Electively between 4-6 months of age in case of less severe
cyanosis without spells
b. Immediately in infants with severe cyanosis (marked right
ventricular outflow obstruction).
24. Ans is d. Do urgent USG and prepare for urgent laparotomy
(Ref: Nelson 19th, Pg. 1288-1289)
This is a classical presentation of Intussusception and in view of X-ray
features of peritoneal irritation (ascites with distended bowel loops),
hydrostatic reduction should not be attempted.
This is not a case of simple gastroenteritis because in enteritis, the
pain is less severe, there is diarrhea which is not the case in this clinical
problem.
Similarly Vit K deficiency is less likely as bleeding per rectum is
painless and there are no associated symptoms of excessive crying and
vomiting in Vit K deficiency.

Intussusception
It occurs when a portion of the alimentary tract is telescoped into an
adjacent segment.
The upper portion of bowel, the intussusceptions, invaginates into
the lower, the intussuscipiens.

112

Pre-NEET Pediatrics

Q. It is the most common cause of intestinal obstruction


between 3 month and 6 yr of age
Q. the most common abdominal emergency in children <2 yr.
Approximately 90% of cases are idiopathic. Swollen peyer patches, in
response to GIT infection or introduction of new food proteins may
lead to mucosal prolapse causing an intussusceptions.
In 2-8% patients, recognizable lead points are found eg. Meckel
diverticulum, polyp, neurofibroma, hemangioma, lymphoma
Q. Intussusceptions are most often Ileocolic

Clinical Features
Suddent onset in a previous well child of severe paroxysmal colicky
pain or episodes of excessive crying. Child may initially be comfortable
between paroxysms but progressively becomes weak and lethargic.
Vomiting occurs in most cases stools of normal appearance with
sometimes passage of blood.
Eventually a shock like states with fever develop.

Classic Triad
Pain, a palpable sausage shaped abdominal mass, and bloody or currant
jelly stool is seen in <15% of patients.

Abdomen Palpation
Usually (about 70% cases) reveals a slightly tender sausage shaped
mass, presence of bloody mucus on rectal examination supports the
diagnosis of Intussusception.

Diagnosis
Ultrasound
3.
4.
5.
6.

Sensitivity 98-100%
Specificity 88%
Tubular mass in longitudinal views
A doughnut or target appearance in transverse images.

Management
Immediate reduction of acute intussusceptions. In patients with prolonged
intussusception and signs of shock, peritoneal irritation, intestinal

Answers

113

perforation or pneumatosis intestinalis, hydrostatic reduction should


not be attempted. In such cases, urgent laparotomy should be planned.
25. Ans is a. 21 hydroxylase deficiency
To understand the pathophysiology and effects of different congenital
adrenal hyperplasia, one should know the synthetic pathway of steroid
hormones.

Horizontal Enzymes
3 3- hydroxystroid dehydrogenase
2 21 - hydroxylase
1 11 - hydroxylase

Vertical Enzymes
All starting with "17"
4
- 17 hydroxylase
5
- 17, 20 lyase
6
- 17 hydroxy steroid dehydrogenase
* 7 - Cholestrol desmolase
* 8 - Aldosterone synthase

114

Pre-NEET Pediatrics

CAH (Congenital Adrenal Hyperplasia)


CAH is a family of autosomal recessive disorders of cortisol biosynthesis.
Cortisol deficiency increases ACTH (corticotrophin) which leads to Adreno
cortical hyperplasia.

1. 21 - Hydroxylase Deficiency
More than 90% of CAH are caused by 21-hydroxylase deficiency.
Enzyme '2' in diagram is required for synthesis of Aldosterone and
cortisol. In its absence, progesterone and metabolites are diverted to
production of androgens.
a. Deficiency of mineralo corticoid ' salt losing - hypotension,
hyponatremia, hyperkalemia, acidosis
b. Excess androgens - Virilization of female. In males, there will
be precocioius puberty due to androgen excess.

2. 11 - Hydroxylase Deficiency
Enzyme '1' in diagram is required for cortisol synthesis
In its absence
a. Excess DOC (Deoxy corticosterone) ' mineralo corticoid '
Hypertension
b. Excess androgens ' Virilization of female, precocious puberty in
males

3. 3 Hydroxyl Steroid Dehydrogenase Deficiency


Enzyme '3' in diagram
Required for Aldosterone, cortisol and androstendione synthesis
In its absence
a. cortisol/ Aldosterone not synthesized that lead to salt wasting
crisis
b. DHEA Virilization of females (weak androgen)

4. 17 - Hydroxylase Deficiency
Enzyme '4' in diagram
Required for cortisol, Sex hormone synthesis in its absence, all
pregnenolone is converted to mineralocorticoid. So there will be:
a. Excess of mineralocorticoid ' Salt retention, hypertension
b. No androgens ' Feminization of external male genitalis, females
development will be normal.

Answers

115

Summary

In 21 OH, 3 OH dehydrogenase deficiency


Salt wasting crisis + female virilization
In 11 hydroxylase, deficiency and 17 hydroxylase deficiency
Hypertension present
11 hydroxylase female virilization (female pseudo
hermaphroditism)
17 hydroxylase male external genitalia feminization (male pseudo
hema phroditism)

26. Ans is c. A correct B wrong


(Ref: Nelson 18th, Pg. 609)
Feature of conjugated hyperbilirubenemia on the 3rd day accompanied
by sepsis and positive reducing substance in urine suggest diagnosis of
galactosemia.
The term Galactosemia generally designates the deficiency of
galactose - I phosphate uridyl transferase deficiency.
Without the transferase enzyme, the infant is unable to metabolize
galactose - I phosphate, the accumulation of which results in injury to
kidney, liver and brain.
Clinical Manifestations
In newborn or young infants, jaundice, hepatomegally, vomiting,
hypoglycaemia, cataracts, sepsis (esp. E coli) are main clinical features.
Symptoms improve when milk is temporarily withdrawn and replaced
with lactose free nutrition.
Diagnosis
Positive reducing substances in urine (Galactosuria)
Direct enzyme assay using erythrocytes
Treatment
Elimination of galactose from the diet reverses the growth failure,
Cataracts, renal and hepatic dysfunction.
Galactokinase Deficiency
The deficient enzyme is galactokinase, which catalyzes the phosphorylation
of galactose.

116

Pre-NEET Pediatrics

The principal metabolites accumulated are galactose and galactitol.


Cataracts are usually the sole manifestation.

Uridine Diphosphate Galactose 4 Epimerase Deficiency


The principal metabolitis accumulated are galactose 1 phosphate, UDP
- galactose.
There are two distinct forms of epimerase deficiency
1. Mild form: affected persons are asymptomatic enzyme deficiency
is limited to Leukocytes, erythrocytes. No treatment is required.
2. Severe form: clinical feature resemble transferase deficiency, with
additional symptoms of hypotonia, nerve deafness. Patients can't
synthesize galactose from glucose, hence they are placed on a
galactose restricted diet.
27. Ans is b. A and B both correct and B is not explanation of A
(Ref: Nelson 18th, Pg. 1672, O.P. Ghai 7th, Pg. 294)
This is a case of neonatal cholestasis possibly due to congenital Rubella.
But as proportion of EHBA (extra hepatic biliary atresia) patients show
TORCH antibodies, liver biopsy is considered as the most accurate (9095%) method to differentiate EHBA from neonatal hepatitis.
Neonatal cholestasis is defined as prolonged elevation of serum.
Conjugated bilirubin beyond teh 1st 14 days of life. Cholestatic jaundice
is always pathological and needs evaluation.
Broadly there are 4 pathophysiological patterns of neonatal
cholestasis:
i. Obstructive - EHBA
ii. Neonatal hepatitis
iii. Paucity of bile ducts
iv. Special categories (metabolic causes, total parental nutrition, alagille
syndrome, Byler's disease)
Clinical Features
Infants show conjugated Hyperbilirubenemia, yellow urine, clay colored
stools. Infants with EHBA are often full term and look apparently healthy
except for jaundice.
Those with neonatal hepatitis are small for date and show increased
association with infections and genetic abnormalities.

Answers

117

Differentiation Between EHBA and Neonatal Hepatitis


Children with EHBA should be identified before 2 months to prevent
irreversible damage and for surgical intervention. The combination of a
non-excreting HIDA scan along with GGTP levels more than 50 IU/L
are highly suggestive of EHBA.
Liver biopsy is considered as the most accurate (90 - 95%) diagnostic
test to differentiate between the two.
In biliary atresia, ductular proliferation and fibrosis are seen, whereas
in neonatal hepatitis, there is alternation in lobular architecture, focal
hepatocellular necrosis, bile ductules show little alteration. Giant cell
transformation is found in either conditionand has no diagnostic
specificity.
Currently, peroperative Cholangiography remains the gold standard
to differentiate between EHBA and neonatal hepatitis.
Kasai's portoenterostomy is done for EHBA to re-establish the biliary
flow into gut. Outcome is good if surgery is done before 2 moths of age.
28. Ans is a. Both active and passive immunization soon after birth
(Ref: O.P. Ghai 7th, Pg. 193)
These infants should receive both active and passive immunization at
birth intramuscularly at different sites. Subsequent doses of active
immunization is at 1-2 months and 6 months of age.

More About Hepatitis


Hepatitis Virus
HAV
HBV
HCV
HDV
HEV

Incubations period

15-45 days (mean 30 days)


30-180 days
15-160 days (mean 50 days)
30-180 days
14-60 days (mean 40 days)

Hepatitis B
Perinatal Transmission of HBV
Most important risk factor that determines perinatal transmission ie.
HBeAg. HBSAg. Carrier mother who are HBeAg positive almost
invariably (>90%) transmit Hep B to their offspring, whereas carrier
mothers with anti HBe rarely (10-15%) infect their offspring.

118

Pre-NEET Pediatrics

Most common time of perinatal transmission is at the time of


delivery.
Most of the infected neonates are asymptomatic carrier with
increased chances of chronic hepatitis and hepato cellular carcinoma.

Common Seropatterns of Hep B. Infection


HBSAg

Anti HBS

Anti HBe

HBe Ag

Anti HBe

Interpretation

+
-

IgM
IgM

+
+/-

+/-

IgG

IgG

+
+

IgG
-

+/-

Acute infection
Acute infection,
anti BHC
window
Chronic
infection,
high infectivity
Chronic
infection,
low infectivity
Recovery
Immunization

29. Ans is d. Decreased hematocrit


(Ref: Nelson 19th, Pg. 1147, O.P. Ghai 7th, Pg. 196)

Dengue Fever
Dengue fever is caused by 4 antigenic types of dengue virus (member of
family flaviviridae) spread through the bite of A.aegypti in Asia.
It is characterized by biphasic fever, myalgia, arthralgia and rash.

Answers

119

Clinical Features
Typically, after an incubation period of 4-6 days the patients may develop
abrupt onset high grade fever, facial fushing and headache. There may
be vomiting, pain abdomen associated with tender hepatomegally. Nearly
all patient have some hemorrhagic phenomenon. After subsidence of
fever, some children may show varying degrees of peripheral circulatory
falure
The presence of thrombocytopenia with concurrent hemoconcentration and evidence of plasma leak differentiate DHF from
dengue fever.
Hemo concentration - hematocrit elevated at least 20% above
baseline
Thrombocytopenia - <1Lakh cells/ mm3
Plasma leakage - Presence of pleural effusion on chest x-ray or
hypoalbuminemia
DHF with hypotension is called dengue shock syndrome.

Diagnostic Tests for Dengue Fever


1. Within first 5 days of onset of fever - Viral culture, Dengue NS1
antigen
2. After defervescence or in convalescent phase
Serology test
IgM - ELISA
IgG - Hemagglutination inhibition test

Treatment
In hospital all children without hypotension should be given Ringer's
lactate infusion at rate of 7 ml/kg over 1hr. This is tapered to 3 ml/kg/hr
for 24-48 hrs if hematocrit decreases, and vital parameters improve.
But if hemocrit is rising and vitals dont' show improvement, fluids infusion
rate is increasd. (till 15 ml/kg/hr)
In children with hypotension, Ringer lactate boluses are given. If
hematocrit, and BP fall despite boluses, Blood is transfused.
Majority of patients recover in 24-48 hrs without sequelae.
30. Ans is b. -Thalassemia
(Ref: Nelson 18th, Pg. 2037)
Out of the given 4 types of haemolytic anemia only -Thalassemia is
able to give rise to such a severe presentation at birth. In all other
conditions, the newborn is either normal or has mild jaundice.

120

Pre-NEET Pediatrics

Thalassemia
Thalassemia are genetic disorders in globin chain production (Autosomal
recessive). Adult haemoglobin is a tetramer, composed of 2 globin
chains and 2 globin chains.
Thalassemia is caused by deficient synthesis of chain with
normal chain synthesis.
Thalassemia is cause by deficient chain with normal chain
synthesis
Thalassemia - complete absence of chain
+ Thalassemia - Partial reduction chain
Most common type of genetic abnormality in Thalessemia is
point mutation ie. Nonsense and in Thalessemia is deletion of
globin genes.
In Thalassemia, excess of globin. Chains form Bart's haemoglobin
(4) infetal life and chains for HbH (4) after birth. Barts
haemoglobin in unable to release oxygen to tissues and hence,
causes severe hypoxia and extravascular hemolysis and causes Nonimmune Hydrops in the fetus.

Thalassemia
There are 3 forms of Thalassemia
1. Thalassemia major (Cooley's anemia)
Homozygous for Thalassemia genes.
Severe transfusion dependent
Manifesting at 6-9 months as haemoglobin synthesis switches
from HbF to HbA.
2. Thalassemia Minor ( Thalassemia trait)
Individual has only one copy of Thalassemia genes
(Heterozygous)
Mildest form - patient are usually asymptomatic
3. Thalassemia intermedia
Condition intermediate between major and minor
Individuals may rarely need occasional transfusion

Hematological Findings in Thalassemia


Peripheral Blood Smear

Anisocytosis (Variation in size of RBCs)


Poikilocytosis (variation in shape of RBCs)

Answers

121

Microcytic hypochromic RBCs


Target cells (Hemoglobin collects in centre of RBCs)
Basophilic stippling
Fragmented RBCs
Others: MCV, MCH, MCHC
S. Iron, S. Ferritin, Percentage Saturation, HbA2, HbF

Genetics
HbF
HbA2

T. Major

T. Intermedia

T. Minor (trait)

Homogygous
30-90%
<3.5%

Double heterozygotes
20-100%
<3.5%

Heterozygotes
0-5%
3.6-8%

AIIMS MAY 2012


1. Ans is b. Air bronchogram in chest x-ray
(Ref: Nelson 19th ed. P. 581-590, O.P. Ghai 7th/ P. 143-144)
Respiratory distress syndrome (Hyaline membrane disease)

Incidence
Its incidence is inversely related to gestational age and birth weight. It
occurs in 60-80% of infants <28 wk of gestational age, in 15-30% of
those between 32 and 36 wk, and rarely in those >37 wk.

Etiology and Pathophysiology


Surfactant deficiency (decreased production and secretion) is the primary
cause of RDS. The major constituents of surfactant are dipalmitoyl
phosphatidyl choline (lecithin), phosphatidyl glycerol, apoproteins
(surfactant proteins SPA, SP-B, SP-C, SD-D), and cholesterol. These
phospholipids are synthesized and stored in type II alveolar cells. They
are released into alveoli, where they reduce surface tension and help
prevent collapse of small air spaces at end expiration.
Abnormalities in surfactant protein B and C genes are associated
with severe and often lethal familial respiratory disease called neonatal
alveolar proteinosis.

Diagnosis
Age of onset: Presents usually within minutes of birth.

122

Pre-NEET Pediatrics

Chest X-ray Features of RDS


Bilateral diffuse reticulo granular opacities in the pulmonary parenchyma.
"Prominent air bronchograms" (aerated bronchioles superimposed
on a background of collapsed alveoli)

Conditions Associated with air Bronchogram

Pneumonia
ARDS
HMD (Hyaline membrane disease)
Pulmonary consolidation
Alveolar proteinosis
Sarcoidosis
Lymphoma

Prevention
Administration of antenatal corticosteroids reduces the incidence of
RDS, IVH, NEC, early onset sepsis and developmental delay.

Drug Regimens

Inj. Betamethasone 12 mg/ 1/m every 24 hours 2 doses (preferred)


Inj. Dexamethasone 6 mg 1/m every 12 hours, 4 doses.
Maximum effect is seen between 24 hours - 7days

Treatment
CPAP indication: If oxygen saturation cannot be kept >85% at inspired
oxygen concentrations of 40-70% or greater.

Mechanical Ventilation Indication


1. Persistent apnea
2. Respiratory failure
a. Arterial blood PH <7.2
b. Arterial blood PCO2 60mm Hg
c. Oxygen saturation <85% at oxygen concentrations of 40-70%
and CPAP 5-10 mm H2O.

Surfactant Indications
i.
ii.

Moderate to severe RDS-Rescue treatment


All neonates less than 29 weeks irrespective of presence or absence
of RDS - Prophylactic treatment.

Answers

123

2. Ans is a. Shivering
(Ref: O.P. Ghai 7th, Pg. 115-118, Meharban Singh 7th, Pg. 200-206)

Definitions

Hypothermia: Axillary temperature of baby < 36.50C


Cold stress - 36.0 36.40C
Moderate hypothermia - 32-35.90C
Severe hypothermia - <320C
Hyperthermia: Axillary temperature of baby >37.50C.
Thermoneutral environment: The narrow range of environmental
temperature at which baby can maintain normal baby temperature
with minimal oxygen consumption. This is also called as the zone
of thermal comfort.

Response to Cold in Newborn


Muscular activity (Shivering) is not a significant source of heat production.

124

Pre-NEET Pediatrics

Metabolic Thermogenesis
Non-Shivering thermogenesis, as result of metabolism of brown fat, is
the most important source of heat production in newborn.
The fetal brown fat is laid down mostly during the third trimester of
pregnancy and is located at the nape of the neck, interscapular region,
axillae, groin, around the Kidneys and adrenals. This fat is metabolically
very active in view of a large number of mitochondria and increased
vascularity. Lipolysis of this fat release fatty acids which are locally
consumed for generation of heat.

Reasons why Newborns have increased Susceptibility


to Hypothermia

Large surface area of babies especially head as compared to their


weight.
Limited heat generating mechanisms.
In LBW babies: Decreased subcutaneous and brown fat, more
permeable skin, even larger surface area than term babies and early
exhaustion of metabolic stores like glucose.
Prevention of hypothermia by institution of Warm Chain (10
steps)
1.
2.
3.
4.
5.
6.
7.
8.
9.
10.

Warm delivery room


Warm resuscitation
Immediate drying
Skin to skin contact called kangaroo mother care.
Breast feeding
Bathing postponed
Appropriate clothing
Mother and baby together
Professional alertness
Warm transportation.

Signs and Symptoms of Hypothermia

Acrocyanosis, Cool extremities, delayed capillary refill time (due to


peripheral vaso constriction).
Poor weight gain (chronic hypothermia)
Cardio vascular manifestations: Bradycardia, hypotension, raised
pulmonary pressure with resultant hypoxemia, tacypnea.
CNS depression - Lethargy, poor reflexes, apnea, decreased oral
acceptance.

Answers

125

Abdominal distension, Vomiting, feeding intolerance.


Acidosis, Hypoglycemia, Azotemia, Oiguria, generalized bleeding.

Management
Cold Stress/ Moderate Hypothermia

Skin to skin contact


Regular temperature monitoring
Supportive measures - Feeding, Monitor vitals.

Severe Hypothermia
Incubator/ Radiant warmer
Regular temperature monitoring
Supportive measures: Oxygen, antibiotics (if sepsis suspected), saline
bolus (if hypotension present), Prewarmed IV fluids, Inj. Vit K, Monitor
Vitals.
3. Ans is c. All in children
(Ref: O.P. Ghai 7th, Pg. 580-584, Nelson 19th, Pg. 1732-1737)
The Leukemias are the most common malignant neoplasms in childhood,
accounting for about 31% of all malignancies that occur in children
<15 yrs of age.
ALL (Acute Lymphoblastic Leukemia) accounts for about 77%
of cases of childhood Leukemia.
The Leukemias are a group of malignant diseases in which genetic
abnormalities in a hematopoietic cell give rise to an unregulated clonal
proliferation of cells.

Factors Predisposing to Childhood Leukemia


Genetic Conditions

Down syndrome
Fanconi anemia
Bloom syndrome
Diamond - Blackfan anemia
Schwachman - diamond syndrome
Kostmann syndrome
Neurofibromatosis type I
Ataxia - telongiectasia
Severe combined immune deficiency

126

Pre-NEET Pediatrics

Paroxysmal Noctural hemoglobinuria


Li- Fraumeni syndrome.

Environmental Factors

Ionizing radiation
Alkylating agents
Nitrosourea
Benzene exposure
Epipodophyllotoxin

Diagnosis
Peripheral blood picture that indicates bone marrow failure -anemia,
thrombocytopenia, elevated WBC count with/without leukemic cells
(atypical lymphocytes).
ALL is diagnosed by a bone marrow evaluation that demonstrates
>25% of bone marrow cells as lymphoblasts.
Classification

Based on

1. Morphology
2. Immunopheno type
Leukemia

FAB criteria - L1, L2, L3 categories


Precursor B cell, B cell or T cells

Prognostic Factors
Good Prognosis

Bad Prognosis

1. Children between 1-9 yrs

Children <1yr, >9 yrs

2. Leukocyte count <50,000/mm3

Leukocyte count >50,000/mm3

3. Girls-Sex

Boys - higher relapse rate

4. B-cell Leukemia

T-cell Leukemia

5. Absence of mediastinal widening,


CNS disease

Mediastinal mass or CNS disease


at diagnosis

6. Hyperdiploidy

Hypodiploidy

7. Trisomy 4 and 10

Philadelphia positive t(9;22), t(4;11),


t(8;14) in B cell ALL

8.

1000/mm3 blasts in peripheral


blood following 7 days of
prednisolone treatment and
intrathecal dose of methotrexate

1000/mm3 blasts in PS following 7


days of prednisolone treatment and
intrathecal dose of methotrexate.

Answers

127

Management
Treatment of ALL is divided into 4 stages
i. Induction therapy (to attain remission)
ii. CNS prophylaxis
iii. Intensification (consolidation)
iv. Maintenance therapy
Average duration of treatment in ALL ranges between 2-2 yrs.
1. Induction therapy: Lasts 4-6 weeks. Current induction regime
includes vincristine, prednisolone, L-asparaginase and an
anthracycline.
2. CNS prophylaxis: Essential to eradicate Leukemic cells which
have passed the blood brain barrier. It comprises cranial irradiation
with Intrathecal methotrexate. Other alternative regimen includes
use of methotrexate, hydrocortisone, cytarabine.
3. Intensification therapy: It comprises intensified treatment after
remission induction to tackle problem of drug resistance. Commonly
used agents include methotrexate, L-asparaginase,
Cyclophosphamide, Cytarabine and Epipodophyllotoxin.
4. Maintenance therapy: This is required to prevent relapse. It is
continued for an additional 2-2.5 years after induction remission.
The main agents used include 6-mercaptopurine daily and
methotrexate once a week given orally

Relapse
Early bone marrow relapse before completing maintenance therapy has
the worst prognosis while late relapses after maintenance therapy have
a better prognosis.
Relapse in extramedullary sites especially testes is more favourable.
The treatment of relapse must be more aggressive than the first line
therapy.
* Glioblastoma multiforme (grade IV) is an aggressive form of
astrocytoma with poor prognosis. Pilocytic astrocytoma is with good
prognosis.
* Esophagealcarcinoma and Cholangio carcinoma usually remain
asymptomatic for a very long time. By the time they are detected these
malignancies would have already spread. So the only treatment that
can be offered is palliation.

128

Pre-NEET Pediatrics

4. Ans is a. Factor X deficiency


(Ref: O.P. Ghai 7th, Pg. 318-319, Nelson 19th, Pg. 1695-1708)
To understand these disorders, we should know the normal pathway of
coagulation.

Abnormal PT and aPTT


Oral anticoagulants, Liver
dysfunction
Vit K deficiency, DIC

Abnormal aPTT
Normal PT
Hemophilia A and B,
Von willebrand disease

Abnormal PT
Normal aPTT
Anti-coagulant
therapy (eg:
Warfarin)
F Vll deficiency

Thrombin time: Prolonged thrombin time occurs with reduced


fibrinogen levels (hypofibrogenemia or afibrogenemia). It measures
the final step in clotting cascade, in which fibrinogen is converted
to fibrin.
Bleeding time (BT): Assesses function of platelets and their
interaction with the vascular wall. Prolonged BT may suggest
decreased platelets, qualitative platelet defect, VWD or defect in
vascular wall.

Answers

129

Deficiency of the contact factors (factor XII, Prekallikrein and HMW


Kininogen) causes prolonged PTT but no bleeding symptoms.
Factor V deficiency: Autosomal recessive (parahemophilia)
Mucocutaneous bleeding, hematomas, severe menorrhagia are
most common symptoms
Lab evaluation shows prolonged PTT and PT.
Factor X deficiency: It is rare autosomal disorder of variable
severity.
A reduced factor X level is associated with prolongation of
both PT and PTT.
In some cases of factor X deficiency, PT and aPTT can be
normal also. Patients with factor X deficiency can present with
umbilical stump bleeding at birth similar to factor XIII deficiency.

Factor XIII Deficiency


PT and aPTT are normal. Test used in qualitative evaluation of factor
XIII is evaluation of clot solubility in 5M urea.

Clinical Presentation

Umbilical stump bleeding


Poor wound healing
Infertility, abortion among affected females.

Von Willibrand Disease


Patients with deficiency of Von Willibrand factor rarely present in
neonatal period, because plasma concentration of the factor is high in
neonates. Usually presents with mucosal bleeding, menorrhagia.
VWF helps in platelet aggregation and also serves as the carrier
protein for factor VIII.
VWF is stored in platelet & granules and endothelial cell Weibelpalade bodies.

Lab Findings

Prolonged bleeding time, prolonged aPTT.


Normal platelet count (except 2B disease or platelet type disease)

130

Pre-NEET Pediatrics

Classification
I.
II.
III.

Type
Type
Type

I
II III -

VWF is quantitatively reduced


VWF is qualitatively abnormal
VWF is absent

Treatment
Desmopressin used in type I disease releases VWF from endothelial
cells.

Bernard-Soulier Syndrome

Caused by absence or severe deficiency of VWF receptor (GPIb


complex) on platelet membrane.
This is characterized by thrombocytopenia, giant platelets, prolonged
BT, absent ristocetin induced platelet aggregation but normal
aggregation to other agonists.

Glanzmann Thrombasthenia

Caused by deficiency of platelet fibrinoger receptor IIb/IIIa.


Characterized by prolonged BT, normal platelet count, normal sized
platelets, abnormal aggregation with all agonists except restocetin.

ALL INDIA PG MEDICAL ENTRANCE EXAMINATION


2012
1. Ans is a. Cystic fibrosis
(Ref: 19th, Pg. 1481-1497, O.P. Ghai 7th, Pg. 274-75)
Cystic fibrosis is an inherited multisystem disorder with autosomal
recessive transmission.
The cystic fibrosis gene is on Chromosome 7 (7q13) and there are
more than 1400 mutations at CF locus, of which Delta F508 (which
devotes a single deletion at 508 position of protein) is the commonest.
The mutation affects the gene's protein product, cystic fibrosis
transmembrane regulator (CFTR) which functions as a chloride channel
and is expressed largely in epithelial cells of airways, the gastrointestinal
tract, the sweat glands and the genito urinary system. Dysfunction of
CFTR, the primary defect, leads to a wide and variable array of clinical
manifestations and complications. For eg. Exocrine pancreatic
insufficiency, severe chronic lung disease in children, pan sinusitis, nasal
polyposis, Cholelithiasis and insulin dependent hyperglycemia.

Answers

131

Pathogenesis
CF epithelial cells especially the respiratory tract are unable to secrete
chloride ions which leads to excessive sodium and water reabsorption.
This leads to viscous desiccated secretions that cause airway obstruction.
In sweat glands, the main function to absorb rather than secrete chloride
not done, consequently sodium levels are elevated in sweat.

Diagnosis

Presence of typical clinical features ( Respiratory, GIT or Genito


urinary) or

History of CF in sibling or

Positive newborn screening test plus

Laboratory evidence for CFTR dysfunction:

2 elevated sweat chloride concentrations obtained on separate days


or

Identification of 2 CF mutations or

An abnormal nasal potential difference measurement.


More than 60 meq/L of chloride in sweat is diagnostic of CF

Management
1. Respiratory: Airway clearance techniques, antibiotics and antiinflammatory agents.
2. Nutritional: Increasing caloric intake, supplement fat soluble
vitamins (A, D, E in twice the recommended doses), replace
pancreatic enzymes.
2. Ans is a. Autism
(Ref: Nelson 19th, Pg. 101)

132

Pre-NEET Pediatrics

The pervasive developmental disorders (PDD) are


disturbance of brain development with genetic basis
PDD includes
Autism

Asperger's Rett's syndrome Childhood


PDD - NOS
syndrome
disintegrative (not otherwise
disorder
specified)

Delayed
and
disorde
redcommunication
Atypical
social
interaction
Restricted
range of
interests
Onset
before
3 yr
of age

Similar to
autism
except
language
skills
relatively
intact.
Usually
not
cognitively
delayed.

Almost, always
affects girls
in skills
between 6-18
months of age.

Clinically
significant
regression
in skills
bowel
and bladder
control,
play motor
skills) before
10 yrs
of age

Features of 1 of
the other
autism
spectrum
disorders,
but insufficient
for a specific
diagnosis

Autistic Disorder
Diagnosis
DSM IV criteria for Autistic disorder
(A) A total of six (or more) items from (1), (2), and (3) with at least 2
from (1) and one each from (2) and (3)
(1) Qualitative impairment in social interaction, as manifested by least
2 of the following:
a. Marked impairment in the use of non-verbal behaviours like eye
eye gaze, facial expression, body postures, and gestures to regulate
social interaction.
b. Failure to develop peer relationships approximate to developmental
level.

Answers

133

c.

A lack of spontaneous seeking to share enjoyment, interests, or


achievements with other people
d. Lack of social or emotional reciprocity.
(2) Qualitative impairments in communication as manifested by at least
one of the following:
a. Delay in, or total lack of, the development of spoken language
b. In individuals with adequate speech, marked impairment in ability
to initiate or sustain a conversation with others
c. Stereo typed and repetitive use of language or idiosyncratic language
d. Lack of varied, spontaneous make believe play or social initative
lay appropriate to developmental level.
(3) Restricted repetitive and Stereo typed patterns of behaviour, and
activities, as manifested by lest one of the following:
a. Encompassing preoccupation with one or more stereo typed and
restricted patterns of interest that is abnormal either in intensity or
focus
b. Apparently inflexible adherence to specific non functional routines
or rituals
c. Stereo typed and repetitive motor manners
d. Persistent preoccupation with parts of objects
(B). Delays or abnormal functioning in at least one of following areas,
with onset prior to 3 yrs.
1. Social interaction
2. Language as used in social communication or
3. Symbolic or imaginative play.
(C) The disturbance is not better accounted for by Retts disorder or
childhood disintegrative disorder.
3. Ans is b. Increased EFA (essential fatty acids)
(Ref. Avory 8th, Pg. 1061, Cloherty 5th, Pg. 115)
The Caloric value of 20% lipid emulsion is 2 kcal/mL. The use of 20%
emulsion is preferred over 10% because the higher ratio of phospholipids
to triglyceride in the 10% emulsion interferes with triglyceride clearance.
Twenty percent emulsion also provide a more concentrated source of
calories. For these reason, 20% lipid emulsions are preferred.

134

Pre-NEET Pediatrics

Parenteral Nutrition (PN)


Indications
1. Infants with b.wt 1500 gm
2. Infants with b.wt of 1501 - 1800 gm for whom significant enteral
nutrition is not expected for >3 days
3. Infants with b.wt >1800 gm for whom significant enteral intake is
not expected for >5 days. Goal of PN is to provide aminoacids to
prevent negative energy and nitrogen balance. Goal therafter includes
promotion of appropriate weight gain and growth, while awating
the attainment of adequate enteral intake.
i. Carbohydrate - Dextrose is the carbohydrate source in I/V solutions.
Calorie value of dextrose is 3.4 kcal/gm. Dextrose concentration
via peripheral line is limited to 12.5% and upto 25% dextrose
for central venous infusions.
Infusion rates above 11-14 mg/kg/min may exceed infants
oxidative capacity and are generally not recommended
ii.

Protein: Crystalline amino acid solutions provide the nitrogen source


in PN.
The calorie value of amino acids is 4 kcal/gm. Infusion rates
are generally initiated at 1gm/kg/dl to a target of 3.5 gm/kg/dl
for neonates >1500 gm at birth.
iii. Lipid: Soybean oil or a combination of soybean and safflower oil
provide for I/V fat emulsions.
Infusion rates are generally initiated at 0.5 - 1gm/kg/dl within
24 - 48 hours of life to a target of 3.0 gm/kg/day.
Use of 20% emulsions is preferred (explained above).
* Electrolytes, Vitamins and Minerals are added as per
requirement.

Complications of PN
1. Cholestasis: as a result of hepatic dysfunction (Reduced bile flow
and bile salt formation).
2. Metabolic bone disease
3. Azotemia, hyperammonemia, hyperchloremic metabolic acidosis
seen especially with amino acid intakes >4 gm/kg/dl.
4. Sepsis: associated with decreased Lipoprotein lipase activity. During
sepsis, lipid infusion is limited to 2 gm/kg/dl if triglyceride> 150 mg/
dL

Answers

135

5. Hyperlipidemia/hypertriglyceridemia
6. Indirect hyperbilirubinemis - due to displacement of bilirubin from
albumin binding sites by free fatty acids.
4. Ans is d. Sacrococcygeal teratoma
(Ref: Avery 8th, Pg. 1454)

Sacrococcygeal Teratoma
Sacrococcygeal teratoma is the most common solid tumor in newborn,
although it is rarely malignant.
Females are affected 2-4 times more frequently than males.
In most cases, the tumor manifests as a mass protruding between the
coccyx and rectum.
Most benign teratomas produce no functional difficulties.
Bowel/bladder dysfunction, painful defection and vascular/ lymphatic
obstruction suggest that the lesion is malignant.

Treatment
It is primarily surgical if age adjusted AFP and HCG levels are normal.
Chemotherapy (using Cisplatin and/ Bleomycin) + Surgery are used for
malignant sacrococcygeal tumors.

Other Infant Tumors

The most common intrarenal neoplasm manifesting at birth is


congenital mesoblastic nephroms followed by Wilm's tumor.
Wilms tumor or nephroblastoma, is the most common intra
abdominal tumor of childhood but is relatively rare in the neonatal
period.

Congenital Leukemia
Rarely occurs during the 1st month of life. Most of neonatal cases arise
from the myeloid lineage. Trisomy 21 is particularly associated with
AML in newborns.
The chemotherapy regimens used in infants with myeloid leukemia
are identical to those used in older children.

Transient Myelo-proliferative Disorder (TMD)

Associated with Down syndrome


Down's syndrome and TMD have a 30% chance of developing
leukemia latter in life.

136

Pre-NEET Pediatrics

TMD is a clonal disorder manifested by Hepatosplenomegally and


circulating myeloblasts with/without anemia thrombocytopenia
Cytogenetic analysis often is normal.
In majority of cases, spontaneous resolution occurs.

CNS Tumours
CNS malignancies are rare in newborn. Most of brain tumors are
supratentorial half of them are gliomas. Infants with primitive
neuroectodermal tumors and ependymoma have poor prognosis.

Hepatoblastoma
Uncommon in infants and children. The most common malignant
neoplasm involving the liver in infancy is "Metastatic neuroblastoma".
The most common benign hepatic neoplasm in neonate are
mesenchymal hamartomas and hemangiomas.
Hereditary conditions with associated tumors
1. Ataxia - telangiectasia - Leukemia, lymphoma
2. Beckwith - Wiedemann syndrome - Wilm's tumor, hepatoblastoma,
Rhabdomyosarcoma
3. Bloom syndrome - Leukemia
4. Denys - Drash syndrome - Familial Wilms tumor
5. Fanconi anemia - Leukemia
6. Klinefelter syndrome - Teratoma, leukemia, Breast cancers
7. Li-fraumeni syndrome - Sarcoma, CNS, Breast tumors
8. Neurofibroatosis - Glioma, leukemia, sarcoma
9. Trisomy 18 - Wilms tumor
10. Von hippel landau syndrome - Non Hodgkin lymphoma
11. WAGR syndrome - Wilms tumor
12. X-linked lymphoproliferative disorder - EBV lymphomas

5. Ans is a. Normal saline


(Ref: O.P. Ghai 7th, Pg. 417, Nelson 19th, Pg. 1998)

Neural Tube Defects (NTDs)


NTDs account for the largest proportion off congenital anomalies of the
CNS and result from failure of neural tube to close spontaneously
between the 3rd and 4th wk of in utero development.
Many factors, including hyperthermia, valproic acid, malnutrition,
chemicals, diabetes (maternal) and genetic determinants (mutations in

Answers

137

folate responsive or folate, dependent enzyme pathways) can affect


normal development of the CNS.
The major NTDs include spina bifida occulta, meningocele,
myelomeningocele, encephalocele, anencephaly, caudal regression
syndrome, dermal sinus, syringomyelia, diastematomyelia, and lipoma
involving the conus medullaris.
Prenatal screening of maternal serum for AFP in the 16th - 18th wk
of gestation identifies foetuses with NIDs in Utero.
Spina bifida occulta is a common anomaly consisting of a midline
defect of vertebral bodies without protrusion of the spinal or meninges.
Most patients are asymptomatic.
Meningocele is formed when the meninges herniate through a defect
in the posterior vertebral arches or the anterior sacrum.
It is often associated with Arnold-Chiari malformation and
hydrocephalus is either present at birth or appears later.
Myelomeningocele is formed when spinal cord and meninges herniate
through a defect in the posterior vertebral arches. It represents the most
severe form of dysraphsim.

Prevention

Dose for primary prevention is 0.4 mg folate per day.


A mother who has previously delivered a child with NTD should
receive 4 mg per day of folic acid in subsequent pregnancies.
Duration of supplementation is 2 months before and 3 months
after conception.
Hydrocephalus in association with a type II Chiari malformation
develops in at least 80% of patients with myelomeningocele.
Generally, the lower the deformity the less likely is the risk of
hydrocephalus.

Treatment
The defects which are covered by skin do not need urgent treatment but
others should be closed soon after birth because they are likely to get
infected.
The Sac should be kept covered with a Sterile Saline - moistened gauze
sponge to prevent infection and fluid loss. After repair of a
myelomeningocele, most infants require a shunting procedure for
hydrocephalus.

138

Pre-NEET Pediatrics

6. Ans is b. Small ASD


(Ref: Nelson 19th, Pg. 1551-1553)

Atrial Septal Defect (ASD)


ASD can occur in any portion of the atrial septum
i. Ostium Secundum: defect in region of fossa ovalis
ii. Ostium primum: defect is situated in the lower portion of atrial
septum
iii. Sinus Venosus ASD: defect in upper part of atrial septum in close
relation to entry of the superior vena cava.
Majority of the cases are sporadic. Autosomal dominant inheritance
occurs as part of Holt-Oram syndrome (hypoplastic/absent radii 1st
degree heart block, ASD) or in families with secundum ASD heart block.
An isolated valve: incompetent patent foramen ovale (PFO) is
usually of no hemodynamic significance and is not considered
an ASD.
Device closure of PFO is considered in young adults with history
of thromboembolic stroke as it may be a risk for right to left
(paradoxical)systemic embolization.

Clinical Manifestations
A child with an ASD is most often asymptomatic However, a history of
exercise intolerance, easy fatigability and recurrent pneumonias
accentuated may be obtained with large left to right shunts.
Auscultatory signs include a normal or accentuated 1st Heart sound;
wide, fixed splitting of the 2nd sound a pulmonary systolic ejection
murmur (due to increased flow of blood across right ventricular outflow
tract into pulmonary artery), a short mid diastolic murmur produced by
increased volume of blood flow across tricuspid valve.
The chest X-ray shows varying degree of enlargement of the right
ventricle and atrium, depending on the size of the shunt. The pulmonary
artery is enlarged and pulmonary vascularity is increased.

Treatment
Surgical closure usually after 1st year and before entry into school.

Answers

139

Indications
i.
ii.

All symptomatic patients


Asymptomatic patients with pulmonary: Systemic blood food (Qp,
Qs ratio) of at least 2:1 or those with right ventricular enlargement.

Complications
Infective endocarditis is extremely rare and antibiotic prophylaxis is not
recommended.
* Partial anomalous pulmonary venous return (one or several
pulmonary veins returning anomalously to superior or inferior vena
cava, right atrium or the coronary (sinus)may be associated with
ASD (mostly of sinus venosus type)
* Scimitar syndrome: an anomalous pulmonary vein draining into
the inferior vena cava visible on CXR as a crescentic shadow of
vascular density along right border of the cardiac silhouette.
7. Ans is a. Can be seen after ventouse delivery
(Ref: Meharban Singh 7th, Pg. 254-272, Nelson 19th, Pg. 602-613)
Jaundice is observed during the 1st wk of life in approximately 60% of
term infants and 80% of preterm infants.
Although bilirubin may have a physiologic role as an antioxidant,
elevations of indirect, unconjugated bilirubin are potentially neurotoxic.
The conjugated form is not neurotoxic, direct hyperbilirubinemia
indicates a potentially serious hepatic disorders or a systemic illness.

Physiological Jaundice
In term babies, it appears after 36 hrs of age. Maximum intensity is
seen on the 4th day, S. Bilirubin does not exceed 15 mg/dL and jaundice
disappears by 10 days of life.
In preterm babies, the maximum intensity of jaundice is reached
on the 5th or 6th day, S. Bilirubin may go upto 15 mg/dL and it may
persist upto 14 days.
The etiology of physiologic jaundice appears to be over production
due to polycythemia, poor hepatic uptake, conjugation and excretion
of bilirubin.
Pathological jaundice: when jaundice in the newborn does not conform
to criteria described for physiological jaundice, it is designated as
pathological.
Cephalohematoma or significant bruising attributable to ventouse
delivery is a major risk factor for development of pathological

140

Pre-NEET Pediatrics

hyperbilirubenemia as it increases the load of bilirubin to be metabolized


by the liver.

Jaundice Associated with Breast Feeding


1. Breast feeding jaundice: is due to insufficient Breast feeding that
leads to increased enterohepatic circulation. It is of early onset,
which occurs in the 1st week of life. Hyperbilirubinemia (>12 mg/
dL) develops in 13% of breast fed infants in 1st wk of life.
2. Breast milk jaundice: develops in an estimates 2% of breast fed
term infants after the 7th day of life with maximal concentrations
as high as 10-30 mg/dL reached during the 2nd 3rd wk.
if breast feeding is continued, the levels will stay elevated and
then fall slowly at 2 wks of age, returning to normal by 4 to 12
wks of age.
if breast feeding is stopped, the bilirubin level will fall rapidly in
48 hrs.
If nursing is then resumed, the bilirubin may rise 2-4 mg/dL but
usually will not reach the previous level.
Mothers with infants who have breast milk jaundice syndrome have a
recurrence rate of 70% in future pregnancies.
The etiology of breast milk jaundice may be attributed to the
presence of glucuronidase in some breast milk.

Measures to Reduce S Bilirubin


I.

Phototherapy: Bilirubin absorbs light at 425-475 mm and is converted


into:
a. Water soluble form of bilirubin by photo-oxidation
b. Water soluble E-isomers (25% total S Bilirubin) by
photoisomerization
c. Most efficient structural photoisomers called lumirubin which
are readily excreted in bile, feces and urine.
The narrow spectral blue light is most effective for phototherapy.

Bronze Baby Syndrome


Infants with Parenchymal liver disease with biliary obstruction may
develop bronze discolouration of skin due to excessive accumulation of
lumirubin which is polymerized to bilifuscin on exposure to phototherapy.

Answers

141

Exchange Blood Transfusion

It is most effective and reliable method to reduce bilirubin levels.


Early indications for exchange blood transfusion in infants with Rhhemolytic disease of the newborn.
a. Cord haemoglobin of <10 g/dL or hematocrit <30%
b. Cord bilirubin >5 mg/dL
c. Unconjugated S.bilirubin 10 mg/dL within 24hrs or rate of
rise of >0.5 mg/dL/hr.

8. Ans is b. 10% Dextrose I.V.

Hypoglycemia in Newborn
Definition is controversial, however most workers agree that blood glucose
level of less than 40 mg/100mL (irrespective of period of gestation), if
associated with symptoms of hypoglycaemia or if confirmed on repeat
analysis in asymptomatic babies, is indicative of hypoglycaemia (1.0
mmd/L or glucose is equivalent to 18.02 mg/dL).

Classification of Neonatal Hypoglycemia


1. Fetal/ Neonatal hyperinsulinism
Maternal diabetes mellitus
Erythroblastosis fetalis
Wiedemann - Beckwith syndrome
Transposition of great vessels
Insulin producing tumors (nesidio blastosis, islet cell adenoma)
Maternal therapy with beta- sympathomimetics to colytic agents
(Salbutamol, Terbutaline)
2. Decreased glycogen stores and/or increased utilization of
glucose
Intrauterine growth retardation
Prematurity
Birth asphyxia
Hypothermia
Polycythemia
Septicemia

142

Pre-NEET Pediatrics

3. Miscellaneous Conditions
Inborn errors of metabolism (glycogen storage disease,
galactosemia, fructose intolerance, CAH, cretinism)
Congenital Hypo pituitarism
Maternal therapy with Blockers (propanolol), Chlorpropamide
Exchange transfusion with heparinised blood/ACD/CPD blood.

Management

If baby requires >12 mg/kg/min glucose infusion or hypoglycaemia


not resolved by day 7, start drugs like steroids, glucagon, diazoxide
and investigate for resistant hypoglycaemia. (S. Cortisol, S. Insulin
levels, CT scan pancreas, Screen for inborn errors of metabolism)

Answers

143

9. Ans is a. S. Uric acid


(Ref: Nelson 18th, Pg. 629-631)

Lesch-Nyhan Disease (LND)


This is a rare X-linked disorder of purine metabolism that results from
HPRT (hypoxanthine guanine PRPP transferase) deficiency.
This enzyme is normally present in each cell in the body, but its
highest concentration is in the brain, especially in the basal ganglia.
Clinical manifestations include hyperuricemia, intellectual disability,
dystonic movement disorder including Choreoathetosis, Spasticity,
dysarthric speech and compulsive self-biting, usually beginning with the
teeth eruption.

Genetics
The HPRT gene has been localized to the long arm of the XChromosome. The disorder appears in males, occurrence in females is
extremely rare and ascribed to non-random inactivation of the normal
X Chromosome.
The mechanism whereby HPRT leads to the neurologic and
behaviour symptoms is unknown but they are not caused by
hyperuricemia or excess hypoxanthine as patients with partial HPRT
deficiency do no self injure themselves.

Diagnosis
The presence of dystonia along with self-mutilation of mouth and fingers
suggests Lesch-Nyhan disease. Serum levels of uric acid >4-5 mg uric
acid/dL and a urine uric acid, creatinine ratio of 3:4 or more are highly
suggestive of HPRT deficiency.
The definitive diagnosis requires an analysis of the HPRT enzyme.

Treatment
Medical management of this disorder focuses on the prevention of renal
failure by pharmacologic treatment of hyperuricemia with high fluid
intake along with alkali and allopurinol.
Bone marrow transplantation (BMT) has been carried out in several
patients, based on the possibility that the CNS damage is produced by
a circulating metabolic toxin.

144

Pre-NEET Pediatrics

10. Ans is a. 15 months


(Ref: Nelson 19th, Pg. 31)
Receptive language Precedes expressive language. By the time infants
speak their first words around 12 months of age, they already respond
appropriately to several simple statements, such as "no", "bye-bye", "giveme".
By 15 months, the average child points to major body parts and
uses 4-6 words spontaneously and correctly. Toddlers also enjoy
polysyllabic jargoning but do not seem upset that no one understands.
Most communication of wants and ideas continues to be nonverbal.
11. Ans is a. Shivering
(Ref: Question AIIMS May 2012)
12. Ans is a. Deleted 21
(Ref: O.P. Ghai 7th, Pg. 613, Nelson 19th, Pg. 401-403)

Down Syndrome
This is the most common disorder occurring with a frequency of 1:800
- 1:1000 newborns.
Chromosome number 21 is present in triplicate, the origin of extra
chromosome 21 is mostly from the mother (97%). The risk in newborn
is 1:1550 if maternal age is between 15 and 29 years, 1:800 at 30-34
yrs, 1:270 at 35-39 yrs, 1:100 at 40-44 yrs and 1:50 after 45 yrs.

Cytogenetics
In approximately 95% of the cases of Down syndrome there are 3
copies of Chromosome 21.
Approximately 1% of Trisomy 21 are mosaics (with some cells
having 46 Chromosomes).
4% have translocation that involves Chromosome 21. Majority of
translocations are fusions at centromere between Chromosomes
13,14,15,21 and 22 called as Robert Sonian translocation. Translocation
(21,21) Carriers have a 100% recurrence risk and other Robert Sonian
translocations have a 5-7% recurrence risk when transmitted by females.
It is not possible to distinguish the phenotypes of persons with full
trisomy 21 and those with a translocation while patients who are mosaics
tend to have a milder phenotype.

Answers

145

Prenatal Diagnosis
Initial Screening - PAPP-A, free hCG in first trimester
Second trimester (Quad testing) - S.FP, hCG, Unconjugated estriol
and inhibin A.
Inhibin A, hCG (Both with 'h'spelling have high levels in Down's
syndrome. Rest all are decreased.
(i.e. PAPP-A, S.FP, Unconjugated estriol)

Ultrasound Findings

1st trimester: Nuchal translucency and nasal bone


2nd trimester: Increased nuchal fold thickness (measured over
occiput), short femur and humerus length and duodenal atresia.

AIIMS NOVEMBER 2011


1. Ans. is c. Obstructive TAPVC
(Ref: Park- Paediatric cardiology 5th, Pg. 164-165, Nelson 18th, Pg.
1856, 1922-1924)
Functional closure of the ductus arteriosus occurs within 10-15 hours
after birth by constriction of the medial smooth muscle in the ductus.
Anatomic closure is completed by 2-3 wks of age by permanent changes
in the endothelium and subintimal layers of the ductus. During fetal
life, patency of the ductus appears to be maintained by the combined
relaxant effects of low oxygen tension and endogenously produced
Prostglandens esp PGE2. However, ductus of a premature infant is less
responsive to oxygen, ever though its musculature is developed.
PGE2 maintains the patency of ductus arteriosus so it is useful in
heart lesions where the patency of ductus is essential to maintain the
blood flow in either Aorta or Pulmonary artery (since it is the connection
of two).
If a cyanotic congenital heart defect or a ductus dependent cardiac
defect (e.g: Pulmonary atresia with or without VSD, Trucuspid atresia,
Hypoplastic left heart syndrome, interrupted aortic arch. Severe CoA,
Transposition of great Vessels) is suspected or confirmed, a PGE,
intravenous infusion should be started.

TAPVR (Total Anomalous Pulmonary Venous Return)


In TAPVR, the heart has no direct pulmonary venous connection into
the left atrium.

146

Pre-NEET Pediatrics

Types of TAPVR: Pulmonary veins may drain into


1. Superior Vena Cava - Supracardiac (50%)
2. Coronary Sinus, Right atrium - Cardiac (25%)
3. Inferior Vena Cava - Infracardiac (20%)
4. Mixed (5%)
The manifestations of TAPVR depend on the presence or absence of
obstruction of the venous Channels.
If pulmonary venous return is obstructed, severe pulmonary
congestion and pulmonary hypertension develop.
Obstructed TAPVR is a Pediatric cardiac surgical emergency
because prostaglandin therapy is usually not effective.
Chest X-ray of supracardiac TAPVR shows a "snowman"
appearance. This is however not useful in early infancy because of the
thymus.
2. Ans is a. VDRL for mother and baby
(Ref: Nelson 19th, Pg. 1016-1022)
Congenital syphilis results from transplancental transmission of
spirochetes. Transmission can occur at any stage of pregnancy. Untreated
syphilis during pregnancy has a vertical transmission rate approaching
100% Fetal or Perinatal death occurs in 40% of affected infants.
In congenital syphilis, manifestations have been divided into early
and late stages.
I. Early signs: appear during first 2 yr of life. They result from
transplacental spirochetemia and are analogous to secondary stage
of acquired syphilis. Symptoms/ Signs include:
a. Hepato splenomegally
b. Lymphadenopathy
c. Coombs negative haemolytic anemia
d. Thrombocytopenia
e. Osteochondritis, periostitis
f. Mucocutaneous rash - Mucous patches
g. Persistent rhinitis (snuffles)
h. Condylomatous lesions
i. X-ray features: Wimberger's lines (metaphyseal demineralization
of medial aspect of proximal tibia), osteochondritis (painful,
resulting in Parrot's Pseudo paralysis)
j. CNS abnormalities, failure to thrive, chorioretinitis, Nephritis
and Nephritic syndrome.

Answers

147

II. Late Manifestation: appear gradually after 2 yrs of age.


These result primarily from chronic granulomatous inflammation of
bone, teeth and CNS. These are:
Olympian brow, Higoumenaki sign (Unilateral or bilateral thickening of
sternoclavicular third of the clavicle), Saber Shin's (anterior bowing of
tibial midportion), Hutchinson teeth (Peg shaped upper central incisors),
Saddle nose (Depression of nasal root), Hutchinson triad (Hutchinson
teeth, interstitial keratitis, and 8th nerve deafness), Clutton joints
(unilateral or bilateral painless joint swelling).

Diagnosis
1. VDRL (Venereal disease research laboratory) tests are sensitive nontreponemal tests that detect antibodies against phospholipid antigens
on the treponema surface that cross react within the mammalian
cardiolipin - lecithin - cholesterol antigens. Titres increase with active
disease and decline with adequate treatment.
False positive VDRL: Infectious mononucleosis, connective tissue
disease, pregnancy.
False negative VDRL: Prozone effect (excess antibody), early
primary syphilis, latent syphilis, late congenital syphilis).
2. Treponemal antibody tests: TPHA (T. Pallidum hemagglutination
assay), FTA-abs (Fluorescent treponemal antibody absorption) test.
These tests become positive soon after initial infection and usually
remain positive for life, even with adequate therapy.
They are useful for diagnosis of a 1st episode of syphilis and for
distinguishing false positive result of VDRL but cannot accurately
identify length of time of infection, response to therapy or
reinfection.

Treatment
Congenital syphilis: Aqueous crystalline penicillin I/V for 10 days.
3. Ans is c. Vein of galen malformation
(Ref: Nelson 18th, Pg. 1988, 2511)
Arterio venous malformations result from failure of normal capillary
bed development between arteries and veins during embryogenesis. AV
malformations produce abnormal shunting of blood, causing vessel
expansion, a space occupying effect or rupture and intracerebral bleeding.
The vein of galen is an arterio venous malformation that is located
under cerebral hemisphere and drains the anterior and central regions
of the brain into the sinuses of posterior cerebral fossa.

148

Pre-NEET Pediatrics

Auscultation of skull is positive for a high pitched bruit. USG


demonstrate a hypoechoic mass located posterior to the third ventricle
with pulsatile flow that helps in differentiating it from other midline
cystic lesions.
Vein of galen malformation can cause high output congestive heart
failure secondary to shunting of large volumes of blood or progressive
hydrocephalus secondary to obstruction of the CSF pathways.
Vein of galen malformations are difficult to treat (surgery or occlusive
therapy) and are associated with a poor prognosis.
4. Ans is a. Kartagener's syndrome
(Ref: Nelson 18th, Pg. 1817-1819)
Primary ciliary dyskinesis (PCD) comprises those respiratory disorders
having malfunction of airway cilia that lead to repeated and chronic
lung and sinus infections.
About 50% of patients with PCD have Kartagener syndrome:
Situs inversus
Chronic sinusitis and otitis
Airway disease leading to bronchectasis
Approximately 25% patients with situs inversus have PCD, the presence
of situs inversus does not establish the presence of PCD.
In classical PCD, there is absence of both inner and outer dynein
arms. Partial dynein arm defects or defects in central doublets are
generally not associated with situs inversus.

Genetics
Most PCD cases are autosomal recessive though there are reported
cases of Autosomal dominant and X-linked modalities. Austosomal
dominant and x-linked modalities.
5. Ans is b. Primary hypothyroidism
(Ref: Nelson 19th, Pg. 1895-1903)
Both choices A and C are similar - Both cause hyperthyroidism with a
high T4 level.
Choice D, TSH resistance is a very rare condition and many reported
cases showed normal T4 level.

Hypothyroidism

Most cases of congenital hypothyroidism are not hereditary and


result from thyroid dysgenesis (accouting for 80-85% cases). Most
common dysgenesis is an ectopic gland. Thyroid peroxidase defects

Answers

149

of organification and coupling are the most common of the T4


synthetic defects.
Pendred syndrome: an autosomal recessive disorder comprising
sensineural deafness and goiter, also have impaired iodide
organification and a positive perchlorate discharge. It is due to a
mutation in the chloride-iodide transport protein common to the
thyroid gland and the Cochlea.
The most common cause of acquired hypothyroidism is chronic
lymphocytic (Hashi Moto's) Thyroiditis.
Maternal medications causing hypothyroidism are iodides,
amiodarone, propylthiouracil, methimazole, Radioiodine.
Kocher-Debre Semelaign syndrome: Generalized muscular
pseudohypertrophy esp. Boys in some affected patients with
hypothyroidism of longer duration and severity.

Laboratory Findings

Newborn screening is done by heel prick between 2 and 5 days of


life (Raised TSH, low T4 level).
Retardation of bone age at birth can be shown in about 60% of
congenital hypothyroidism infants. The distal femoral epiphyses,
normally present at birth is after absent.
The epiphysis often have multiple foci of ossification (epiphyseal
dysgenesis), deformity ("beaking") of the 12th thoracic or 1st or 2nd
lumbar vertebra is common.
Treatment: Levothyroxine is the treatment of choice
Neonates
- Initial starting dose is 10-15 g/kg/d
Children
- 4 g/kg/24 hr
Adults
- 2 g/kg/24 hr

6. Ans is b. Sickle cell anemia


(Ref: Nelson 19th, Pg. 1662)
G6Pd and hereditary spherocytosis can cause episodic anemia with
jaundice since birth.
Our choice here is sickle cell anemia as it never manifests in first 6
month of life.
Beta globin disorders such as sickle cell disease or beta-thalassemia
major do not become apparent clinically until several months of
age, when the switch from Haemoglobin F to haemoglobin A
synthesis reveals the defect.
Moreover sickle cell anemia usually presents with anemia, fever
(infections), acute splenic sequestration, dactylitis , stroke etc.

150

Pre-NEET Pediatrics

While Paroxysmal nocturnal hemoglobinuria is a rare disorder of


abnormal marrow stem cells that affects each blood cell lineage.
It is an acquired disorder that is characterized by a defect in proteins
of the cell membrane (including decay accelerating factor, the C8
binding protein) that renders RBCs and other cells susceptible to
damage by plasma complement. The underlying defect involves
the glycolipid anchor that maintains these protective proteins on
the cell surface.
PIGA gene encodes for phosphatidyl glycan protein which is essential
for synthesis of GPI (glycolipid anchor).

Clinical Manifestations
1. Nocturnal and morning hemoglobinuria is a classic finding in adults.
However, chronic hemolysis (intravascular) is more common in PNH,
despite its name.
2. Thrombocytopenia and leukopenia are after seen.
3. Thrombosis and thromboembolic phenomena are sometimes seen
(due to altered glycoproteins on the platelet surface and resultant
platelet activation).
4. Hypoplastic or a plastic pancytopenia may follow/precede the onset
of PNH
5. PNH rarely progresses to acute myelogenous leukemia

Lab Findings
1. Acidified serum hemolysis (Ham) test or sucrose lysis test. These
tests activate the alternative and classic pathways of complement
lysis, respectively.
2. Flow cytometry is the diagnostic test of choice for PNH. It uses anti
CD59 for RBCs and anti CD55, anti CD59 for granulocytes.

Treatment
1.
2.
3.
4.
5.

Splenectomy is not indicated.


Glucocorticoids for acute hemolytic episodes
Prolonged anticoagulant therapy
Bone marrow transplantation
Eculizumab, monoclonal antibody against complement proteins
C5, stabilized haemoglobin levels and decrease the rate of hemolysis.

Previous Years Questions of

DNB
QUESTIONS
PAEDIATRICS 2010
Q 1. Earliest indication of sexual maturation in a girl is:
A. Menarche
B. Pubarche
C. Thelarche
D. Maturation of breasts
Ans. is C. Thelarche
(Ref: O.P. Ghai 7th, Pg. 498)
Q 2. Epiphyseal dysgenesis is a pathognomonic feature of:
A. Hypoparathyroidism
B. Hyperparathyroidism
C. Hypothyroidism
D. Hyperthyroidism
Ans. is C. Hypothyroidism
(Ref: Nelson 19th, Pg. 1899)
Q 3. Commonest cause of stridor in a new born is:
A. Laryngomalacia
B. Foreign body
C. Meconium aspiration
D. Recurrent laryngeal nerve palsy due to birth
Ans. is A. Laryngomalacia
(Ref: O.P. Ghai 7th, Pg. 340)
Q 4. Precocious puberty is seen in:
A. Hyperthyroidism
B. Addisons disease

152

Pre-NEET Pediatrics

C. McCune Albright syndrome


D. Neuroblastoma
Ans. is C. McCune Albright syndrome
(Ref: Nelson 19th, Pg. 1892)
Q 5. The commonest cause of death in diphtheric child is:
A. IIIrd nerve palsy
C. Tonsilitis
B. Myocarditis
D. Septicemia
Ans. is B. Myocarditis
(Ref: IAP Textbook of Pediatrics 4th, Pg. 364)
Q 6. Berger nephropathy is due to mesangiai deposition of:
A. Fibrin and C3
B. IgD and C3
C. IgE and C3
D. IgA and C3
Ans. is D. IgA and C3
(Ref: O.P. Ghai 7th, Pg. 446)
Q 7. Pawn ball megakaryocytes are characteristic of:
A. Myelodysplastic syndrome
B. Idiopathic thrombocytopenic purpura
C. Thrombotic thrombocytopenic purpura
D. Chloramphenicol toxicity
Ans. is A. Myelodysplastic syndrome
Q 8. Commonest cause of heart failure in infancy is:
A. Myocarditis
B. Rheumatic fever
C. Cardiomyopathy
D. Congenital heart disease
Ans. is D. Congenital heart disease
(Ref: O.P. Ghai 7th, Pg. 372)
Q 9. A 3.5 kg baby born to diabetic mother develops seizures
at 16 hours.The most likely cause is:
A. Hypoglycemia
B. Hypoxia/Respiratory distress syndrome

Previous Years Questions of DNB

153

C. Hypomagnesiumia
D. Hypocalcemia
Ans. is A. Hypoglycemia
(Ref: O.P. Ghai 7th, Pg. 156)
Q 10. Which of the following is not used a in term baby as
vigorous:
A. Color
B . Heart rate
C. Respiratory effort
D. Muscle tone
Ans. is A. Color
(Ref: O.P. Ghai 7th, Pg. 100)
Q 11. Early neonatal sepsis in india is most commonly due
to:
A. Escherichia coli
B . Group-B Stretococci
C. Staphylococci
D. Pseudomonas
Ans. is A. Escherichia coli
(Ref: O.P. Ghai 7th, Pg. 136)
Q 12. A neonate with recurrent infection and abscess was
diagnosed with Kostmann syndrome. Treament is:
A. G-CSF
B. GM-CSF
C. Antithymocyte globulin+cyclosporin
D. Antithymocyte globulin+cyclosporin +GM-CSF
Ans. is A. G-CSF
(Ref: Nelson 19th, Pg. 750)
Q 13. Aniridia is associated with:
A. Hepatoblastoma
B. Medulloblastoma
C. Nephroblastoma
D. Retinoblastoma
Ans. is C. Nephroblastoma
(Ref: Nelson 19th, Pg. 1758)

154

Pre-NEET Pediatrics

Q 14. Which of the following is associated with >20% risk of


chromosomal anomalies:
A. Cleft lip
B. Gastroschisis
C. Omphalocele
D. Spina bifida
Ans. is C. Omphalocele
(Ref: Avery 8th, Pg. 1114)
Q 15. A premature baby weighing 1.5 kg, born with
emergnancy C.S. at 32 weeks, now develops respiratory
distress with grunting the best management would be:
A. C-pap
B. Mechanical ventilation
C. Moist oxygen through headbox
D. Surfactant therapy plus mechanical ventilation
Ans. is D. Surfactant therapy plus mechanical ventilation
(Ref: Nelson 19th, Pg. 584)

PAEDIATRICS 2009
Q 1. Commonest cause of heart failure in infancy is:
A. Myocarditis
B. Rheumatic fever
C. Cardiomyopathy
D. Congenital heart disease
Ans is D. Congenital heart disease
(Ref: O.P. Ghai 7th, Pg. 372)
Q 2. Congenital long QT syndrome is associated with
neonatal:
A. Sinus bradycardia
B. Sinus tachycardia
C. Supra ventricular. tachycardia
D. Ventricular tachycardia
Ans. is A. Sinus bradycardia
(Ref: Nelson 19th, Pg. 1617)

Previous Years Questions of DNB

155

Q 3. Single gene defect causing multiple unrelated problems:


A. Pleiotropism
B. Pseudodominance
C. Penetrance
D. Anticipation
Ans. is A. Pleiotropism
Q 4. A newborn baby presented with profuse bleeding from
umbilical stump after birth. Probable diagnosis is:
A. Factor XIII deficiency
B. VWF deficiency
C. Factor XII deficiency
D. Glanzmann thrombosthenia
Ans. is A. Factor XIII deficiency
(Ref: Nelson 19th, Pg. 1704)
Q 5. Common to both acute and chronic malnutrition is:
A. Weight for age
C. Height for age
B. Weight for height
D. BMI
Ans. is A. Weight for age
(Ref: O.P. Ghai 7th, Pg. 62)
Q 6. Persistence of Moros reflex is abnormal beyond the age
of:
A. 3rd month
B. 4th month
C. 5th month
D. 6th month
Ans. is D. 6th month
(Ref: Forfar 6th, Pg. 311)
Q 7. Most common cause of renal artery stenosis in children
in India is:
A. Takayasu Aortoarteritis
B. Fibromedial hypertrophy

156

Pre-NEET Pediatrics

C. Fibrointimal hyperplasia
D. Polyarteritis Nodosa
Ans. is A. Takayasu Aortoarteritis
(Ref: Paediatric Nephrology 5th, Pg. 162)
Q 8. Drug of choice for Rheumatic fever prophylaxis in
penicillin allergic patient:
A. Erythromycin
B. Clindamycin
C. Vancomycin
D. Gentamycin
Ans. is A. Erythromycin
(Ref: Nelson 19th, Pg. 924)
Q 9. Common to both acute and chronic malnutrition is:
A. Weight for age
B. Weight of height
C. Height for age
D. BMI
Ans. is A. Weight for age
(Ref: O.P. Ghai 7th, Pg. 62)
Q 10. Essential criteria for TOF includes all, except:
A. Valvular stenosis
B. Infundibular stenosis
C. Over riding of aorta
D. RVH
Ans. is A. Valvular stenosis
(Ref: O.P. Ghai 7th, Pg. 408)
Q 11. All of the following are features of downs syndrome,
except:
A. Increased PAPPA
B. Increased free beta HCG levels
C. Absent nasal bone
D. Abnormal ductus venous flow velocity
Ans. is A. Increased PAPPA
(Ref: IAP textbook of Pediatrics 4th, Pg. 995)

Previous Years Questions of DNB

157

Q 12. Most common inherited childhood tumor is:


A. Leukemia
B. Neuroblastoma
C. Retinoblastoma
D. Wiulms tumor
Ans. is C. Retinoblastoma
Q 13. Turner syndrome is maximally associated with:
A. Horseshoe kidney
B. Coarctation of arota
C. VSD
D. ASD
Ans. is B. Coarctation of arota
Q 14. Most common sequelae to periventricular leukomalacia
is:
A. Spastic diplegia
B. Spastic quadriplegia
C. Hypotonia
D. Mental retardation
Ans. is A. Spastic diplegia
(Ref: O.P. Ghai 7th, Pg. 559)
Q 15. Neonate with recurrent infection and abscess diagnosed
of kostmann syndrome. Treatment include:
A. Anti thymocyte globulin + cyclosporin
B. Anti thymocyte globulin + cyclosporin + gm-csf
C. G-CSF
D. GM-CSF
Ans. is C. G-CSF
(Ref: Nelson 19th, Pg. 750)
Q 16. What constitutes Pentalogy of Fallot:
A. TOF + PDA
B. TOF + ASD
C. TOF+ COA
D. TOF + Polysplenia
Ans. is B. TOF + ASD

158

Pre-NEET Pediatrics

Q 17. Most sensitive indicator of intravascular volume in


infant is:
A. Cardiac output
B. Heart rate
C. Stroke volume
D. Preload
Ans. is B. Heart rate
(Ref: O.P. Ghai 7th, Pg. 697)
Q 18. Chang staging is for:
A. Retino blastoma
B. Rhabdo myosarcoma
C. Ewings sarcoma
D. Medulloblastoma
Ans. is D. Medulloblastoma
(Ref: Forfar 6th, Pg. 1115)
Q 19. Wilms tumour is associated with all except:
A. Aniridia
B. Hemihypertrophy
C. Hypertension
D. Bilateral polycystic Kidney
Ans. is D. Bilateral polycystic Kidney
(Ref: Nelson 19th, Pg. 1758)

PAEDIATRICS 2007
Q 1. Earliest indication of sexual maturation in a girl is:
A. Menarche
B. Pubarche
C. Thelarche
D. Maturation of breasts
Ans. is C. Thelarche
(Ref: O.P. Ghai 7th, Pg. 498)
Q 2. The first permanent teeth to erupt are usually the:
A. Lateral incisors
B. Central incisors

Previous Years Questions of DNB

159

C. Second molars
D. First molars
Ans. is D. First molars
(Ref: O.P. Ghai 7th, Pg. 4)
Q 3. Blood specimen for neonatal thyroid screening is
obtained on:
A. Cord blood
B. 24 hours after birth
C. 48 hours after birth
D. 72 hours after birth
Ans is C. 48 hours after birth
(Ref: O.P. Ghai 7th, Pg. 483)
Q 4. Treatment of choice for thalassemia major is:
A. Blood transfusion and iron therapy
B. Folic acid and desferrioxamine
C. Blood transfusion and desferrioxamine
D. Iron, blood transfusion and desferrioxamine
Ans. is C. Blood transfusion and desferrioxamine
(Ref: O.P.Ghai 7th, Pg. 309)
Q 5. One of the intestinal enzymes that is generally deficient
in children following an attack of severe infectious enteritis
is:
A. Lactase
B. Trypsin
C. Lipase
D. Amylase
Ans. is A. Lactase
(Ref: O.P. Ghai 7th, Pg. 266)
Q 6. Sure sign of CCF in a infant is:
A. Basal crepts
B. JVP
C. Pedal oedema
D. Liver enlargement
Ans. is D. Liver enlargement
(Ref: O.P. Ghai 7th, Pg. 375)

160

Pre-NEET Pediatrics

Q 7. Epiphyseal dysgenesis is a pathognomonic feature of:


A. Hypoparathyroidism
B. Hyperparathyroidism
C. Hypothyroidism
D. Hyperthyroidism
Ans. is C. Hypothyroidism
(Ref: Nelson 19th, Pg. 1899)
Q 9. Commonest cause of stridor in a new born is:
A. Laryngomalacia
B. Foreign body
C. Meconium aspiration
D. Recurrent laryngeal nerve palsy due to birth
Ans. is A. Laryngomalacia
(Ref: O.P. Ghai 7th, Pg. 340)
Q 9. Attainment of weight of a preschool normal child is:
A. 2-2.5 Kg
B. 3-3.5 Kg
C. 4-4.5 Kg
D. 5-5.5 Kg
Ans. is B. 3-3.5 Kg
(Ref: IAP textbook of Pediatrics 4th, Pg. 84)
Q 10. Precocious puberty is seen in:
A. Hyperthyroidism
B. Addisons disease
C. McCune Albright syndrome
D. Neuroblastoma
Ans. is C. McCune Albright syndrome
(Ref: Nelson 19th, Pg. 1892)
Q 11. The commonest cause of death in diphtheric child is:
A. IIIrd nerve palsy
B. Myocarditis
C. Tonsilitis
D. Septicemia
Ans. is B. Myocarditis
(Ref: IAP textbook of Pediatrics 4th, Pg. 364)

Previous Years Questions of DNB

161

Q 12. 15 months old child can do all, except:


A. Feeds self with spoon
B. Says three words
C. Builds tower of 2 blocks
D. Creeps upstairs
Ans. is A. Feeds self with spoon
(Ref: Nelson 19th, Pg 32)
Q 13. Best method of diagnosis of childhood HIV:
A. CD4 cell counts
B. P24 antigen
C. ELISA
D. Anti HIV antibody
Ans is B. P24 antigen
(Ref: Nelson 19th, Pg. 1167)
Q 14. Association of sexual precocity, multiple cystic bone
lesions and endocrinopathies are seen in:
A. McCune-Albrights syndrome
B. Granulosa cell tumor
C. Androblastoma
D. Hepatoblastoma
Ans. is A. McCune-Albrights syndrome
(Ref: Nelson 19th, Pg. 1892)
Q 15. Berger nepbropathy is due to mesangial deposition of:
A. Fibrin and C3
B. IgD and C3
C. IgE and C3
D. IgA and C3
Ans. is D. IgA and C3
(Ref: O.P. Ghai 7th, Pg. 446)
Q 16. Commonest cause of heart failure in infancy is:
A. Myocarditis
B. Rheumatic fever
C. Cardiomyopathy
D. Congenital heart disease
Ans. is D. Congenital heart disease
(Ref: O.P. Ghai 7th, Pg. 372)

162

Pre-NEET Pediatrics

Q 17. Congenital long QT syndrome is associated with


neonatal:
A. Sinus bradycardia
B. Sinus tachycardia
C. Supra ventricular tachycardia
D. Ventricular tachycardia
Ans. is A. Sinus bradycardia
(Ref: Nelson 19th, Pg. 1617)
Q 18. Pawn ball megakaryocytes are characteristic of:
A. Myelodysplastic syndrome
B. Idiopathic thrombocytopenic purpura
C. Thrombotic thrombocytopenic purpura
D. Chloramphenicol toxicity
Ans. is A. Myelodysplastic syndrome

PAEDIATRICS 2006
Q 1. Commonest cause of meningitis in postneonatal period
is:
A. Mycobacterium tuberculosis
B. Staph. aureus
C. Str. pneumonae
D. Klebsiella
Ans is C. Str. Pneumonae
(Ref: O.P. Ghai 7th, Pg. 536)
Q 2. Epiphyseal dysgenesis is a pathognomonic feature of:
A. Hypoparathyroidism
B. Hyperparathyroidism
C. Hypothyroidism
D. Hyperthyroidism
Ans. is C. Hypothyroidism
(Ref: Nelson 19th, Pg. 1899)
Q 3. In Downs syndrome, following congenital defect is
common:
A. PDA
B. PS

Previous Years Questions of DNB

163

C. ASD
D. VSD
Ans. is C. ASD
(Ref: IAP textbook of Pediatrics 4th, Pg. 994)
Q 4.Craniotabes is found in children with the following
conditions except:
A. Rickets
B. Hydrocephalus
C. Syphilis
D. Kernicterus
Ans. is D. Kernicterus
(Ref: Nelson 19th, Pg. 200)
Q 5. Commonest haematological malignancy in children is:
A. CLL
B. AML
C. CML
D. ALL
Ans. is D. ALL
(Ref: Nelson 19th, Pg. 1732)
Q 6. Commonest cause of stridor in a new born is:
A. Laryngomalacia
B. Foreign body
C. Meconium aspiration
D. Recurrent laryngeal nerve palsy due to birth
Ans. is A. Laryngomalacia
(Ref: O.P. Ghai 7th, Pg. 340

PAEDIATRICS 2005
Q 1. The most important cause of under 5 mortality is:
A. Diarrhoea
B. Malnutrition
C. Respiratory infections
D. Trauma
Ans. is C. Respiratory infections
(Ref: O.P. Ghai 7th, Pg. 356)

164

Pre-NEET Pediatrics

Q 2. Commonest cause of stridor in a new born is:


A. Laryngomalacia
B. Foreign body
C. Meconium aspiration
D. Recurrent laryngeal nerve palsy due to birth
Ans. is A. Laryngomalacia
(Ref: O.P. Ghai 7th, Pg. 340)
Q 3. 15 months old child do all, except:
A. Feeds self with spoon
B. Says three words
C. Builds tower of 2 blocks
D. Creeps upstairs
Ans. is A. Feeds self with spoon
(Ref: Nelson 19th, Pg. 32)
Q 4. Commonest nephrotic syndrome in child:
A. Minimal change
B. Chronic glomerulonephritis
C. Hemolytic uremic syndrome
D. Congenital
Ans. is A. Minimal change
(Ref: O.P. Ghai 7th, Pg. 451)
Q 5. A six years old girl child presents with spotting, no
secondary sexual characteristic present, cause can be:
A. Menarche
B. Foreign body
C. Gonococcal infection
D. Haemorrhagic disease
Ans. is B. Foreign body
(Ref: Forfar 6th, Pg. 1152)
Q 6. Best method of diagnosis of childhood HIV.
A. CD4 cell counts
B. P24 antigen
C. ELISA
D. Anti HIV antibody
Ans. is B. P24 antigen
(Ref: Nelson 19th, Pg. 1167)

Previous Years Questions of DNB

165

Q 7. Attainment of weight of a preschool normal child is:


A. 2-2.5 Kg
B. 3-3.5 Kg
C. 4-4.5 Kg
D. 5-5.5 Kg
Ans. is B. 3-3.5 Kg
(Ref: IAP textbook of Pediatrics 4th, Pg.84)

PAEDIATRICS 2004
Q 1. Craniotabes is found in children with the following
conditions except:
A. Rickets
B. Hydrocephalus
C. Syphilis
D. Kernicterus
Ans. is D. Kernicterus
(Ref: Nelson 19th, Pg. 200)
Q 2. In minimal change disease, correct is:
A. Most cases recover spontaneously
B. 20% go to CRF
C. 20% develop local glomerulosclerosis
D. 90% cases best respond to short course of steroid therapy
Ans. is D. 90% cases best respond to short course of steroid therapy
(Ref: Nelson 19th, Pg. 1803)
Q 3. Commonest haematological malignancy in children is:
A. CLL
B. AML
C. CML
D. ALL
Ans. is D. ALL
(Ref: Nelson 19th, Pg. 1732)
Q 4. Metabolic acidosis is accompanied with:
A. Acetazolamide
B. Phenformin

166

Pre-NEET Pediatrics

C. Verapamil
D. Triamterene
Ans. is A. Acetazolamide
(Ref: O.P. Ghai 7th, Pg. 55)
Q 5. The commonest cause of death in diphtheric child is:
A. IIIrd nerve palsy
B. Myocarditis
C. Tonsillitis
D. Septicemia
Ans. is B. Myocarditis
(Ref: IAP textbook of Pediatrics 4th, Pg. 364)
Q 6. Unconjugated hyperbilirubinemia in newborn is caused
by following, except:
A. Breast milk jaundice
B. Galactosemia
C. Sphereocytosis
D. Gilberts syndrome
Ans. is B. Galactosemia
(Ref: Nelson 19th, Pg. 1376)
Q 7. Commonest cause of meningitis in postneonatal period
is:
A. Mycobacterium tuberculosis
B. Staph. aureus
C. Str. pneumoniae
D. Klebsiella
Ans. is C. Str. Pneumonia
(Ref: Nelson 19th, Pg. 2087)
Q 8. Sure sign of CCF in a infant is:
A. Basal crepts
B. JVP
C. Pedal oedema
D. Liver enlargement
Ans. is D. Liver enlargement
(Ref: O.P. Ghai 7th, Pg. 375)

Previous Years Questions of DNB

167

PAEDIATRICS 2003
Q 1. Epiphyseal dysgenesis is a pathognomonic feature of:
A. Hypoparathyroidism
B. Hyperparathyroidism
C. Hypothyroidism
D. Hyperthyroidism
Ans. is C. Hypothyroidism
(Ref: Nelson 19th, Pg. 1899)
Q 2. Commonest cause of systemic hypertension in children
is:
A. Coarctation of aorta
B. Acute glomerulonephritis
C. Nephrotic syndrome
D. Lactic acidosis
Ans. is B. Acute glomerulonephritis
(Ref: O.P. Ghai 7th, Pg. 434)
Q 3. Organism in bronchiolitis is:
A. Adeno virus
B. Influenza virus
C. Rhino virus
D. RSV
Ans. is D. RSV
(Ref: Nelson 19th, Pg.1456)
Q 4. Steroids is useful in:
A. Post-streptococcal glomerulonephritis
B. Membranous glomerulonephritis
C. Rapidly progressing glomerulonephritis
D. Minimal change type
Ans. is D. Minimal change type
(Ref: Nelson 19th, Pg. 1803)
Q 5. Not seen in Fallots tetralogy:
A. ASD
B. VSD

168

Pre-NEET Pediatrics

C. Pulmonary stenosis
D. Left ventricular hypertrophy
Ans. is A. ASD
(Ref: Nelson 19th, Pg. 1573)
Q 6. In Downs syndrome, following congenital defect is
common:
A. PDA
B. PS
C. ASD
D. VSD
Ans. is C. ASD
(Ref: IAP textbook of Pediatrics 4th, Pg. 994)
Q 7. Following are causes of Pan systolic murmur, except:
A. MR
B. MS
C. VSD
D. TR
Ans. is B. MS
(Ref: Nelson 19th, Pg. 1556, 1627, 1628)
Q 8. Precocious puberty is seen in:
A. Hyperthyroidism
B. Addisonsdisease
C. McCune Albright syndrome
D. Neuroblastoma
Ans. is C. McCune Albright syndrome
(Ref: Nelson 19th, Pg. 1892)
Q 9. Craniotabes is found in children with the following
conditions, except:
A. Rickets
B. Hydrocephalus
C. Syphilis
D. Kernicterus
Ans. is D. Kernicterus
(Ref: Nelson 19th, Pg. 1072)

Previous Years Questions of DNB

169

PAEDIATRICS 2002
Q 1. The commonest cause of death in diphtheric child is:
A. IIIrd-nerve palsy
B. Myocarditis
C. Tonsillitis
D. Septicemia
Ans. is B. Myocarditis
(Ref: IAP textbook of Pediatrics 4th, Pg. 364)
Q 2. Best method of diagnosis of childhood HIV:
A. CD 4 cell counts
B. P24 antigen
C. ELISA
D. Anti HlV antibody
Ans. is B. P24 antigen
(Ref: Nelson 19th, Pg. 1167)
Q 3. Hyaline membrane is seen in all of the following
conditions, except:
A. Radiation pneumonitis
B. Viral pneumonitis
C. Uremic pneumonitis
D. Staphylococcal bronchopneumonia
Ans. is D. Staphylococcal bronchopneumonia
Q 4. Attainment of weight is a preschool normal child is:
A. 2-2.5 Kg
B. 3-3.5 Kg
C. 4-4.5 Kg
D. 5-5.5 Kg
Ans. is B. 3-3.5 Kg
(Ref: IAP textbook of Pediatrics 4th, Pg. 84)
Q 5. Subacute sclerosing panencephalit is complication of:
A. Pneumonia
B. Measles

170

Pre-NEET Pediatrics

C. Diphtheria
D. Pertussis
Ans. is B. Measles
(Ref: Nelson 19th, Pg. 1072)
Q 6. Casonis test is diagnostic of:
A. Echinococcus granulosum
B. Toxoplasmosis
C. Toxocariasis
D. Syphilis
Ans. is A. Echinococcus granulosum

PAEDIATRICS 2001
Q 1. A child with diarrhoea has deep and rapid respiration.
Diagnosis is:
A. Metabolic alkalosis
B. Metabolic acidosis
C. Respiratory alkalosis
D. Respiratory acidosis
Ans. is B. Metabolic acidosis
(Ref: Nelson 19th, Pg. 231)
Q 2. The percentage rise in length of infant in first year of life
is:
A. 20%
B. 30%
C. 40%
D. 50%
Ans. is D. 50%
(Ref: IAP textbook of paediatrics 4th, Pg. 84)
Q 3.The most important cause of under 5 mortality is:
A. Diarrhoea
B. Malnutrition
C. Respiratory infections
D. Trauma
Ans. is C. Respiratory infections
(Ref: O.P. Ghai 7th, Pg. 350)

Previous Years Questions of DNB

171

Q 4. A neonate after 12 hrs. of birth passes black colored


meconium. True is:
A. Intestinal haemorrhage
B. Fibrocystic disease of pancreas
C. Normal finding
D. Hirschsprungs disease
Ans. is C. Normal finding
(Ref. O.P. Ghai 7th, Pg. 105)
Q 5. All of the following are used to assess IUGR, except:
A. Fetal movements
B. Head size
C. Fundus height
D. Liquor volume
Ans. is A. Fetal movements
(Ref: Avery 8th, Pg. 36)
Q 6. Sure sign of CCF in an infant is:
A. Basal crepts
B. JVP
C. Pedal oedema
D. Liver enlargement
Ans. is D. Liver enlargement
(Ref: O.P. Ghai 7th, Pg. 375)
Q 7.Commonest cause of stridor in a new born is:
A. Laryngomalacia
B. Foreign body
C. Meconium aspiration
D. Recurrent laryngeal nerve palsy due to birth
Ans. is A. Laryngomalacia
(Ref: Nelson 19th, Pg. 1450)
Q 8. 15 months old child do all, except:
A. Feeds self with spoon
B. Says three words
C. Builds tower of 2 blocks
D. Creeps upstairs
Ans. is A. Feeds self with spoon
(Ref: Nelson 19th, Pg. 32)

172

Pre-NEET Pediatrics

Q 9. At what age does a child sees the toy hidden and then
again hides it and gives to mother if she asks for:
A. 6 months
B. 8 months
C. 10 months
D. 12 months
Ans. is D. 12 months
(Ref: Forfar 6th, Pg. 119)
Q 10. Commonest nephrotic syndrome in child:
A. Minimal change
B. Chronic glomerulonephritis
C. Hemolytic uremic syndrome
D. Congenital
Ans. is A. Minimal change
(Ref: Nelson 19th, Pg. 1804)
Q 11. Commonest haematological malignancy in children
is:
A. CLL
B. AML
C. CML
D. ALL
Ans. is D. ALL
(Ref: Nelson 19th, Pg. 1732)
Q 12. A six years old girl child presents with spotting, no
secondary sexual characteristic present. Cause can be:
A. Menarche
B. Foreign body
C. Gonococcal infection
D. Haemorrhagic disease
Ans. is B. Foreign body
(Ref: For far 6th, Pg. 1152)
Q 13. Metabolic acidosis is accompanied with:
A. Acetazolamide
B. Phenformin

Previous Years Questions of DNB

173

C. Verapamil
D. Triamterene
Ans. is A. Acetazolamide
(Ref: O.P. Ghai 7th, Pg. 55

PAEDIATRICS 2000
Q 1. Not seen in Fallots tetralogy:
A. ASD
B. VSD
C. Pulmonary stenosis
D. Left ventricular hypertrophy
Ans. is A. ASD
(Ref: Nelson 19th, Pg. 1573)
Q 2. Congestive heart failure in children is best diagnosed
by:
A. Tachycardia and tender hepatomegaly
B. JVP
C. JVP + pedal edema
D. Hypotension
Ans. is A. Tachycardia and tender hepatomegaly
(Ref: O.P. Ghai 7th, Pg. 375)
Q 3. APGAR score of a child born blue with HR 70/ mt. floppy
with feeble cry with grimacing on nasal suction is:
A. 3
B. 2
C. 4
D. 5
Ans is A. 3
(Ref: Nelson 19th, Pg. 536)
Q 4. In Downs syndrome, following congenital defect is
common:
A. PDA
B. PS

174

Pre-NEET Pediatrics

C. ASD
D. VSD
Ans. is C. ASD
(Ref: IAP textbook of paediatrics 4th, Pg. 994)
Q 5. Following is cause of pan systolic murmur:
A. MR
B. MS
C. VSD
D. ASD
E. TR
Ans. is A. MR, C. VSD & E. TR
(Ref: Nelson 19th, Pg. 1556, 1627, 1628)
Q 6. In hyperparathyroidism, seen is:
Ca++

PO4-

A.

B.

C.

D.

Ans is A.
(Ref: Nelson 19th, Pg. 1921)
Q 7. Precocious puberty is seen in:
A. Hyperthyroidism
B. Addisons disease
C. McCune Albright syndrome
D. Neuroblastoma
Ans. is C. McCune Albright syndrome
(Ref: Nelson 19th, Pg. 1892)

Das könnte Ihnen auch gefallen